financial reporting and analysis_the income statement, balance sheet, and cash flow statement

203

Upload: n-s

Post on 31-Oct-2015

3.235 views

Category:

Documents


5 download

DESCRIPTION

Financial Reporting and Analysis_The Income Statement, Balance Sheet, and Cash Flow Statement

TRANSCRIPT

Page 1: Financial Reporting and Analysis_The Income Statement, Balance Sheet, and Cash Flow Statement
Page 2: Financial Reporting and Analysis_The Income Statement, Balance Sheet, and Cash Flow Statement
Page 3: Financial Reporting and Analysis_The Income Statement, Balance Sheet, and Cash Flow Statement

Which of the following statements about the indirect method of calculating the statement of cash flows is NOT correct?

Question 8 - #96543

Which of the following ratios is NOT part of the original DuPont system?

Question 9 - #96767

Given the following income statement and balance sheet for a company:

Which of the following is closest to the company's return on equity (ROE)?

A) No adjustment is needed to account for changes in accounts receivable since no cash was involved.

B) No adjustment is needed to account for extraordinary items because they are found above net income and are thus already accounted for.

C) An adjustment is needed for the payment of deferred taxes.

A) Asset turnover.B) Equity multiplier.C) Debt to total capital.

Balance Sheet Assets Year 2006 Year 2007Cash 200 450Accounts Receivable 600 660Inventory 500 550Total CA 1300 1660Plant, prop. equip 1000 1580Total Assets 2600 3240LiabilitiesAccounts Payable 500 550Long term debt 700 1052Total liabilities 1200 1602EquityCommon Stock 400 538Retained Earnings 1000 1100Total Liabilities & Equity 2600 3240

Income Statement Sales 3000Cost of Goods Sold (1000)Gross Profit 2000SG&A 500Interest Expense 151EBT 1349Taxes (30%) 405Net Income 944

A) 0.29.B) 1.83.

www.LotBook.net

[email protected]

Page 4: Financial Reporting and Analysis_The Income Statement, Balance Sheet, and Cash Flow Statement

Question 10 - #98138

Are the following essential characteristics of an asset?

Question 11 - #97362

If Jackson Ski Company issues common stock, and uses the proceeds to purchase fixed assets such as equipment:

Question 12 - #93917

Which of the following items is least appropriately described as a liability arising from an operating activity for a non-financial company?

Question 13 - #97449

Financial information for Jefferson Corp. for the year ended December 31st, was as follows:

Based upon this data and using the direct method, what was Jefferson Corp.’s cash flow from operations (CFO) for the year ended December 31st?

C) 0.62.

Characteristic #1: The asset provides future economic benefits as a result of past transactions. Characteristic #2: The asset is tangible and is obtained at a cost.

Characteristic #1 Characteristic #2

A) Yes Yes

B) No No

C) Yes No

A) both cash flow from operations and cash flow from financing would increase.B) cash flow from financing would increase and cash flow from investing would decrease. C) cash flow from financing would decrease and cash flow from investing would increase.

A) Cash advances from customers.B) Trade payables.C) The current portion of long-term debt.

Sales $3,000,000Purchases 1,800,000Inventory at Beginning 500,000Inventory at Ending 800,000Accounts Receivable at Beginning 300,000Accounts Receivable at Ending 200,000Other Operating Expenses Paid 400,000

www.LotBook.net

[email protected]

Page 5: Financial Reporting and Analysis_The Income Statement, Balance Sheet, and Cash Flow Statement

Question 14 - #97391

John Stone, CFA, is an investment advisor specializing in the preparation of company and industry reports for high net worth customers at Learmon Brothers. Currently, Stone is preparing a report on Soft Corporation, a rapidly growing software company. The explosive growth of this company was financed primarily by an initial public offering in which 3,000,000 shares were issued at a price of $20 per share on June 27, 2004. Soft Corporation received additional capital when employee stock options for 1,000,000 shares at a price of $10 were exercised on January 1, 2005. Stone realizes the importance of cash flow on a company's financial health and would like to include a projected statement of cash flows for 2005. Soft Corporation financial statements are presented in Tables 1 and 2. Included are the actual statements for the year ending December 31, 2004.

Under the indirect method, what will Stone find Soft Corporation's projected net change in cash to be for the year ending December 31, 2005?

A) $1,200,000.B) $800,000.C) $900,000.

Table 1 Soft Corporation Balance Sheets

as of December 31 (in millions)

Actual 2004 Projected 2005Cash $24.0 $26.0 Accounts Receivable 17.0 24.0 Inventory 100.0 150.0 PP&E 100.0 125.0 Accumulated depreciation (30.0) (35.0)Total Assets $211.0 $290.0

Accounts payable $91.0 $101.0 Long-term debt 20.0 40.0 Common stock 80.0 90.0 Retained earnings 20.0 59.0Total liabilities and equity $211.0 $290.0

Table 2 Soft Corporation Income Statement

for Years Ended December 31 (in millions except per share data)

Actual 2004 Projected 2005Sales $80.0 $198.0 COGS (38.0) (90.0)Gross profit $42.0 $108.0

SG&A (13.0) (30.0) Depreciation (3.0) (5.0)Operating expenses $(16.0) $(35.0)

Interest expense $(4.0) $(5.0) Pretax Income 22.0 68.0 Income tax expense (7.0) (25.0)Net income $15.0 $43.0 EPS $2.0 $4.3 Average shares outstanding (millions) 7.5 10.0 Dividends per share $0.1 $0.4

www.LotBook.net

[email protected]

Page 6: Financial Reporting and Analysis_The Income Statement, Balance Sheet, and Cash Flow Statement

Question 15 - #97345

Coleman Corporation’s unadjusted trial balance at the end of 2007 reflected compensation expense of $90 million. The trial balance did not include the following:

Because of the holidays, no salary accrual was made for the last week of the year. Salaries for the last week totaled $3.5 million and were paid on January 4, 2008.

Employee bonuses for 2007 totaled $5 million. The bonuses were paid on January 31, 2008.

Ignoring payroll taxes, what is Coleman’s adjusted compensation expense for the year ended 2007 and what impact will the adjustment have on Coleman’s 2007 current ratio?

Question 16 - #97939

Given the following information about a firm:

Net Sales = $1,000. Cost of Goods Sold = $600. Operating Expenses = $200. Interest Expenses = $50. Tax Rate = 34%.

What are the gross and operating profit margins?

Question 17 - #98050

According to the installment method of accounting, gross profit on an installment sale is recognized:

A) $9,000,000.B) $4,000,000.C) $7,000,000.D)E)F) $2,000,000.

Compensation expense Current ratio

A) $94.5 million Decrease

B) $98.5 million No effect

C) $98.5 million Decrease

Gross Operating Margin Operating Profit Margin

A) 40% 10%

B) 20% 15%

C) 40% 20%

www.LotBook.net

[email protected]

Page 7: Financial Reporting and Analysis_The Income Statement, Balance Sheet, and Cash Flow Statement

Question 18 - #94690

This information pertains to equipment owned by Brigade Company.

Cost of equipment: $10,000.

Estimated residual value: $2,000.

Estimated useful life: 5 years.

Depreciation method: straight-line.

The accumulated depreciation at the end of year 3 is:

Question 19 - #98010

Which of the following statements regarding the income statement is least accurate?

Question 20 - #97688

In preparing a forecast of future financial performance, which of the following best describes sensitivity analysis and scenario analysis, respectively?

Description #1 – A computer generated analysis based on developing probability distributions of key variables that are used to drive the potential outcomes.

Description #2 – The process of analyzing the impact of future events by considering multiple key variables.

Description #3 – A technique whereby key financial variables are changed one at a time and a range of possible outcomes are observed. Also known as “what-if” analysis.

A) after cash collections equal to the cost of sales have been received.B) on the date the final cash collection is received.C) in proportion to the cash collection.

A) $1,600. B) $5,200. C) $4,800.

A) Extraordinary items are both unusual in nature and infrequent in occurrence. Extraordinary items are disclosed net of taxes after income from continuing operations in the income statements.

B) Items that are unusual in nature or infrequent in occurrence appear below income from continuing operations on a pretax basis.

C) The results of discontinued operations are reported below income from continuing operations on the income statement net of taxes.

Sensitivity analysis Scenario analysis

A) Description #3 Description #2

B) Description #3 Description #1

www.LotBook.net

[email protected]

Page 8: Financial Reporting and Analysis_The Income Statement, Balance Sheet, and Cash Flow Statement

Question 21 - #97360

In converting a statement of cash flows from the indirect to the direct method, which of the following adjustments should be made for a decrease in unearned revenue when calculating cash collected from customers, and for an inventory writedown (when market value is less than cost) when calculating cash payments to suppliers?

Question 22 - #94016

The actual coupon payment on a bond is reported on the statement of cash flow as:

Question 23 - #98016

The First National Bank is a commercial bank that specializes in consumer financing, particularly automobile loans. The majority of the loans are funded from customer deposits. In addition, the bank purchases various investment securities with available cash. The investments are debt securities and have an average maturity date of less than 30 days. Should First National Bank report the interest received from the consumer loans and the interest received from the investment securities as an operating or as a nonoperating component in its year-end income statement?

Question 24 - #98038

The following data pertains to the Sapphire Company:

Net income equals $15,000. 5,000 shares of common stock issued on January 1st. 10% stock dividend issued on June 1st.

C) Description #2 Description #3

Cash collections from customers: Cash payments to suppliers:

A) Subtract decrease in unearned revenue Add an inventory writedown

B) Add decrease in unearned revenue Subtract an inventory writedown

C) Subtract decrease in unearned revenue Subtract an inventory writedown

A) a financing cash outflow.B) an operating cash outflow.C) an investing cash outflow.

Consumer loans Investment securities

A) Operating Operating

B) Operating Nonoperating

C) Nonoperating Operating

www.LotBook.net

[email protected]

Page 9: Financial Reporting and Analysis_The Income Statement, Balance Sheet, and Cash Flow Statement

1,000 shares of common stock were repurchased on July 1st.

1,000 shares of 10%, $100 par preferred stock each convertible into 8 shares of common were outstanding the whole year.

What is the company’s diluted earnings per share (EPS)?

Question 25 - #97771

All of the following are considered a potentially dilutive securities EXCEPT:

Question 26 - #97376

An examination of the cash receipts and payments of Xavier Corporation reveals the following:

Under U.S. GAAP, Xavier's cash flow from financing (CFF) and cash flow from investing (CFI) will be:

Question 27 - #97076

Given the following income statement:

A) $2.50.B) $1.15.C) $1.00.

A) preferred stock.B) stock options.C) warrants.

Cash paid to suppliers for purchase of merchandise $5,000Cash received from customers 14,000Cash paid for purchase of equipment 22,000Dividends paid 2,000 Cash received from issuance of preferred stock 10,000Interest received on short-term investments 1,000 Wages paid 4,000 Repayment of loan to the bank 5,000 Cash from sale of land 12,000

CFF CFI

A) $10,000 $12,000

B) $3,000 -$10,000

C) $3,000 $12,000

Net Sales 200Cost of Goods Sold 55Gross Profit 145Operating Expenses 30

www.LotBook.net

[email protected]

Page 10: Financial Reporting and Analysis_The Income Statement, Balance Sheet, and Cash Flow Statement

What are the interest coverage ratio and the net profit margin?

Question 28 - #97252

Earlier this year, Ponca Corporation purchased non-dividend paying equity securities which it classified as trading securities. Information related to the securities follows:

What amounts should Ponca report in its year-end income statement and balance sheet as a result of its investment in securities X and Y?

Question 29 - #97915

A firm’s financial statements reflect the following:

Based on this information and assuming that the firm’s debt has a cost of 9% and has been outstanding for a full

Operating Profit (EBIT) 115Interest 15Earnings Before Taxes (EBT) 100

Taxes 40Earnings After Taxes (EAT) 60

Interest Coverage Ratio Net Profit Margin

A) 2.63 0.30

B) 7.67 0.30

C) 0.57 0.56

Security Cost Fair value at year-end X $400,000 $435,000 Y $550,000 $545,000

Income Statement Balance Sheet

A) $30,000 unrealized gain $950,000

B) No gain or loss $980,000

C) $30,000 unrealized gain $980,000

EBIT $2,000,000 Sales $16,000,000 Interest expense $900,000 Total assets $18,400,000 Equity $7,000,000 Effective tax rate 35% Dividend rate 28% Current liabilities $1,400,000

www.LotBook.net

[email protected]

Page 11: Financial Reporting and Analysis_The Income Statement, Balance Sheet, and Cash Flow Statement

year, what is the firm’s total debt ratio and interest coverage ratio?

Question 30 - #97968

On December 31, 2004, JME Corporation had 350,000 shares of common stock outstanding. On September 1, 2005, an additional 150,000 shares of common stock were issued. In addition, JME had $10 million of 8% convertible bonds outstanding at December 31, 2004, which are convertible into 200,000 shares of common stock. Net income for 2005 was $3 million. Assuming an income tax rate of 40%, what amount should be reported as the diluted earnings per share for 2005?

Question 31 - #97986

Ajax Company's capital structure was as follows:

During 2004, Ajax paid dividends of $2.00 per share on its preferred stock. The preferred shares are convertible into 10,000 shares of common stock. The 6% bonds are convertible into 15,000 shares of common stock. Net income for 2004 was $400,000. Assume that income tax rate is 40%.

Ajax’s basic and diluted earnings per share for 2004 are:

Question 32 - #97923

XYZ, Inc., latest Income Statement, Balance Sheet and Statement of Cash Flows are below. Use this information to answer the following questions:

Total Debt Ratio Interest Coverage Ratio

A) 0.90 1.80

B) 0.62 2.22

C) 0.70 0.96

A) $5.80.B) $5.00. C) $6.00.

December 31, 2004 December 31, 2003Outstanding shares of stock: Common 200,000 200,000 Convertible preferred 5,000 5,000 6% Convertible Bonds $500,000 $500,000

Basic EPS Diluted EPS

A) $1.95 $1.95

B) $1.95 $1.86

C) $1.80 $1.86

www.LotBook.net

[email protected]

Page 12: Financial Reporting and Analysis_The Income Statement, Balance Sheet, and Cash Flow Statement

Income Statement Sales Revenue 19,580Cost of Goods Sold 7,319

Gross Margin 12,261Wage Expense 900SG&A 4,336Depreciation Expense 662

5,898Income from Operations 6,363

Other Income/ExpensesInterest Expense (750)

Gain on Sale of Land 119(631)

Pretax Income 5,732Income tax 1,605Net Income 4,127

Balance Sheet 12/31/04 12/31/03

AssetsCurrent AssetsCash 2,098 410Accounts receivable 4,570 4,900Inventory 4,752 4,500Prepaid SGA 877 908Total 12,297 10,718Land 0 4,000Property, Plant & Equipment 11,000 11,000Accumulated Depreciation (5,862) (5,200)Total Assets 17,435 20,518

Cash Flow from Operations Net Income 4,127Increase in Accounts Receivable 330Increase in Accounts Payable (489)Increase in Inventory (252)Increase in Wages Payable 94Increase in Prepaid SGA 31Depreciation 662Gain on Sale of Land (119)Net cash from Operations 4,384Cash Flow from InvestmentsSale of Land 4,119Net Cash from Investments 4,119Cash Flow from FinancingRetirement of LT Debt (6,042)Dividends Paid (773)Net Cash from Financing (6,815)Net Increase in Cash 1,688

Page 13: Financial Reporting and Analysis_The Income Statement, Balance Sheet, and Cash Flow Statement

Part 1) At the end of 2004, what were XYZ’s current, quick and cash ratios?

Part 2) What was the return on equity (ROE) based on year-end equity?

Question 33 - #97735

An analyst has gathered the following information about a company:

Beginning Cash 410Ending Cash 2,098

Liabilities and Equity 12/31/04 12/31/03

Current LiabilitiesAccounts Payable 4,651 5,140Wages Payable 2,984 2,890Dividends Payable 100 100Total 7,735 8,130Long term Debt 1,346 7,388EquityCommon Stock 4,000 4,000Retained Earnings 4,354 1,000Total Liabilities and Equity 17,435 20,518

Current Ratio Quick Ratio Cash Ratio

A) 1.59 1.59 0.27

B) 1.48 0.86 0.27

C) 1.59 0.86 0.27

A) 0.49.B) 0.67.C) 0.58.

Balance Sheet AssetsCash 100Accounts Receivable 750Marketable Securities 300Inventory 850Property, Plant & Equip 900Accumulated Depreciation (150)

Total Assets 2750

Liabilities and EquityAccounts Payable 300Short-Term Debt 130

Page 14: Financial Reporting and Analysis_The Income Statement, Balance Sheet, and Cash Flow Statement

What is the quick ratio?

Question 34 - #122499

Ratio analysis is most useful for comparing companies:

Question 35 - #98083

Which, if any, of the following statements about the installment sales method and cost recovery method is correct?

Statement 1: The cost recovery method recognizes revenue and associated costs of goods sold only when cash is received, based on gross profit margin.

Statement 2: The installment sales method recognizes sales when cash is received, but no gross profit is recognized until all of the cost of goods sold is collected.

Question 36 - #97972

The following information pertains to Bender, Inc., for last year:

Net income of $25 million.

Long-Term Debt 700Common Stock 1000Retained Earnings 620

Total Liab. and Stockholder's equity 2750

Income Statement Sales 1500COGS 1100Gross Profit 400SG&A 150Operating Profit 250Interest Expense 25Taxes 75Net Income 150

A) 1.53.B) 2.67.C) 0.62.

A) in different industries that use the same accounting standards.B) of different size in the same industry.C) that operate in multiple lines of business.

A) Only one of these statements is correct.B) Both statements are correct.C) Neither statement is correct.

Page 15: Financial Reporting and Analysis_The Income Statement, Balance Sheet, and Cash Flow Statement

1 million shares of $10 par value preferred stock outstanding paying a 10% dividend. 50 million shares of common stock outstanding at the beginning of the year. Issued an additional 5 million shares of common stock on 7/1.

What is Bender, Inc.’s basic earnings per share (EPS)?

Question 37 - #97974

Connecticut, Inc.’s stock transactions during the year 20X5 were as follows:

January 1: 360,000 common shares outstanding. April 1: 1 for 3 reverse stock split. July 1: 60,000 common shares issued.

When computing for earnings per share (EPS) computation purposes, what is Connecticut’s weighted average number of shares outstanding during 20X5?

Question 38 - #97835

The Allen Corporation had 100,000 shares of common stock outstanding at the beginning of the year. Allen issued 30,000 shares of common May 1. On July 1, the company issued a 10% stock dividend. On September 1, Allen issued 1,000, 10% bonds convertible into 21 shares of stock each. What is the weighted average number of shares to be used in computing basic and diluted earnings per share (EPS), assuming the convertible bonds are dilutive?

Question 39 - #97907

Are the following ratios best classified as profitability ratios?

Ratio #1 – Cash plus short-term marketable investments plus receivables divided by average daily cash expenditures.

Ratio #2 – Earnings before interest and taxes divided by average total assets.

A) $0.476.B) $0.457.C) $0.384.

A) 210,000.B) 140,000.C) 150,000.

Basic Shares Diluted Shares

A) 130,000 132,000

B) 132,000 146,000

C) 132,000 139,000

A) Only one of the ratios is a profitability ratio.B) Both of the ratios are profitability ratios.

Page 16: Financial Reporting and Analysis_The Income Statement, Balance Sheet, and Cash Flow Statement

Question 40 - #97330

Galaxy, Inc.’s U.S. GAAP balance sheet as of December 31, 2004 included the following information (in $):

Galaxy’s net income in 2004 was $800,000. What was Galaxy’s cash flow from financing (CFF) in 2004?

Question 41 - #97299

The following information is from the balance sheet of Silverstone Company:

Net Income for 5/1/05 to 5/31/05: $8,000

Using the indirect method, calculate the cash flow from operations for Silverstone Company as of 5/31/05:

Question 42 - #97999

Assume that the exercise price of an option is $5, and the average market price of the stock is $8. Assuming 816 options are outstanding during the entire year, what is the number of shares to be added to the denominator of the diluted EPS?

C) Neither of the ratios is a profitability ratio.

12-31-03 12-31-04 Accounts Payable 300,000 500,000Dividends Payable 200,000 300,000Common Stock 1,000,000 1,000,000Retained Earnings 700,000 1,000,000

A) -$400,000.B) -$500,000.C) -$300,000.D) -$700,000.E)F)

Balance 5/01/05 Account Balance 5/31/05$2,000 Inventory $1,750$1,200 Prepaid exp. $1,700$800 Accum. Depr. $975$425 Accounts payable $625$650 Bonds payable $550

A) Increase in cash of $8,025. B) Increase in cash of $7,725. C) The indirect method cannot be calculated from the information provided. D) Increase in cash of $8,125. E)F)

A) 510.

Page 17: Financial Reporting and Analysis_The Income Statement, Balance Sheet, and Cash Flow Statement

Question 43 - #97119

The following footnote appeared in Crabtree Company’s 20X7 annual report:

“On December 31, 20X7, Crabtree recognized a restructuring charge of $20 million, of which $5 million was for severance pay for employees who will be terminated in 20X8 and $15 million was for land that became permanently impaired in 20X7.”

Based only on these changes, Crabtree’s net profit margin and fixed asset turnover ratio in 20X8 as compared to 20X7 will be?

Question 44 - #98017

Assume that the exercise price of an option is $9, and the average market price of the stock is $12. Assuming 992 options are outstanding during the entire year, what is the number of shares to be added to the denominator of the Diluted EPS?

Question 45 - #97358

The RR Corporation had cash flow from operations of $20 million. RR purchased $5 million in equipment and sold $3 million of equipment during the period. What is RR's free cash flow to equity for the period?

Question 46 - #95629

Which of the following items is NOT found in the financing cash flow part of the statement of cash flows?

B) 306.C) 816.

Net profit margin Fixed asset turnover

A) Higher Higher

B) Lower Higher

C) Higher Unchanged

A) 744.B) 248.C) 992.

A) $18 million.B) $15 million.C) $22 million.

A) Change in retained earnings. B) Change in long-term debt. C) Dividends paid.

Page 18: Financial Reporting and Analysis_The Income Statement, Balance Sheet, and Cash Flow Statement

Question 47 - #97476

If the inventory turnover ratio is 7, what is the average number of days the inventory is in stock?

Question 48 - #97385

Johnson Corp. had the following financial results for the fiscal 2004 year:

The only current assets are cash, accounts receivable, and inventory. The balance in these accounts has remained constant throughout the year. Johnson’s net sales for 2004 were:

Question 49 - #93821

Which of the following is NOT a cash flow from operation?

Question 50 - #98025

Walker Company received a letter in November 2003 indicating that Johnson, Inc. would purchase a specialty machine priced at $4,000,000. In February 2004, a binding contract was executed for the machine’s construction. Materials costing $2,000,000 were ordered in December 2003, arrived with an invoice in August 2004, and were used in the manufacturing process in the first quarter of 2005. Walker completed and delivered the machine in December 2006. Johnson received the first invoice in 2007 and paid the $4,000,000 purchase price in 2007. Walker Company uses the accrual method of accounting. Walker should record the materials used to construct the machine as expenses in the year:

A) 70 days.B) 52 days.C) 25 days.

Current ratio 2.00 Quick ratio 1.25 Current liabilities $100,000 Inventory turnover 12 Gross profit % 25

A) $300,000. B) $900,000. C) $1,200,000.

A) interest payments.B) dividends paid to shareholders.C) dividends received.

A) 2007.B) 2004.C) 2006.

Page 19: Financial Reporting and Analysis_The Income Statement, Balance Sheet, and Cash Flow Statement

Question 51 - #98032

JME Construction always uses the percentage of completion method of recognizing revenue. During 2004 JME signs a contract in the amount of $10 million with the following data available:

How much gross profit should JME recognize for 2004?

Question 52 - #97950

A complex capital structure would typically contain:

Question 53 - #97712

Where in the financial statements should a firm recognize the unrealized gains and losses on cash flow hedging derivatives and the unrealized gains and losses on available-for-sale securities?

Question 54 - #97965

The ZZT Company went public on June 1, 2004, by issuing 25 million shares of common stock. In 2005, the firm raised additional capital by issuing 2 million shares of preferred stock. What is the weighted average number of common shares outstanding for the year ending December 31, 2005?

Question 55 - #98069

Costs incurred to date $2,200,000 Billings to date $2,000,000 Cash collected $1,750,000 Total cost of project $8,800,000

A) -$200,000.B) $300,000.C) -$450,000.

A) variable rate notes.B) bank notes.C) convertible bonds.

Cash flow hedging derivatives Available-for-sale securities

A) Other comprehensive income Net income

B) Other comprehensive income Other comprehensive income

C) Net income Other comprehensive income

A) 25,000,000.B) 14,583,333.C) 10,416,667.

Page 20: Financial Reporting and Analysis_The Income Statement, Balance Sheet, and Cash Flow Statement

Consider the following:

Statement #1 – The presentation formats of balance sheets vary from company to company. Statement #2 – A classified balance sheet cannot follow an account format.

With respect to these statements:

Question 56 - #97398

An analyst has gathered the following information about a company:

Note: the dividend payout ratio equals 20%.

What is the net increase or decrease in cash?

Question 57 - #97285

Earlier this year, Slayton Corporation repurchased 5% of its total shares outstanding. At the time, the book value of Slayton shares exceeded their market value. The shares are expected to be reissued in the future when the market price of Slayton’s stock increases. Do Slayton’s repurchased shares continue to have voting rights and to pay cash dividends?

A) both are correct.B) only one is correct.C) both are incorrect.

Income Statement 2005Sales $650Expenses

COGS $445Depreciation 10Selling, General & Admin. 112

Interest 10Total expenses 577

Pre-tax income $73Taxes 29Net income $44

Balance SheetAssets 2004 2005 Liabilities 2004 2005Cash 50 35 Accts. Payable 115 90Accts. Rec. 120 140 Wages Payable 55 50Inventories 75 70 Bonds 100 90Fixed Assets 215 190 Common Stock 50 20Accum. Depr. (95) (105) Retained Earnings 45 80Total 365 330 365 330

A) -$15.B) +$15.C) +$43.D) -$43.E)F)

Cash dividends

Page 21: Financial Reporting and Analysis_The Income Statement, Balance Sheet, and Cash Flow Statement

Question 58 - #97668

Would an increase in net profit margin or in the firm’s dividend payout ratio increase a firm’s sustainable growth rate?

Question 59 - #97781

Which of the following statements regarding the treasury stock method of computing diluted shares is least accurate? The treasury stock method:

Question 60 - #98022

On January 1, 2007, Sneed Corporation purchased machinery costing $8 million with a salvage value of $1 million. For the year ended 2007, Sneed recognized depreciation expense of $3.2 million from the machinery using the double-declining-balance method. Should the depreciation expense be reported as an operating component in the income statement, and what is the estimated useful life of the machinery?

Question 61 - #97323

A company has the following changes in its balance sheet accounts:

Voting rights paid

A) No No

B) Yes No

C) No Yes

Net profit margin Dividend payout ratio

A) Yes Yes

B) No No

C) Yes No

A) assumes that the hypothetical funds received by the company from the exercise of the options are used to sell shares of the company’s common stock in the market at the average market price.

B) is used when the exercise price of the option is less than the average market price.

C) increases the total number of shares by less than the number that the exercise of the options would create.

Operating expense Useful life

A) No 5 years

B) Yes 5 years

C) Yes 4 years

Page 22: Financial Reporting and Analysis_The Income Statement, Balance Sheet, and Cash Flow Statement

The company’s cash flow from financing is:

Question 62 - #97945

During 2007, Brownfield Incorporated purchased $140 million of inventory. For the year just ended, Brownfield reported cost of goods sold of $130 million. Inventory at year-end was $45 million. Calculate inventory turnover for the year.

Question 63 - #97938

Which of the following statements best describes vertical common-size analysis and horizontal common-size analysis?

Statement #1 – Each line item is expressed as a percentage of its base-year amount.

Statement #2 – Each line item of the income statement is expressed as a percentage of revenue and each line item of the balance sheet is expressed as a percentage of ending total assets.

Statement #3 – Each line item is expressed as a percentage of the prior year’s amount.

Question 64 - #98029

The JME Jumpers, a professional volleyball team, sells season tickets to all home games. The cost of a season ticket is $1,000 and the team plays 20 home games, which run from April through August. For the year ended June 30, 2005, JME sold 1,200 tickets, collected 80 percent of the amount owed, and played 12 home games.

Net Sales $500 An increase in accounts receivable 20 A decrease in accounts payable 40 An increase in inventory 30 Sale of common stock 100 Repayment of debt 10 Depreciation 2 Net Income 100 Interest expense on debt 5

A) $90.B) $100.C) -$10.

A) 3.71.B) 2.89.C) 3.25.

Vertical analysis Horizontal analysis

A) Statement #1 Statement #2

B) Statement #2 Statement #1

C) Statement #2 Statement #3

Page 23: Financial Reporting and Analysis_The Income Statement, Balance Sheet, and Cash Flow Statement

How much revenue should JME recognize?

Question 65 - #97743

Which of the following securities would least likely be found in a simple capital structure?

Question 66 - #97948

Maine Company’s stock transactions during the year are described below:

January 1 100,000 common shares outstanding

March 1 2 for 1 stock split

August 1 10% stock dividend

The weighted average number of shares outstanding used to calculate earnings per share is:

Question 67 - #97328

Which of the following statements about accounting procedures and their impact on the statement of cash flows is least valid? All else equal:

Question 68 - #97864

Selected information from Caledonia, Inc.’s financial activities in the year 20X6 is as follows:

Net income = $460,000. 2,300,000 shares of common stock were outstanding on January 1. The average market price per share was $2 and the year-end stock price was $1.50.

A) $960,000. B) $1,200,000. C) $720,000.

A) 3%, $100 par value convertible preferred.B) 6%, $5000 par value putable bond.C) 7%, $100 par value non convertible preferred.

A) 211,111.B) 220,000.C) 201,666.

A) the cash flow from operations (CFO) for a company that issues a bond at a premium will be understated compared to a firm that issues a bond at par.

B) a company that finances through common stock issues may have the same cash flow from financing (CFF) as a firm that issues debt.

C) a nonprofitable company that uses LIFO to account for inventory will have higher total cash flow than a nonprofitable company that uses FIFO during a period of rising prices.

Page 24: Financial Reporting and Analysis_The Income Statement, Balance Sheet, and Cash Flow Statement

1,000 shares of 8%, $1,000 par value preferred shares were outstanding on January 1. Preferred dividends were paid in 20X6. 10,000 warrants, each of which allows the holder to purchase 100 shares of common stock at an exercise price of $1.50 per common share, were outstanding the entire year.

Caledonia’s diluted earnings per share for 20X6 are closest to:

Question 69 - #97852

Which of the following ratios would NOT be used to evaluate how efficiently management is utilizing the firm’s assets?

Question 70 - #97301

Information related to Bledsoe Corporation’s inventory, as of December 31, 2007, follows:

Using the appropriate valuation method, what adjustment is necessary to accurately report Bledsoe’s inventory at the end of 2007, and will this adjustment affect Bledsoe’s quick ratio?

Question 71 - #97957

At the beginning of this year Aristotle Co. had 400,000 shares of common stock outstanding. During the year, Aristotle paid a 10 percent stock dividend on May 31, issued 90,000 new common shares on June 30, and repurchased 12,000 shares on December 1. The number of shares Aristotle should use in computing earnings per share at the end of the year is:

A) $0.15.B) $0.165.C) $0.180.

A) Gross profit margin.B) Fixed asset turnover.C) Payables turnover.

Estimated selling price $3,500,000 Estimated disposal costs 50,000 Estimated completion costs 300,000 Original FIFO cost 3,200,000 Replacement cost 3,300,000

Adjustment Quick ratio

A) $50,000 write-down Yes

B) $50,000 write-down No

C) $100,000 write-up No

A) 475,000. B) 484,000.C) 476,000.

Page 25: Financial Reporting and Analysis_The Income Statement, Balance Sheet, and Cash Flow Statement

Question 72 - #97802

Orange Company’s net income for 2004 was $7,600,000 with 2,000,000 shares outstanding. The average share price in 2004 was $55. Orange had 10,000 shares of eight percent $1,000 par value convertible preferred stock outstanding since 2003. Each preferred share was convertible into 20 shares of common stock. Orange Company’s diluted earnings per share (Diluted EPS) for 2004 is closest to:

Question 73 - #97393

Use the following financial data for Moose Printing Corporation to calculate the cash flow from operations (CFO) using the indirect method.

Net income: $225 Increase in accounts receivable: $55 Decrease in inventory: $33 Depreciation: $65 Decrease in accounts payable: $25 Increase in wages payable: $15 Decrease in deferred taxes: $10 Purchase of new equipment: $65 Dividends paid: $75

Question 74 - #97983

Zichron, Inc., had the following equity accounts on December 31:

Common stock: 20,000 shares.

Preferred stock A: 10,000 shares convertible into common on a 2 for 1 basis, dividend of $40,000 was declared during the year.

Preferred stock B: 10,000 shares, convertible to common on a 4 for 1 basis, dividend of $5,000 was declared during the year.

The company reported net income of $120,000 and paid a $20,000 dividend to its common shareholders.

Part 1) What are the basic earnings per share reported for the year?

Part 2)

A) $3.45.B) $3.40.C) $3.80.

A) Increase in cash of $173.B) Increase in cash of $248.C) Increase in cash of $183.

A) $2.75.B) $3.75.C) $2.00.

Page 26: Financial Reporting and Analysis_The Income Statement, Balance Sheet, and Cash Flow Statement

What are the diluted earnings per share reported for the year?

Question 75 - #97921

Goldstar Manufacturing has an accounts receivable turnover of 10.5 times, an inventory turnover of 4 times, and payables turnover of 8 times. What is Goldstar’s cash conversion cycle?

Question 76 - #95656

Which of the following choices most accurately illustrates an operating liability and which most accurately illustrates a financing liability?

Question 77 - #98003

All the following items are reported net of taxes below net income from continuing operations on the income statement EXCEPT:

Question 78 - #97920

An analyst gathered the following data about a company:

Current liabilities are $300. Total debt is $900. Working capital is $200. Capital expenditures are $250. Total assets are $2,000. Cash flow from operations is $400.

If the company would like a current ratio of 2, they could:

A) $1.50. B) $3.00. C) $1.33.

A) 80.38 days.B) 6.50 days.C) 171.64 days.

Operating liabilities Financing liabilities

A) Short-term note payable Current portion of long-term debt

B) Accounts payable Current portion of long-term debt

C) Customer advances Accrued liabilities

A) extraordinary items.B) unusual or infrequent items.C) expropriations by foreign governments.

Page 27: Financial Reporting and Analysis_The Income Statement, Balance Sheet, and Cash Flow Statement

Question 79 - #97794

Which of the following statements regarding basic and diluted earnings per share (EPS) is most accurate?

Question 80 - #97314

The following data is from Delta's common size financial statement:

What is Delta's total-debt-to-equity ratio?

Question 81 - #97832

Given the following income statement and balance sheet for a company:

A) increase current assets by 100 or decrease current liabilities by 50.B) decrease current assets by 100 or increase current liabilities by 50.C) increase current assets by 100 or increase current liabilities by 50.

A) To calculate diluted EPS, use net income less preferred dividends in the numerator.B) Neither basic nor diluted EPS considers antidilutive securities in its computation.C) If diluted EPS is less than basic EPS then the convertible securities are said to be antidilutive.

Earnings after taxes 18%

Equity 40%Current assets 60%Current liabilities 30%Sales $300Total assets $1,400

A) 1.5.B) 1.0.C) 2.0.

Balance Sheet Assets Year 2003 Year 2004Cash 500 450Accounts Receivable 600 660Inventory 500 550Total CA 1600 1660Plant, prop. equip 1000 1250Total Assets 2600 2910

LiabilitiesAccounts Payable 500 550Long term debt 700 1002Total liabilities 1200 1552

Page 28: Financial Reporting and Analysis_The Income Statement, Balance Sheet, and Cash Flow Statement

What is the current ratio for 2004?

Question 82 - #97419

An analyst has gathered the following information about a company:

EquityCommon Stock 400 538Retained Earnings 1000 820Total Liabilities & Equity 2600 2910

Income Statement Sales 3000Cost of Goods Sold (1000)Gross Profit 2000SG&A (500)Interest Expense (151)EBT 1349Taxes (30%) (405)Net Income 944

A) 0.331.B) 3.018.C) 2.018.

Income Statement for the Year 2005Sales $1,500ExpensesCOGS $1,300Depreciation 20Goodwill 10Int. Expenses 40

Total expenses 1,370Income from cont. op. 130

Gain on sale 30Income before tax 160Income tax 64Net Income $96

Additional Information:Dividends paid 30Common stock sold 20Equipment purchased 50Bonds issued 80Fixed asset sold for (original cost of $100 with accumulated depreciation of $70) 60

Accounts receivable decreased by 30Inventory decreased by 20Accounts payable increased by 20Wages payable decreased by 10

Page 29: Financial Reporting and Analysis_The Income Statement, Balance Sheet, and Cash Flow Statement

What is the cash flow from investing?

Question 83 - #97724

Are dividends paid to common shareholders and foreign currency translation gains and losses included in a firm’s other comprehensive income?

Question 84 - #98223

An analyst has gathered the following information about a company:

Cost of goods sold = 65% of sales. Inventory of $450,000. Sales of $1 million.

What is the value of this firm’s average inventory processing period using a 365-day year?

Question 85 - #97423

Determine the cash flow from financing given the following table.

A) $130.B) $110.C) $20.D)E) $10.F)

Dividends paid Foreign currency translation gains and losses

A) No Yes

B) Yes Yes

C) No No

A) 252.7 days.B) 0.7 days.C) 1.4 days.

Item AmountCash payment of dividends $30Sale of equipment $10Net income $25Purchase of land $15Increase in accounts payable $20Sale of preferred stock $25Increase in deferred taxes $5Profit on sale of equipment $15

Page 30: Financial Reporting and Analysis_The Income Statement, Balance Sheet, and Cash Flow Statement

Question 86 - #97855

Selected information from Able Company’s financial activities is as follows:

Net Income was $720,000. 1,000,000 shares of common stock were outstanding on January 1. 1,000 shares of 8%, $1,000 par value preferred shares were outstanding on January 1. The tax rate was 40%. The average market price per share for the year was $20. 6,000 shares of 3%, $500 par value preferred shares, convertible into common shares at a rate of 40 common shares for each preferred share, were outstanding for the entire year.

Able’s basic and diluted earnings per share (EPS) are closest to:

Question 87 - #97060

With other variables remaining constant, if profit margin rises, ROE will:

Question 88 - #97788

Selected information from Doors, Inc.’s financial activities in the year 2005 included the following:

Net income was $372,000.

100,000 shares of common stock were outstanding on January 1.

The average market price per share was $18 in 2005.

Dividends were paid in 2005.

2,000, 6 percent $1,000 par value convertible bonds, which are convertible at a ratio of 25 shares for each bond, were outstanding the entire year.

Doors, Inc.’s tax rate is 40%.

Doors, Inc.’s diluted earnings per share (Diluted EPS) for 2005 was closest to:

A) -$5.B) $15.C) $20.

Basic EPS Diluted EPS

A) $0.55 $0.52

B) $0.55 $0.55

C) $0.64 $0.64

A) fall. B) increase.C) remain the same.

Page 31: Financial Reporting and Analysis_The Income Statement, Balance Sheet, and Cash Flow Statement

Question 89 - #97386

In preparing its cash flow statement for the year ended December 31, 2004, Giant Corporation collected the following data:

In its December 31, 2004, statement of cash flows, what amounts should Giant report as net cash used in investing activities and net cash used in financing activities?

Question 90 - #97901

Do the following characteristics have to be met in order to classify a liability as current on the balance sheet?

Characteristic #1 – Settlement is expected within one year or operating cycle, whichever is less. Characteristic #2 – Settlement will require the use of cash within one year or operating cycle, whichever is greater.

Question 91 - #98034

Bug-Be-Gone is a residential pest control company that offers a 12 month home-service contract to eliminate insect infestation. Customers are required to prepay for the service at the beginning of each year. If Bug-Be-Gone erroneously records these payments as revenue and include the estimated cost of performing the service, what is the most likely effect on the firm’s liabilities and equity compared to the correct treatment?

A) $3.72.B) $2.96.C) $3.28.

Gain on sale of equipment $6,000 Proceeds from sale of equipment 10,000 Purchase of Zip Co. bonds for 180,000 (maturity value $200,000) Amortization of bond discount 2,000 Dividends paid (75,000) Proceeds from sale of Treasury stock 38,000

Investing Activities Financing Activities

A) $170,000 $37,000

B) $170,000 -$38,000

C) $178,000 -$37,000

Characteristic #1 Characteristic #2

A) Yes No

B) No No

C) No Yes

Liabilities Equity

Page 32: Financial Reporting and Analysis_The Income Statement, Balance Sheet, and Cash Flow Statement

Question 92 - #97814

Which of the following statements about the earnings per share calculation are most accurate?

Question 93 - #97871

To calculate the cash ratio, the total of cash and marketable securities is divided by:

Question 94 - #97932

Which of the following ratios would least likely measure liquidity?

Question 95 - #97713

Statement #1 – As compared to the price-to-earnings ratio, the price-to-cash flow ratio is easier to manipulate because management can easily control the timing of the cash flows.

Statement #2 – One of the benefits of earnings per share as a valuation metric is that it facilitates the comparison of firms of different sizes.

With respect to these statements:

Question 96 - #98023

A) Overstated Overstated

B) Understated Overstated

C) Overstated Understated

A) When calculating diluted EPS you must add the shares created from the conversion of the bonds to the denominator and the interest expense times the tax rate to the numerator.

B) If the diluted EPS is less than the basic EPS, then the diluted EPS is said to be anti-dilutive.C) None of these choices are correct.

A) current liabilities.B) total liabilities.C) total assets.

A) Return on assets (ROA).B) Quick ratio.C) Current ratio.

A) both are incorrect.B) only one is correct.C) both are correct.

Page 33: Financial Reporting and Analysis_The Income Statement, Balance Sheet, and Cash Flow Statement

Assume that the exercise price of an option is $6, and the average market price of the stock is $10. Assuming 802 options are outstanding during the entire year, what is the number of shares to be added to the denominator of the diluted earnings per share (EPS)?

Question 97 - #97779

When calculating earnings per share (EPS) for firms with complex capital structures, convertible bonds are ordinarily considered to be:

Question 98 - #97833

Given the following income statement and balance sheet for a company:

A) 321.B) 481.C) 802.

A) potentially dilutive securities.B) embedded debt securities.C) antidilutive securities.

Balance Sheet Assets Year 2003 Year 2004Cash 500 450Accounts Receivable 600 660Inventory 500 550Total CA 1300 1660Plant, prop. equip 1000 1250Total Assets 2600 2910

LiabilitiesAccounts Payable 500 550Long term debt 700 1102Total liabilities 1200 1652

EquityCommon Stock 400 538Retained Earnings 1000 720Total Liabilities & Equity 2600 2910

Income Statement Sales 3000Cost of Goods Sold (1000)Gross Profit 2000SG&A 500Interest Expense 151EBT 1349Taxes (30%) 405Net Income 944

Page 34: Financial Reporting and Analysis_The Income Statement, Balance Sheet, and Cash Flow Statement

What is the quick ratio for 2004?

Question 99 - #95268

Which of the following would NOT be a component of cash flow from investing?

Question 100 - #94229

JME acquired an asset on January 1, 2004, for $60,000 cash. At that time JME estimated the asset would last 10 years and have no salvage. During 2006 JME estimated the remaining life of the asset to be only three more years with a salvage value of $3,000. If JME uses straight line depreciation, what is the depreciation expense for 2006?

Question 101 - #97801

Consider the following:

Statement #1 – Copyrights and patents are tangible assets that can be separately identified. Statement #2 – Purchased copyrights and patents are amortized on a straight line basis over 30 years.

With respect to the statements about copyrights and patents acquired from an independent third party:

Question 102 - #97061

Income Statements for Royal, Inc. for the years ended December 31, 20X0 and December 31, 20X1 were as follows (in $ millions):

A) 2.018.B) 3.018.C) 0.331.

A) Purchase of equipment. B) Dividends paid. C) Sale of land.

A) $6,000.B) $15,000. C) $12,000.

A) only statement #2 is incorrect.B) both are incorrect.C) only statement #1 is incorrect.

20X0 20X1Sales 78 82 Cost of Goods Sold (47) (48) Gross Profit 31 34 Sales and Administration (13) (14) Operating Profit (EBIT) 18 20 Interest Expense (6) (10)

Page 35: Financial Reporting and Analysis_The Income Statement, Balance Sheet, and Cash Flow Statement

Analysis of these statements for trends in operating profitability reveals that, with respect to Royal’s gross profit margin and net profit margin:

Question 103 - #98227

Using a 365-day year, if a firm has net annual sales of $250,000 and average receivables of $150,000, what is its average collection period?

Question 104 - #98039

When considering the impact of warrants on earnings per share, the method to calculate the number of shares added to the denominator is derived using which method?

Question 105 - #95812

According to U.S. Generally Accepted Accounting Principles (GAAP) and International Accounting Standards (IAS) GAAP, should dividends paid be treated as a cash flow from financing (CFF) or as a cash flow from operations (CFO)?

Question 106 - #98000

Savannah Corp.’s financial accounts for the year ended December 31 included the following information:

Net Income: $122,000 Preferred Stock Dividends Paid: $35,000 Common Stock Dividends Paid: $42,000

Earnings Before Taxes 12 10 Income Taxes (5) (4) Earnings after Taxes 7 6

A) gross profit margin decreased but net profit margin increased in 20X1.B) gross profit margin increased in 20X1 but net profit margin decreased.C) both gross profit margin and net profit margin increased in 20X1.

A) 1.7 days.B) 46.5 days.C) 219.0 days.

A) Cost recovery method.B) Weighted average method.C) Treasury Stock method.

U.S. GAAP IAS GAAP

A) CFF or CFO CFO

B) CFO CFF

C) CFF CFF or CFO

Page 36: Financial Reporting and Analysis_The Income Statement, Balance Sheet, and Cash Flow Statement

Common Shares outstanding at January 1: 50,000 10% preferred $100 par value shares outstanding at January 1: 3,500

No stock transactions occurred during the year and all preferred stock dividends were paid. Basic earnings per share for Savannah are closest to:

Question 107 - #98088

A company has the following sequence of events regarding their stock:

One million shares outstanding at the beginning of the year. On June 30th, they declared and issued a 10% stock dividend. On September 30th, they sold 400,000 shares of common stock at par.

Basic earnings per share at year-end will be computed on how many shares?

Question 108 - #97780

When calculating earnings per share (EPS) for firms with complex capital structures, convertible preferred stock is ordinarily considered to be a:

Question 109 - #96769

Given the following information about a firm what is its return on equity (ROE)?

An asset turnover of 1.2. An after tax profit margin of 10%. A financial leverage multiplier of 1.5.

Question 110 - #93882

For the year ended December 31, 2007, Gremlin Corporation reported the following transactions:

A) $0.90.B) $2.44.C) $1.74.

A) 1,000,000.B) 1,100,000.C) 1,200,000.

A) non-equity security.B) antidilutive security.C) potentially dilutive security.

A) 0.09.B) 0.12.C) 0.18.

Page 37: Financial Reporting and Analysis_The Income Statement, Balance Sheet, and Cash Flow Statement

Issued 5,000 shares of preferred stock for land with a fair value of $4.8 million. Purchased a patent for $3.3 million cash. Acquired 40% of the common stock of an affiliate for $2.7 million cash which was borrowed from a bank. Exchanged equipment with a book value of $1.7 million for equipment valued at $2.1 million. The exchange was an even trade. Converted bonds payable with a book value of $5 million to 50,000 shares of common stock with a fair value of $6 million.

Calculate Gremlin’s cash flow from investing activities and cash flow from financing activities for the year ended December 31, 2007.

Question 111 - #98081

The following data pertains to the Megatron company:

Net income equals $15,000. 5,000 shares of common stock issued on January 1. 10% stock dividend issued on June 1. 1000 shares of common stock were repurchased on July 1. 1000 shares of 10%, par $100 preferred stock each convertible into 8 shares of common were outstanding the whole year.

How many common shares should be used in computing the company’s basic earnings per share (EPS)?

Question 112 - #97403

An analyst compiled the following information for Universe, Inc. for the year ended December 31, 20X4:

Net income was $850,000. Depreciation expense was $200,000. Interest paid was $100,000. Income taxes paid were $50,000. Common stock was sold for $100,000. Preferred stock (eight percent annual dividend) was sold at par value of $125,000. Common stock dividends of $25,000 were paid. Preferred stock dividends of $10,000 were paid. Equipment with a book value of $50,000 was sold for $100,000.

Using the indirect method and assuming U.S. GAAP, what was Universe Inc.’s cash flow from operations (CFO) for the year ended December 31, 20X4?

Cash flow from investing activities

Cash flow from financing activities

A) $1.7 million inflow $1.3 million outflow

B) $6.0 million outflow $2.7 million inflow

C) $2.7 million outflow $6.0 million inflow

A) 5,000.B) 4,500.C) 5,500.

A) $1,000,000.B) $1,050,000.

Page 38: Financial Reporting and Analysis_The Income Statement, Balance Sheet, and Cash Flow Statement

Question 113 - #98041

Stanley Corp. had 100,000 shares of common stock outstanding throughout 2004. It also had 20,000 stock options with an exercise price of $20 and another 20,000 options with an exercise price of $28. The average market price for the company's stock was $25 throughout the year. The stock closed at $30 on December 31, 2004. What are the number of shares used to calculate diluted earnings per share for the year?

Question 114 - #97933

Selected information from Indigo Corp.’s financial activities in the year 20X9 included the following:

Net income is $5,600,000. The tax rate is 40%. 500,000 shares of common stock were outstanding on January 1. The average market price per share was $82 in 20X9. 6,000 5% coupon $1,000 par value convertible bonds, which are convertible at a ratio of 20 shares for each bond, were outstanding the entire year. 200,000 shares of common stock were issued on July 1. 100,000 shares of common stock were purchased by the company as treasury stock on October 1.

Indigo Corp.’s diluted earnings per share for 20X9 are closest to:

Question 115 - #98130

Which of the following items would least likely be included in cash flow from financing?

Question 116 - #98048

To be classified as an extraordinary item on the income statement under U.S. GAAP, the item must be:

C) $1,015,000.

A) 105,000. B) 104,000. C) 110,000.

A) $8.32.B) $9.74.C) $8.49.

A) Gain on sale of stock of a subsidiary.B) Dividends paid to shareholders.C) Purchase of treasury stock.

A) probable and infrequent in nature.B) unusual in nature and infrequent in occurrence.C) estimated and probable.

Page 39: Financial Reporting and Analysis_The Income Statement, Balance Sheet, and Cash Flow Statement

Question 117 - #97906

Which of the following is least likely a routinely used operating profitability ratio?

Question 118 - #98040

When considering convertible preferred stock which of the following components of the earnings per share (EPS) equation needs to be adjusted to calculate diluted earnings per share?

© 2010 Kaplan Schweser

A) Net income/net sales.B) Gross profit/net sales.C) Sales/Total Assets

A) The numerator and denominator.B) The numerator.C) The denominator.

Page 40: Financial Reporting and Analysis_The Income Statement, Balance Sheet, and Cash Flow Statement

Kaplan Schweser Printable Exams - Test Management Exam 19

Test ID#: 7788317

Question 1 - #122497

Your answer: A was incorrect. The correct answer was B) always equal to zero.

The direct and indirect methods are two ways of presenting the same total for cash from operations.

This question tested from Session 8, Reading 34, LOS d

Question 2 - #98070

Your answer: A was incorrect. The correct answer was C) Percentage-of-completion method.

The installment sales method recognizes revenue and associated cost of goods sold only when cash is received. Gross profit (sales – cost of goods sold) reflects the proportion of cash received.

The cost recovery method is similar to the installment sales method but is more conservative. Sales are recognized when cash is received, but no gross profit is recognized until all of the cost of goods sold is collected.

This question tested from Session 8, Reading 32, LOS b

Question 3 - #95386

Your answer: A was incorrect. The correct answer was B) Changes in accounts receivable, inventory, accounts payable, and items that flow through the income statement.

The main components of cash flow from operations are changes in working capital items (accounts receivable, inventory, accounts payable), and items that flow through the income statement. Capitalization activities, sale of assets and purchases of securities would all be part of cash flows from investing. Repayment of bonds and issuance of common stock would also be part of cash flows from financing. The stock split would be a non-cash activity.

This question tested from Session 8, Reading 34, LOS a

Question 4 - #96857

Your answer: A was incorrect. The correct answer was B) Current ratio.

Total asset turnover measures operating efficiency and interest coverage measures a company’s financial risk.

This question tested from Session 8, Reading 35, LOS d

Question 5 - #97795

Your answer: A was incorrect. The correct answer was C) 2.0.

Receivables turnover = 1,500(sales) / 750(receivables) = 2.0

Page 41: Financial Reporting and Analysis_The Income Statement, Balance Sheet, and Cash Flow Statement

This question tested from Session 8, Reading 35, LOS d

Question 6 - #97710

Your answer: A was incorrect. The correct answer was B)

Comprehensive income includes all transactions that affect shareholders’ equity except transactions with shareholders. Thus, any transaction that affects net income would also affect comprehensive income. Since the inventory write-down is included in net income, it is part of comprehensive income. The acquisition of treasury stock is a transaction with shareholders; thus, it is not a part of comprehensive income.

This question tested from Session 8, Reading 32, LOS i

Question 7 - #97400

Your answer: A was correct!

Extraordinary items are reported below income from continuing operations but above net income. You must adjust for changes in the working capital accounts: accounts receivable, inventory, and accounts payable.

This question tested from Session 8, Reading 34, LOS e

Question 8 - #96543

Your answer: A was incorrect. The correct answer was C) Debt to total capital.

The debt to total capital ratio is not part of the original DuPont system. The firm’s leverage is accounted for through the equity multiplier.

This question tested from Session 8, Reading 35, LOS f

Question 9 - #96767

Your answer: A was incorrect. The correct answer was C) 0.62.

There are several ways to approach this question but the easiest way is to recognize that ROE = NI / average equity thus ROE = 944 / 1,519 = 0.622.

If using the traditional DuPont, ROE = (NI / Sales) × (Sales / Assets) × (Assets / Equity):

ROE = (944 / 3,000) × (3,000 / 2,920) × (2,920 / 1,519) = 0.622

The 5-part Dupont formula gives the same result:

ROE = (net income / EBT)(EBT / EBIT)(EBIT / revenue)(revenue / total assets)(total assets / total equity)

Where EBIT = EBT + interest = 1,349 + 151 = 1,500

ROE 2007 = (944 / 1,349)(1,349 / 1,500)(1,500 / 3,000)(3,000 / 2,920)(2,920 / 1,519) = 0.622

This question tested from Session 8, Reading 35, LOS e

Yes No

Page 42: Financial Reporting and Analysis_The Income Statement, Balance Sheet, and Cash Flow Statement

Question 10 - #98138

Your answer: A was incorrect. The correct answer was C)

An asset provides future economic benefits as a result of past transactions. Assets can be tangible or intangible. In some cases, assets are acquired without cost, but will be reported to the extent that they will provide future economic benefit, and thus have value.

This question tested from Session 8, Reading 33, LOS a

Question 11 - #97362

Your answer: A was incorrect. The correct answer was B) cash flow from financing would increase and cash flow from investing would decrease.

Cash flow from financing increases when stock is issued, while cash flow from investing decreases when spending for purchases of fixed assets.

This question tested from Session 8, Reading 34, LOS a

Question 12 - #93917

Your answer: A was incorrect. The correct answer was C) The current portion of long-term debt.

The current portion of long-term debt arises from a financing activity. The other items listed arise from operating activities.

This question tested from Session 8, Reading 34, LOS a

Question 13 - #97449

Your answer: A was incorrect. The correct answer was C) $900,000.

CFO = sales $3,000,000 – purchases $1,800,000 – change in accounts receivable ($200,000 – $300,000) – other cash operating expenses $400,000 = $900,000.

This question tested from Session 8, Reading 34, LOS e

Question 14 - #97391

Your answer: A was incorrect. The correct answer was F) $2,000,000.

Using the easiest method of all, the difference in the cash account at the end of 2004 and the cash balance projected for the end of 2005 is $26.0 million - $24.0 million = $2.0 million. If the cash balances were not available, the change in cash could be calculated using the indirect method. Starting with cash flow from operations (CFO) in $ millions projected for 2005:

Yes No

Net Income 43 Add: Noncash Expenses or Losses Depreciation 5 Add: Changes in Current Assets and Liabilities Less: Increase in Accounts Receivable -7

Page 43: Financial Reporting and Analysis_The Income Statement, Balance Sheet, and Cash Flow Statement

Net Cash Flow = CFO + CFI + CFF = 1 – 25 + 26 = $2 million.

This question tested from Session 8, Reading 34, LOS e

Question 15 - #97345

Your answer: A was incorrect. The correct answer was C)

Because of the matching principle, compensation expense should be increased by the (accrued) salary expense for the last week of 2007 and the liability for the bonuses was incurred in 2007. Thus, total compensation expense for 2007 is $98.5 million ($90 million unadjusted compensation expense + $3.5 million salary accrual + $5 million bonus accrual). Since the salaries and bonuses were not paid in 2007, accrued liabilities would increase by $8.5 million. An increase in accrued liabilities, a current liability, would decrease the current ratio.

This question tested from Session 8, Reading 33, LOS h

Question 16 - #97939

Your answer: A was incorrect. The correct answer was C)

Gross profit margin = ($1,000 net sales − $600 COGS) / $1,000 net sales = 400 / 1,000 = 0.4

Operating profit margin = ($1,000 net sales − $600 COGS − $200 operating expenses) / $1,000 net sales = $200 / $1000 = 0.2

This question tested from Session 8, Reading 35, LOS d

Question 17 - #98050

Your answer: A was incorrect. The correct answer was C) in proportion to the cash collection.

The installment sales method recognizes sales and COGS in proportion to cash collections.

This question tested from Session 8, Reading 32, LOS b

Question 18 - #94690

Your answer: A was incorrect. The correct answer was C) $4,800.

Accumulated depreciation at the end of year 3 = [($10,000 − $2,000) / 5] × 3 = $4,800

Less: Increase in Inventory -50 Plus: Increase in Accounts Payable 10Net Cash Flow from Operations (CFO) 1 Increase in Property Plant & Equipment -25Net Cash Flow from Investing (CFI) -25 Increase in Long-Term Debt 20 Increase in Common Stock 10 Less: Dividends Paid (10 million × $0.40) - 4Net Cash Flow from Financing (CFF) 26

$98.5 million Decrease

40% 20%

Page 44: Financial Reporting and Analysis_The Income Statement, Balance Sheet, and Cash Flow Statement

This question tested from Session 8, Reading 32, LOS d

Question 19 - #98010

Your answer: A was incorrect. The correct answer was B) Items that are unusual in nature or infrequent in occurrence appear below income from continuing operations on a pretax basis.

The key word here is "or." Unusual or infrequent items are unusual or infrequent, but NOT both. These items are reported (as a separate line item) as a component of net income from continuing operations.

Examples of unusual or infrequent items include:

Gains or losses from the disposal of a business segment (employee separation costs, plant shutdown costs, etc.) Gains or losses from the sale of assets or investments in subsidiaries Provisions for environmental remediation Impairments, write-offs, write-downs, and restructuring costs Integration expenses associated iwth businesses that have been recently acquired.

This question tested from Session 8, Reading 32, LOS f

Question 20 - #97688

Your answer: A was correct!

Sensitivity analysis develops a range of possible outcomes as specific inputs are changed one at a time. Sensitivity analysis is also known as “what-if” analysis. Scenario analysis is based on a specific set of outcomes for multiple variables. Computer generated analysis, based on developing probability distributions of key variables, is known as simulation analysis.

This question tested from Session 8, Reading 35, LOS h

Question 21 - #97360

Your answer: A was incorrect. The correct answer was C)

Beginning with net sales, calculating cash collected from customers requires the addition (subtraction) of any increase (decrease) in unearned revenue. Cash advances from customers represent unearned revenue and are not included in net sales, so any advances must be added to net sales in order to calculate cash collected.

An inventory writedown, as a result of applying the lower of cost or market rule, will reduce ending inventory and increase COGS for the period. However, no cash flow is associated with the writedown, so COGS is reduced by the amount of the writedown in calculating cash paid to suppliers.

This question tested from Session 8, Reading 34, LOS f

Question 22 - #94016

Your answer: A was incorrect. The correct answer was B) an operating cash outflow.

The coupon payment is recorded on the statement of cash flows as an operating cash outflow because cash flow from operations includes a deduction for interest expense.

Subtract decrease in unearned revenue Subtract an inventory writedown

Page 45: Financial Reporting and Analysis_The Income Statement, Balance Sheet, and Cash Flow Statement

This question tested from Session 8, Reading 34, LOS a

Question 23 - #98016

Your answer: A was correct!

Interest received from customers and interest received from investments are a part of normal operations of a financial institution. Thus, the First National Bank will report the interest income from both sources as components of operating income.

This question tested from Session 8, Reading 32, LOS e

Question 24 - #98038

Your answer: A was incorrect. The correct answer was C) $1.00.

Number of average common shares:

1/1 5,500 shares issued (includes 10% stock dividend on 6/1) × 12 = 66,000

7/1 1,000 shares repurchased × 6 months = -6,000

= 60,000

60,000 shares / 12 months = 5,000 average shares

Preferred dividends = ($10)(1,000) = $10,000

Number of shares from the conversion of the preferred shares = (1,000 preferred shares)(8 shares of common/share of preferred) = 8,000 common

Diluted EPS = [$15,000(NI) − $10,000(pfd) + $10,000(pfd)] / 5000(common shares) + 8000(shares from the conv. pfd. shares) = $15,000 / 13,000 shares = $1.15/share

This number needs to be compared to basic EPS to see if the preferred shares are antidilutive.

Basic EPS = [$15,000(NI) − $10,000(preferred dividends)] / 5,000 shares = $5,000 / 5,000 shares = $1/share

Since the EPS after the conversion of the preferred shares is greater than before the conversion the preferred shares are antidilutive and they should not be treated as common in computing diluted EPS. Therefore diluted EPS is the same as basic EPS or $1/share.

This question tested from Session 8, Reading 32, LOS h

Question 25 - #97771

Your answer: A was correct!

Not all preferred stock is dilutive. Only convertible preferred stock is potentially dilutive.

This question tested from Session 8, Reading 32, LOS h

Question 26 - #97376

Page 46: Financial Reporting and Analysis_The Income Statement, Balance Sheet, and Cash Flow Statement

Your answer: A was incorrect. The correct answer was B)

Cash flow relating to financing activities includes dividends paid, cash received from preferred stock, and repayment of loan. -2,000 + 10,000 + -5,000 = 3,000.

Cash flow relating to investing activities includes cash paid for equipment and cash from sale of land. -22,000 + 12,000 = -10,000.

This question tested from Session 8, Reading 34, LOS a

Question 27 - #97076

Your answer: A was incorrect. The correct answer was B)

Interest coverage ratio = (EBIT / interest expense) = (115 / 15) = 7.67

Net profit margin = (net income / net sales) = (60 / 200) = 0.30

This question tested from Session 8, Reading 35, LOS d

Question 28 - #97252

Your answer: A was incorrect. The correct answer was C)

Trading securities are reported in the balance sheet at fair value. At the end of the year, the fair value of the securities was $980,000 ($435,000 + $545,000). The unrealized gains and losses from trading securities are recognized in the income statement. Thus, Ponca would recognize an unrealized gain of $30,000 ($980,000 fair value – $950,000 cost).

This question tested from Session 8, Reading 33, LOS f

Question 29 - #97915

Your answer: A was incorrect. The correct answer was B)

The total debt ratio = ($18.4m – 7.0m) / ($18.4m) or 0.62. The interest coverage ratio = $2,000,000 / $900,000 = 2.22.

This question tested from Session 8, Reading 35, LOS d

Question 30 - #97968

Your answer: A was correct!

If bonds are converted, then net income will increase by 480,000 [10 million × 0.08 × (1 − 0.4)] and shares outstanding will increase by 200,000.

numerator = 3,000,000 + 480,000 = 3,480,000 denominator = 350,000 + (150,000 × 4/12) + 200,000 = 600,000

$3,000 -$10,000

7.67 0.30

$30,000 unrealized gain $980,000

0.62 2.22

Page 47: Financial Reporting and Analysis_The Income Statement, Balance Sheet, and Cash Flow Statement

diluted EPS = 3,480,000 / 600,000 = 5.80

This question tested from Session 8, Reading 32, LOS h

Question 31 - #97986

Your answer: A was incorrect. The correct answer was B)

Basic EPS: [400,000 − 10,000] / 200,000 shares = $1.95 per share

Diluted EPS: [400,000 + (30,000 × 0.6)] / [200,000 + 10,000 + 15,000] = $1.86 per share

This question tested from Session 8, Reading 32, LOS h

Question 32 - #97923

Part 1) Your answer: A was incorrect. The correct answer was C)

Current ratio = current assets / current liabilities = 12,297 / 7,735 = 1.59

Quick ratio = (cash + receivables) / current liabilities = 2,098 + 4,570 / 7,735 = 0.86

Cash ratio = cash / current liabilities = 2,098 / 7,735 = 0.271

This question tested from Session 8, Reading 35, LOS d

Part 2) Your answer: A was correct!

ROE = net income / equity = 4,127 / 8,354 = 0.49

This question tested from Session 8, Reading 35, LOS d

Question 33 - #97735

Your answer: A was incorrect. The correct answer was B) 2.67.

Quick ratio = [100(cash) + 750(AR) + 300(marketable securities)] / [300(AP) + 130(short-term debt)] = (1,150 / 430) = 2.67

This question tested from Session 8, Reading 35, LOS d

Question 34 - #122499

Your answer: A was incorrect. The correct answer was B) of different size in the same industry.

Ratio analysis is a useful way of comparing companies that are similar in operations but different in size. Ratios of companies that operate in different industries are often not directly comparable. For companies that operate in several industries, ratio analysis is limited by the difficulty of determining appropriate industry benchmarks.

$1.95 $1.86

1.59 0.86 0.27

Page 48: Financial Reporting and Analysis_The Income Statement, Balance Sheet, and Cash Flow Statement

This question tested from Session 8, Reading 35, LOS b

Question 35 - #98083

Your answer: A was incorrect. The correct answer was C) Neither statement is correct.

Neither statement is correct because the definitions are reversed.

This question tested from Session 8, Reading 32, LOS b

Question 36 - #97972

Your answer: A was incorrect. The correct answer was B) $0.457.

50,000,000 common shares × 12 months = 600,000,000

5,000,000 common shares × 6 months = 30,000,000 = 630,000,000

630,000,000 / 12 = 52,500,000 average shares

[$25,000,000(NI) − $1,000,000(preferred dividends)] / 52,500,000 shares = $24,000,000 / 52,5000,000 = $0.457

This question tested from Session 8, Reading 32, LOS g

Question 37 - #97974

Your answer: A was incorrect. The correct answer was C) 150,000.

Connecticut’s January 1 balance of common shares outstanding is adjusted retroactively for the 1 for 3 reverse stock split, meaning there are (360,000 / 3) = 120,000 “new” shares treated as if they had been outstanding since January 1. The weighted average of the shares issued in July, (60,000 × 6 / 12) = 30,000 is added to that figure, for a total of 150,000.

This question tested from Session 8, Reading 32, LOS g

Question 38 - #97835

Your answer: A was incorrect. The correct answer was C)

Calculating Basic Shares:

Jan 1 100,000 shares outstanding May 1 30,000 shares issued July 1 10% stock dividend issued

The 10% stock dividend is retroactive therefore:

110,000 shares × 12 months = 1,320,000 33,000 shares × 8 months = 264,000 Total share-month = 1,584,000 Average shares = (1,584,000 / 12) = 132,000

Calculating diluted shares:

132,000 139,000

Page 49: Financial Reporting and Analysis_The Income Statement, Balance Sheet, and Cash Flow Statement

(1,000 bonds) × (21 shares each) × (4 months) = 84,000 total share-month 84,000 / 12 = 7,000 Average shares

Total diluted shares = 7,000 (from convertible bonds) + 132,000 (from stock) = 139,000

This question tested from Session 8, Reading 32, LOS h

Question 39 - #97907

Your answer: A was correct!

(Cash + short-term marketable investments + receivables) divided by average daily cash expenditures is known as the defensive interval ratio. The defensive interval ratio is a liquidity ratio that measures the firm’s ability to pay cash expenditures in the absence of external cash flows, but does not directly measure profitability. EBIT / average total assets is one variation of the return on assets ratio. Return on assets is a profitability ratio that measures the efficiency of managing assets and generating profits.

This question tested from Session 8, Reading 35, LOS d

Question 40 - #97330

Your answer: A was correct!

Dividends declared in 2004 are net income less the increase in retained earnings ($800,000 - $300,000 = $500,000). Dividends declared less the increase in dividends payable is dividends paid ($500,000 – ($300,000 - $200,000) = $400,000). This is a cash outflow so it is a negative number. Dividends paid are always cash flow from financing under U.S. GAAP. Note that accounts payable changes are included in cash flow from operations (CFO).

This question tested from Session 8, Reading 34, LOS e

Question 41 - #97299

Your answer: A was incorrect. The correct answer was D) Increase in cash of $8,125.

Silverstone Company’s cash flow from operations would be calculated as +Net Income $8,000 + Inventory $250 - Prepaid exp. $500 + Depreciation $175 + A/P $200 = $8,125.

Bonds payable is a financing activity and would not be included in the cash flow from operations. The indirect method takes the change in the non-cash accounts and decreases or increases net income to get to the change in cash flow.

This question tested from Session 8, Reading 34, LOS e

Question 42 - #97999

Your answer: A was incorrect. The correct answer was B) 306.

(816)(5) = $4,080. $4,080 / $8 = 510 shares. 816 − 510 = 306 new shares or [(8 − 5) / 8]816 = 306.

This question tested from Session 8, Reading 32, LOS h

Question 43 - #97119

Page 50: Financial Reporting and Analysis_The Income Statement, Balance Sheet, and Cash Flow Statement

Your answer: A was incorrect. The correct answer was C)

The restructuring charge and asset write-down are non-recurring transactions; thus, net income will be higher in 20X8, all else equal. In 20X8, fixed asset turnover will be the same as 20X7, all else equal. The asset impairment charge is a one-time charge, so fixed assets will not be reduced further in 20X8.

This question tested from Session 8, Reading 35, LOS d

Question 44 - #98017

Your answer: A was incorrect. The correct answer was B) 248.

(992)($9) = $8928 $8928 / 12 = 744 992 − 744 = 248 new shares or [(12 − 9) / 12]992 = 248

This question tested from Session 8, Reading 32, LOS h

Question 45 - #97358

Your answer: A was correct!

Free cash flow to equity (FCFE) is generally defined as cash flow from operations (CFO) less net fixed capital expenditures plus net borrowing. No information on borrowing is given here, so FCFE = 20 − (5 − 3) = $18 million.

This question tested from Session 8, Reading 34, LOS h

Question 46 - #95629

Your answer: A was correct!

Changes in retained earnings are not included in the calculation of financing cash flows.

This question tested from Session 8, Reading 34, LOS a

Question 47 - #97476

Your answer: A was incorrect. The correct answer was B) 52 days.

Average Inventory Processing Period = 365 / inventory turnover = 365 / 7 = 52 days.

This question tested from Session 8, Reading 35, LOS d

Question 48 - #97385

Your answer: A was incorrect. The correct answer was C) $1,200,000.

The 25% GP indicates that the cost of goods sold is 75% of sales. The inventory is derived from the difference between current ratio and the quick ratio. The current ratio indicates that the current assets are $200,000 and the quick assets are $125,000. The difference represents the inventory of $75,000. The inventory turnover is used to obtain cost of goods sold of $900,000. The cost of goods sold is 75% of sales, indicating that sales are $1,200,000.

Higher Unchanged

Page 51: Financial Reporting and Analysis_The Income Statement, Balance Sheet, and Cash Flow Statement

This question tested from Session 8, Reading 35, LOS d

Question 49 - #93821

Your answer: A was incorrect. The correct answer was B) dividends paid to shareholders.

Dividends paid are a financing cash flow. Dividends received and interest paid are both operating cash flows.

This question tested from Session 8, Reading 34, LOS a

Question 50 - #98025

Your answer: A was incorrect. The correct answer was C) 2006.

Under the accrual concept, income is recognized when the earning activities are substantially completed, risk of ownership has transferred from buyer to seller, and payment is realizable and collectible. Under the matching principle, expenses incurred that directly relate to the sold item are expensed in the same period as the revenue is recognized.

This question tested from Session 8, Reading 32, LOS b

Question 51 - #98032

Your answer: A was incorrect. The correct answer was B) $300,000.

stage of completion = 25%(2.2 / 8.8)

revenue to be recognized = 0.25 × 10 million = 2.5 million

gross profit = 2.5 million − 2.2 million = 300,000

This question tested from Session 8, Reading 32, LOS b

Question 52 - #97950

Your answer: A was incorrect. The correct answer was C) convertible bonds.

A complex capital structure is one that contains securities that have the potential to dilute a firm’s earnings per share. For example, convertible bonds, convertible preferred stock, options, and warrants have the potential to dilute earnings per share upon conversion or exercise.

This question tested from Session 8, Reading 32, LOS g

Question 53 - #97712

Your answer: A was incorrect. The correct answer was B)

Unrealized gains and losses from cash flow hedging derivatives and unrealized gains and losses from available-for-sale securities are not recognized in the income statement; rather, they are both recognized as a component of stockholders’ equity as a part of other comprehensive income.

This question tested from Session 8, Reading 32, LOS j

Other comprehensive income Other comprehensive income

Page 52: Financial Reporting and Analysis_The Income Statement, Balance Sheet, and Cash Flow Statement

Question 54 - #97965

Your answer: A was correct!

The weighted average number of common shares outstanding is the number of shares outstanding during the year weighted by the portion of the year they were outstanding. Since no new common shares were issued in 2005, and there were 25 million shares at the end of 2004, there are 25 million shares at the end of 2005. Note that the preferred stock shares do not affect the common shares outstanding.

This question tested from Session 8, Reading 32, LOS g

Question 55 - #98069

Your answer: A was incorrect. The correct answer was B) only one is correct.

The presentation formats of balance sheets vary by company. A classified balance sheet can be presented in a report format or an account format.

This question tested from Session 8, Reading 33, LOS b

Question 56 - #97398

Your answer: A was correct!

There are two ways to approach this problem. The easier way is to just take the difference in cash from the two years: $35 − $50 = -$15.

The harder way is to create a statement of cash flows:

CFO = Net Income (44) + (Depreciation) (10) – (increase in Accounts Receivable) (20) + (decrease in Inventory) (5) – (decrease in Accounts Payable) (25) – (decrease in Wages Payable) (5) = $9.

CFI = $25 (fixed assets decreased by $25 representing a source of cash)

CFF = Dividends paid ((0.20) × (44)) = -9 – (decrease in bonds) (10) − (decrease in common stock) (30) = -$49.

The net change in cash = 9 + 25 – 49 = -$15, or a decrease of $15.

This question tested from Session 8, Reading 34, LOS e

Question 57 - #97285

Your answer: A was correct!

Repurchased stock that is not cancelled is called treasury stock. Treasury stock does not have voting rights and does not receive cash dividends.

This question tested from Session 8, Reading 33, LOS g

Question 58 - #97668

Your answer: A was incorrect. The correct answer was C) Yes No

Page 53: Financial Reporting and Analysis_The Income Statement, Balance Sheet, and Cash Flow Statement

The sustainable growth rate is equal to ROE multiplied by the retention rate. According to the Dupont formula, an increase in net profit margin will result in higher ROE. Thus, an increase in net profit margin will result in a higher growth rate. The retention rate is equal to 1 minus the dividend payout ratio. Thus, an increase in the dividend payout ratio will lower the retention rate and lower the growth rate.

This question tested from Session 8, Reading 35, LOS g

Question 59 - #97781

Your answer: A was correct!

The treasury stock method assumes any funds received by the company from the exercise of the options are used to purchase shares (not sell shares) of the company’s common stock in the market at the average market price.

This question tested from Session 8, Reading 32, LOS h

Question 60 - #98022

Your answer: A was incorrect. The correct answer was B)

Depreciation expense is reported as an operating component in the income statement. Given the first year depreciation expense of $3.2 million, and the original cost of $8 million, the declining balance percentage is 40% ($3.2 million depreciation expense / $8 million cost). The double declining balance percentage is equal to 2 / useful life = 40%. Thus, the useful life is 5 years (2 / 0.40).

This question tested from Session 8, Reading 32, LOS e

Question 61 - #97323

Your answer: A was correct!

This question tested from Session 8, Reading 34, LOS e

Question 62 - #97945

Your answer: A was incorrect. The correct answer was C) 3.25.

First, calculate beginning inventory given COGS, purchases, and ending inventory. Beginning inventory was $35 million [$130 million COGS + $45 million ending inventory – $140 million purchases]. Next, calculate average inventory of $40 million [($35 million beginning inventory + $45 million ending inventory) / 2]. Finally, calculate inventory turnover of 3.25 [$130 million COGS / $40 million average inventory].

This question tested from Session 8, Reading 35, LOS d

Question 63 - #97938

Your answer: A was incorrect. The correct answer was B)

Yes 5 years

Sale of common stock $100 Repayment of debt (10)Financing cash flows $ 90

Page 54: Financial Reporting and Analysis_The Income Statement, Balance Sheet, and Cash Flow Statement

Horizontal common-size analysis involves expressing each line item as a percentage of the base-year figure. Vertical common-size analysis involves expressing each line item of the income statement as a percentage of revenue and each line item of the balance sheet as a percentage of ending total assets.

This question tested from Session 8, Reading 35, LOS a

Question 64 - #98029

Your answer: A was incorrect. The correct answer was C) $720,000.

(1,200 × $1,000 × 12/20) = $720,000

This question tested from Session 8, Reading 32, LOS b

Question 65 - #97743

Your answer: A was correct!

A simple capital structure contains no potentially dilutive securities such as stock options, warrants, or convertible preferred stock.

This question tested from Session 8, Reading 32, LOS g

Question 66 - #97948

Your answer: A was incorrect. The correct answer was B) 220,000.

The January 1 balance of common shares outstanding is adjusted retroactively for both stock dividends and stock splits. The weighted average shares outstanding for the year = 100,000 × 2 × 1.1 = 220,000.

This question tested from Session 8, Reading 32, LOS g

Question 67 - #97328

Your answer: A was incorrect. The correct answer was C) a nonprofitable company that uses LIFO to account for inventory will have higher total cash flow than a nonprofitable company that uses FIFO during a period of rising prices.

Because of the impact of income taxes, a profitable company that accounts for inventory using LIFO will have higher total cash flow than a profitable company that uses FIFO. The company that uses LIFO will have higher cost of goods sold, resulting in lower net income and thus lower taxes.

The other statements are correct. A company that issues common stock is not required to pay dividends (which would reduce cash flow from financing). Thus, it may have the same CFF as a firm that issues debt since interest paid on debt is a component of CFO.

The cash flow from operations for a company that issues a bond at a premium will be understated compared to a firm that issues a bond at par. When a company issues bonds at a premium, the coupon payment is “too big” (reduces CFO) and thus interest expense is reduced by the amount of the amortization of the premium (increases CFF).

This question tested from Session 8, Reading 34, LOS e

Statement #2 Statement #1

Page 55: Financial Reporting and Analysis_The Income Statement, Balance Sheet, and Cash Flow Statement

Question 68 - #97864

Your answer: A was correct!

Caledonia’s basic EPS = (net income − preferred stock dividends) / (weighted average common shares outstanding) = [$460,000 − ($1,000 × 1,000 × 0.08)] / 2,300,000 = $0.17.

Using the treasury stock method, if the warrants were exercised, cash inflow would be 10,000 × 100 × $1.50 = $1,500,000. The number of Caledonia shares that could be purchased with the inflow, using the average share price, is $1,500,000 / $2 = 750,000. The net increase in common shares outstanding would have been 1,000,000 − 750,000 = 250,000.

Diluted EPS = $380,000 / (2,300,000 + 250,000) = $0.15.

This question tested from Session 8, Reading 32, LOS h

Question 69 - #97852

Your answer: A was correct!

The gross profit margin is used to measure a firm's operating profitability, not operating efficiency.

This question tested from Session 8, Reading 35, LOS d

Question 70 - #97301

Your answer: A was incorrect. The correct answer was B)

Inventories are valued on the balance sheet at the lower of cost or net realizable value. Net realizable value is equal to $3,150,000 ($3,500,000 selling price – $300,000 completion costs – $50,000 disposal costs). Since the original cost of $3,200,000 exceeds the net realizable value of $3,150,000, a $50,000 write-down is necessary. An inventory write-down has no impact on the quick ratio since inventory is excluded from both the numerator and denominator of the quick ratio.

This question tested from Session 8, Reading 33, LOS h

Question 71 - #97957

Your answer: A was incorrect. The correct answer was B) 484,000.

[400,000 shares × 12 months + 40,000 × 12 months + 90,000 × 6 months - (12,000 × 1 months)] divided by 12 = 484,000 shares.

This question tested from Session 8, Reading 32, LOS g

Question 72 - #97802

Your answer: A was incorrect. The correct answer was B) $3.40.

Orange’s basic EPS ((net income – preferred dividends) / weighted average common shares outstanding) is [($7,600,000 − (10,000 × $1,000 × 0.08)] / 2,000,000 = $3.40. To check for dilution, EPS is calculated under the

$50,000 write-down No

Page 56: Financial Reporting and Analysis_The Income Statement, Balance Sheet, and Cash Flow Statement

assumption that the convertible preferred shares are converted into common shares at the beginning of the year. The preferred dividends paid are added back to the numerator of the Diluted EPS equation, and the additional common shares are added to the denominator of the equation. Orange’s if-converted EPS is $7,600,000 / (2,000,000 + 200,000) = $3.45. Because if-converted EPS is higher than basic EPS, the preferred stock is antidilutive and no adjustment is made to basic EPS.

This question tested from Session 8, Reading 32, LOS h

Question 73 - #97393

Your answer: A was incorrect. The correct answer was B) Increase in cash of $248.

CFO for Moose Printing Corporation is calculated as follows:

+Net Income $225 − A/R $55 + Inventory $33 + Depreciation $65 − A/P $25 + Wages Payable $15 − Deferred taxes $10 = $248.

The purchase of new equipment would be an investing activity and, therefore, would not be included in the CFO. Dividends paid would be a financing activity and would not be included in the CFO.

This question tested from Session 8, Reading 34, LOS e

Question 74 - #97983

Part 1) Your answer: A was incorrect. The correct answer was B) $3.75.

($120,000 − 40,000 − 5,000) / 20,000 shares = $3.75.

This question tested from Session 8, Reading 32, LOS g

Part 2) Your answer: A was correct!

($120,000) / (20,000 + 20,000 + 40,000) = $1.50.

This question tested from Session 8, Reading 32, LOS g

Question 75 - #97921

Your answer: A was correct!

The cash conversion cycle = average receivables collection period + average inventory processing period – payables payment period. The average receivables collection period = 365 / average receivables turnover or 365 / 10.5 = 34.76. The average inventory processing period = 365 / inventory turnover or 365 / 4 = 91.25. The payables payment period = 365 / payables turnover ratio = 365 / 8 = 45.63. Putting it all together: cash conversion cycle = 34.76 + 91.25 – 45.63 = 80.38.

This question tested from Session 8, Reading 35, LOS d

Question 76 - #95656

Your answer: A was incorrect. The correct answer was B) Accounts payable Current portion of long-term debt

Page 57: Financial Reporting and Analysis_The Income Statement, Balance Sheet, and Cash Flow Statement

Operating liabilities result from the operations of the firm and consist of operating and trade liabilities such as accounts payable, customer advances, and accrued liabilities. Financing liabilities are a result of prior financing inflows. Financing liabilities (current) include short-term notes payable and the current maturities of long-term debt.

This question tested from Session 8, Reading 34, LOS a

Question 77 - #98003

Your answer: A was incorrect. The correct answer was B) unusual or infrequent items.

Unusual or infrequent items appear as a component of net income from continuing operations and are reported "above the line." Extraordinary items, such as expropriations, are unusual and infrequent and appear "below the line."

This question tested from Session 8, Reading 32, LOS f

Question 78 - #97920

Your answer: A was correct!

For the current ratio to equal 2.0, current assets would need to move to $600 (or up by $100) or current liabilities would need to decrease to $250 (or down by $50). Remember that CA − CL = working capital (500 − 300 = 200).

This question tested from Session 8, Reading 35, LOS d

Question 79 - #97794

Your answer: A was incorrect. The correct answer was B) Neither basic nor diluted EPS considers antidilutive securities in its computation.

To calculate diluted EPS, dividends on convertible preferred stock and the after tax interest on convertible debt need to be added to net income in the numerator. If diluted EPS are more than basic EPS, the convertible securities are antidilutive and should NOT be used in computing diluted EPS.

This question tested from Session 8, Reading 32, LOS h

Question 80 - #97314

Your answer: A was correct!

If equity = 40%, debt must = 60%, thus 60 / 40 = 1.5.

This question tested from Session 8, Reading 33, LOS h

Question 81 - #97832

Your answer: A was incorrect. The correct answer was B) 3.018.

Current ratio = (CA / CL) = (1,660 / 550) = 3.018

This question tested from Session 8, Reading 35, LOS d

Page 58: Financial Reporting and Analysis_The Income Statement, Balance Sheet, and Cash Flow Statement

Question 82 - #97419

Your answer: A was incorrect. The correct answer was E) $10.

This question tested from Session 8, Reading 34, LOS e

Question 83 - #97724

Your answer: A was correct!

Other comprehensive income includes non-owner transactions that affect shareholders’ equity and are not recognized in net income. Dividends paid are transactions with the owners of the firm, so dividends paid are not included in other comprehensive income. Foreign currency translation gains and losses are non-owner transactions that are not recognized in net income. Thus, foreign currency translation gains and losses are included in other comprehensive income.

This question tested from Session 8, Reading 32, LOS j

Question 84 - #98223

Your answer: A was correct!

COGS = (0.65)($1,000,000) = $650,000

Inventory turnover = CGS / Inventory = $650,000 / $450,000 = 1.4444

Average Inventory Processing Period = 365 / 1.4444 = 252.7 days

This question tested from Session 8, Reading 35, LOS d

Question 85 - #97423

Your answer: A was correct!

CFF = 25(Sale of Stock) − 30(Div Paid) = -$5

This question tested from Session 8, Reading 34, LOS e

Question 86 - #97855

Your answer: A was correct!

Able’s basic earnings per share ((Net Income − Preferred Stock Dividends) / weighted average shares outstanding) for 2004 was [($720,000 − ($500 × 6,000 × 0.03) − ($1,000 × 1,000 × 0.08)] / 1,000,000 = $0.55. If the convertible preferred were converted to common stock on January 1, 6,000 × 40 = 240,000 additional shares would have been issued. Also, dividends on the convertible preferred would not have been paid.

So diluted EPS was ($720,000 − 80,000) / (1,000,000 + 240,000) = $0.52.

Purchase of equipment -$50Fixed asset sold $60

CFI $10

Page 59: Financial Reporting and Analysis_The Income Statement, Balance Sheet, and Cash Flow Statement

This question tested from Session 8, Reading 32, LOS g

Question 87 - #97060

Your answer: A was incorrect. The correct answer was B) increase.

The DuPont equation shows clearly that ROE will increase as profit margin increases, as long as asset turn and leverage do not fall.

This question tested from Session 8, Reading 35, LOS f

Question 88 - #97788

Your answer: A was incorrect. The correct answer was B) $2.96.

Doors basic earnings per share (EPS) was ($372,000 / 100,000 =) $3.72. If the bonds were converted, interest payments would not have been made. Net income is increased by the interest paid on the bonds net of taxes: $372,000 + (($1000 × 2,000 × 0.06) × (1 − 0.40)) = $444,000.

Diluted EPS was $444,000 / (100,000 + (2,000 × 25)) = $2.96.

This question tested from Session 8, Reading 32, LOS h

Question 89 - #97386

Your answer: A was correct!

Investing Activities: $10,000 – $180,000 = -$170,000 cash flow from investing or $170,000 used

Financing Activities: $38,000 − $75,000 = -$37,000 cash flow from financing or $37,000 used

Note that the question asked for net cash used therefore this is a positive cash outflow.

This question tested from Session 8, Reading 34, LOS a

Question 90 - #97901

Your answer: A was incorrect. The correct answer was B)

A current liability is expected to be settled within one year or operating cycle, whichever is greater. It is not necessary to settle a current liability with cash. There are a number of ways to settle a current liability. For example, unearned revenue is a liability that is settled by providing goods or services.

This question tested from Session 8, Reading 33, LOS d

Question 91 - #98034

Your answer: A was incorrect. The correct answer was B)

No No

Understated Overstated

Page 60: Financial Reporting and Analysis_The Income Statement, Balance Sheet, and Cash Flow Statement

When payment is received, the firm has an obligation to provide the service. This obligation is reported as a liability ‘unearned revenue’ as a liability, offsetting the increase in cash. If they book the revenue and estimated expenses of providing the service this will overstate equity (assuming revenue greater than expected expense) and liabilities will be understated.

This question tested from Session 8, Reading 33, LOS c

Question 92 - #97814

Your answer: A was incorrect. The correct answer was C) None of these choices are correct.

Anti-dilutive is when dilutive EPS > basic EPS. When calculating diluted EPS, you must add the shares created from the conversion of the bonds to the denominator and the interest (1 – tax rate) to the numerator.

This question tested from Session 8, Reading 32, LOS h

Question 93 - #97871

Your answer: A was correct!

Current liabilities are used in the denominator for the: current, quick, and cash ratios.

This question tested from Session 8, Reading 35, LOS d

Question 94 - #97932

Your answer: A was correct!

ROA = (EBIT / average total assets) which measures management's ability and efficiency in using the firm's assets to generate operating profits. Other ratios that measure liquidity (if a company can pay its current bills) besides the quick, cash, and current ratios are the: receivables turnover, inventory turnover, and payables turnover ratios.

This question tested from Session 8, Reading 35, LOS d

Question 95 - #97713

Your answer: A was correct!

Although manipulation of cash flow can occur, the P/E ratio is easier to manipulate because earnings are based on the numerous estimates and judgments of accrual accounting. EPS does not facilitate comparisons among firms. Two firms may have the same amount of earnings but the number of shares outstanding may differ significantly.

This question tested from Session 8, Reading 35, LOS g

Question 96 - #98023

Your answer: A was correct!

(802)(6) = 4,812

4,812 / 10 = 481.2

Page 61: Financial Reporting and Analysis_The Income Statement, Balance Sheet, and Cash Flow Statement

802 − 481 = 321 or [(10 − 6) / 10] × 802 = 321

This question tested from Session 8, Reading 32, LOS h

Question 97 - #97779

Your answer: A was correct!

Dilutive securities are securities that decrease EPS if they are exercised or converted to common stock. Stock options, warrants, convertible debt, and convertible preferred stock are examples of potentially dilutive securities. Note that if diluted EPS when considering the convertible bonds is greater than basic EPS, the convertible bonds would be antidilutive and should not be treated as common stock in computing diluted EPS.

This question tested from Session 8, Reading 32, LOS h

Question 98 - #97833

Your answer: A was correct!

Quick ratio = (cash + marketable securities + receivables) / CL = (450 + 0 + 660) / 550 = 2.018

This question tested from Session 8, Reading 35, LOS d

Question 99 - #95268

Your answer: A was incorrect. The correct answer was B) Dividends paid.

Dividends paid is not a component of cash flow from investing, it is a component of cash flow from financing. The other items are all components of cash flow from investing.

This question tested from Session 8, Reading 34, LOS a

Question 100 - #94229

Your answer: A was incorrect. The correct answer was B) $15,000.

first two years = (60,000 − 0) / 10 = 6,000 per year

yr. 2006 = (60,000 − 12,000 − 3,000) / 3 = 15,000

This question tested from Session 8, Reading 32, LOS d

Question 101 - #97801

Your answer: A was incorrect. The correct answer was B) both are incorrect.

Acquired copyrights and patents are intangible assets that can be separately identified. Identifiable intangible assets are amortized over their useful lives.

This question tested from Session 8, Reading 33, LOS e

Question 102 - #97061

Page 62: Financial Reporting and Analysis_The Income Statement, Balance Sheet, and Cash Flow Statement

Your answer: A was incorrect. The correct answer was B) gross profit margin increased in 20X1 but net profit margin decreased.

Royal’s gross profit margin (gross profit / sales) was higher in 20X1 (34 / 82 = 41.5%) than in 20X0 (31 / 78 = 39.7%), but net profit margin (earnings after taxes / sales) declined from 7 / 78 = 9.0% in 20X0 to 6 / 82 = 7.3% in 20X1.

This question tested from Session 8, Reading 35, LOS e

Question 103 - #98227

Your answer: A was incorrect. The correct answer was C) 219.0 days.

Receivables turnover = $250,000 / $150,000 = 1.66667

Collection period = 365 / 1.66667 = 219 days

This question tested from Session 8, Reading 35, LOS d

Question 104 - #98039

Your answer: A was incorrect. The correct answer was C) Treasury Stock method.

The treasury stock method assumes the hypothetical funds received by the company from the exercise of the options are used to purchase shares of the company's common stock in the market at the average market price.

This question tested from Session 8, Reading 32, LOS h

Question 105 - #95812

Your answer: A was incorrect. The correct answer was C)

U.S. GAAP treats dividends paid as CFF whereas IAS GAAP treats dividends paid as either CFO or CFF.

This question tested from Session 8, Reading 34, LOS c

Question 106 - #98000

Your answer: A was incorrect. The correct answer was C) $1.74.

Savannah Corp.’s basic EPS ((net income – preferred dividends) / weighted average number of common shares outstanding) was (($122,000 − $35,000) / $50,000 =) $1.74.

This question tested from Session 8, Reading 32, LOS g

Question 107 - #98088

Your answer: A was incorrect. The correct answer was C) 1,200,000.

CFF CFF or CFO

Page 63: Financial Reporting and Analysis_The Income Statement, Balance Sheet, and Cash Flow Statement

This question tested from Session 8, Reading 32, LOS g

Question 108 - #97780

Your answer: A was incorrect. The correct answer was C) potentially dilutive security.

Dilutive securities are securities that decrease EPS if they are exercised or converted to common stock. Stock options, warrants, convertible debt, and convertible preferred stock are examples of potentially dilutive securities. Note that if diluted EPS when considering the convertible preferred stock is greater than basic EPS, the convertible preferred stock would be antidilutive and should not be treated as common stock in computing diluted EPS.

This question tested from Session 8, Reading 32, LOS h

Question 109 - #96769

Your answer: A was incorrect. The correct answer was C) 0.18.

ROE = (EAT / S)(S / A)(A / EQ) ROE = (0.1)(1.2)(1.5) = 0.18

This question tested from Session 8, Reading 35, LOS f

Question 110 - #93882

Your answer: A was incorrect. The correct answer was B)

Only the acquisition of common stock of the affiliate for $2.7 million and the purchase of the patent for $3.3 million are included in cash flow from investing activities. Since the acquisition of the stock purchase was financed with a bank loan, $2.7 million will be reported as a financing inflow. Both remaining transactions are non-cash transactions and are disclosed in the notes to or in a supplementarty schedule to the cash flow statement.

This question tested from Session 8, Reading 34, LOS b

Question 111 - #98081

Your answer: A was correct!

1/1 5,500 shares issued (includes 10% stock dividend on 6/1) × 12 = 66,000

7/1 1,000 shares repurchased × 6 months = 6,000

66,000 − 6,000 = 60,000 shares

60,000 shares / 12 months = 5,000 average shares

$6.0 million outflow $2.7 million inflow

Page 64: Financial Reporting and Analysis_The Income Statement, Balance Sheet, and Cash Flow Statement

This question tested from Session 8, Reading 32, LOS g

Question 112 - #97403

Your answer: A was correct!

Cash flow from operations (CFO) using the indirect method is computed by taking net income plus non-cash expenses (i.e. depreciation) less gains from the equipment sale. Note that cash flow from operations must be adjusted downward for the amount of the gain on the sale of the equipment. Cash flow from operations is ($850,000 + $200,000 – ($100,000 − $50,000)) = $1,000,000. Note that interest and income taxes paid are expenses shown on the income statement and will already be factored into net income. The other information relates to financial and investing cash flows.

This question tested from Session 8, Reading 34, LOS e

Question 113 - #98041

Your answer: A was incorrect. The correct answer was B) 104,000.

Only the stock options with an exercise price of $20 are dilutive. The additional shares of 4,000 (20,000 − [(20,000 × 20) / 25]) are added to the 100,000 common shares outstanding.

This question tested from Session 8, Reading 32, LOS h

Question 114 - #97933

Your answer: A was correct!

Indigo’s weighted average common shares = [(500,000 × 12) + (200,000 × 6) – (100,000 × 3)] / 12 = 575,000. Basic EPS = $5,600,000 / 575,000 = $9.74.

For diluted EPS, assume the bonds were converted on January 1, and that interest payments were not made on the bonds. Increasing net income by the amount of bond interest net of tax = $5,600,000 + [6,000 × $1,000 × 0.05 × (1 − 0.40)] = $5,780,000. Diluted EPS = $5,780,000 / (575,000 + 120,000) = $8.32.

This question tested from Session 8, Reading 32, LOS h

Question 115 - #98130

Your answer: A was correct!

Gains or losses will be found in cash flow from investments.

This question tested from Session 8, Reading 34, LOS a

Question 116 - #98048

Your answer: A was incorrect. The correct answer was B) unusual in nature and infrequent in occurrence.

Extraordinary items are unusual and infrequent events that are reported separately, net of tax "below the line." Examples are expropriations by foreign governments and uninsured losses from earthquakes, eruptions, and tornadoes.

Page 65: Financial Reporting and Analysis_The Income Statement, Balance Sheet, and Cash Flow Statement

This question tested from Session 8, Reading 32, LOS f

Question 117 - #97906

Your answer: A was incorrect. The correct answer was C) Sales/Total Assets

Sales/Total Assets, or Total Asset Turnover is a measure of operating efficiency, not operating profitability.

This question tested from Session 8, Reading 35, LOS d

Question 118 - #98040

Your answer: A was correct!

The numerator will increase because earnings available to the common shareholder are increased by the reduction in preferred dividends. The denominator increases because the weighted average number of shares increases upon conversion of the preferred stock.

This question tested from Session 8, Reading 32, LOS h

© 2010 Kaplan Schweser

Page 66: Financial Reporting and Analysis_The Income Statement, Balance Sheet, and Cash Flow Statement

Kaplan Schweser Printable Exams - 2011 CFA Level 1

You can print this page by going to file -> print in your internet browser.

Question 1 - #97985

An airplane manufacturing company routinely builds fighter jets for the U.S. armed forces. It takes fourteen months to build one jet, and the government pays for them in installments over the fourteen-month period. Which revenue recognition method should be used?

Question 2 - #98058

Extraordinary items are:

Question 3 - #97966

Oregon Corp.’s stock transactions during the year were as follows:

January 1: 320,000 shares outstanding. April 1: 1-for-2 reverse stock split occurred. July 1: Acquisition of Smith, Inc. in exchange for issuance of 60,000 shares. October 1: 30,000 shares issued for cash.

What is Oregon’s weighted average number of shares outstanding?

Question 4 - #98080

Which of the following statements about a classified balance sheet is least likely accurate? A classified balance sheet:

Question 5 - #97831

Selected information from Baltimore Corp’s financial activities in the year 2004 is as follows:

Net income was $4,200,000 .

750,000 shares of common stock were outstanding on January 1.

A) Installment sales method.B) Completed contract method.C) Percentage-of-completion method.

A) unusual in nature or infrequent.B) unusual in nature and infrequent.C) related to the normal course of business.

A) 197,500.B) 167,500.C) 250,000.

A) groups accounts by subcategories.B) distinguishes between current and noncurrent assets.C) presents the net equity of each asset by subtracting its related liability.

Page 67: Financial Reporting and Analysis_The Income Statement, Balance Sheet, and Cash Flow Statement

The average market price per share was $50 in 2004.

Dividends were paid in 2004.

10,000 warrants, which allowed the holder to purchase 10 shares of common stock for each warrant held at a price of $40 per common share, were outstanding the entire year.

Baltimore’s diluted earnings per share (Diluted EPS) for 2004 is closest to:

Question 6 - #97752

A complex capital structure, for purposes of determining disclosure of diluted Earnings Per Share, is distinguished from a simple capital structure by the:

Question 7 - #98004

Under the cost recovery method, profit is recognized:

Question 8 - #97822

How will dilutive securities affect earnings per share (EPS) when determining diluted earnings per share?

Question 9 - #94899

How would a stock split be reported on the statement of cash flows? A stock split would:

Question 10 - #97797

Which of the following statements regarding basic and diluted EPS is least accurate?

A) $5.60.B) $5.45.C) $4.94.

A) company's use of debt to finance its operations.B) company having issued warrants, convertible securities, or options.C) company having preferred stock outstanding.

A) after the amount of cost has been collected. B) at time of delivery. C) as collection occurs.

A) Decrease EPS.B) Increase EPS.C) Either decrease or increase EPS depending upon if the security is dilutive or antidilutive.

A) be reported as a source of cash in the cash flows from financing.B) not be reported on the statement of cash flows because it is a non-cash event.C) be reported as a use of cash in the cash flows from financing.

A) Antidilutive securities decrease EPS if they are exercised or converted.B) A simple capital structure contains no potentially dilutive securities.C) Dilutive securities decrease EPS if they are exercised or converted to common stock.

Page 68: Financial Reporting and Analysis_The Income Statement, Balance Sheet, and Cash Flow Statement

Question 11 - #93535

Which of the following items would NOT be included in cash flow from investing?

Question 12 - #97896

>An analyst has gathered the following data about a company:

Average receivables collection period of 95 days. Average inventory processing period of 183 days. A payables payment period of 274 days.

What is their cash conversion cycle?

Question 13 - #97698

An analysis of the industry reveals that firms have been paying out 45% of their earnings in dividends, asset turnover = 1.2; asset-to-equity (A/E) = 1.1 and profit margins are 8%. What is the industry’s projected growth rate?

Question 14 - #97690

A firm’s financial statements reflect the following:

Based on this information, what is the firm’s sustainable growth rate?

Question 15 - #97300

Impala Corporation reported the following financial information:

A) Proceeds related to acquisitions.B) Selling stock of the company.C) Buying or selling a building.

A) -4 days.B) 4 days.C) 186 days.

A) 5.81%. B) 4.55%. C) 4.95%.

EBIT $2,000,000 Sales $16,000,000 Interest expense $900,000 Total assets $12,300,000 Equity $7,000,000 Effective tax rate 35% Dividend payout rate 28%

A) 8.82%.B) 7.35%.C) 10.63%.

2006 2007

Page 69: Financial Reporting and Analysis_The Income Statement, Balance Sheet, and Cash Flow Statement

Calculate Impala’s insurance expense and interest expense for the year ended December 31, 2007.

Question 16 - #97975

The following data pertains to the McGuire Company:

Net income equals $15,000. 5,000 shares of common stock issued on January 1. 10% stock dividend issued on June 1. 1000 shares of common stock were repurchased on July 1. 1000 shares of 10%, par $100 preferred stock each convertible into 8 shares of common were outstanding the whole year.

What is the company’s basic earnings per share (EPS)?

Question 17 - #95772

In its first year of business, Digmore Corporation’s balance sheet shows gross fixed assets at $90 million and accumulated depreciation of $10 million. If the estimated salvage value of these assets is $10 million, and the original estimated useful life is 8 years, what method of depreciation did Digmore most likely use?

Question 18 - #96532

If a company has a net profit margin of 15%, an asset turnover ratio of 4.5 and a ROE of 18%, what is the equity multiplier?

Question 19 - #97893

Nichols Company’s net income for 20X6 was $978,000 with 1,250,000 shares outstanding. The average share price in 20X6 was $8.50. Nichols issued 2,000 warrants to purchase 100 shares each for $10 per share in 20X5. Nichols Company’s diluted earnings per share (diluted EPS) for 20X6 is closest to:

Balance sheet values as of December 31: Prepaid insurance $650,000 $475,000 Interest payable 250,000 300,000

Cash flows for the year ended December 31: Insurance premiums paid $845,000 $750,000 Interest paid 900,000 900,000

Insurance expense Interest expense

A) $1,020,000 $950,000

B) $925,000 $950,000

C) $925,000 $850,000

A) $1.00.B) $2.50.C) $1.20.

A) Units of production.B) Double-declining-balance.C) Straight Line.

A) 2.667.B) 0.267.C) 0.523.

A) $0.777.

Page 70: Financial Reporting and Analysis_The Income Statement, Balance Sheet, and Cash Flow Statement

Question 20 - #97996

An analyst gathered the following information about a company:

01/01/06 - 20,000 shares issued and outstanding 04/01/06 - 5.0% stock dividend 07/01/06 - 5,000 shares repurchased 10/01/06 - 2:1 stock split

What is the company’s weighted average number of shares outstanding at the end of 2006?

Question 21 - #97827

An analyst has gathered the following information about Artcraft, Inc. for the year:

Net income of $30,000. 5,000 shares of common stock and 500 shares of 8%, $90 par convertible preferred stock outstanding during the whole year. Each share of convertible preferred can be converted into 4 shares of common stock. Last year, Artcraft issued at par, $60,000 total face value of 6.0% convertible bonds, with each of the 60 bonds convertible into 110 shares of the Artcraft common stock.

If Artcraft's effective tax rate is 40%, what will Artcraft report as diluted earnings per share (EPS)?

Question 22 - #98014

The Fischer Company had net income of $1,500,000. Fischer paid preferred dividends of $5 on each of the 100,000 preferred shares. There are 1 million Fischer common shares outstanding. In addition to the common and preferred stock, Fischer has $25 million of 4% bonds outstanding. The face value of each bond is $1,000. Each bond is convertible into 40 common shares. If Fischer's tax rate is 40%, determine its basic and diluted earnings per share (EPS)?

Question 23 - #93588

Which of the following does NOT represent a cash flow relating to operating activity?

B) $0.782.C) $0.793.

A) 47,000.B) 39,500.C) 37,000.

A) $2.36.B) $3.12.C) $3.37.

Basic EPS Diluted EPS

A) $1.00 $1.25

B) $1.50 $1.25

C) $1.00 $0.80

A) Cash received from customers.B) Dividends paid to stockholders.C) Interest paid to bondholders.

Page 71: Financial Reporting and Analysis_The Income Statement, Balance Sheet, and Cash Flow Statement

Question 24 - #97691

A firm’s financial statements reflect the following:

Which of the following is the closest estimate of the firm’s sustainable growth rate?

Question 25 - #119452

Comparing a company’s ratios with those of its competitors is best described as:

Question 26 - #98033

When a firm recognizes revenue in excess of expenses on a product not covered by a warranty before cash is collected, what is the impact on the firm’s assets and liabilities, ignoring taxes?

Question 27 - #97118

Selected financial information gathered from the Matador Corporation follows:

Using only the data presented, which of the following statements is most correct?

Net profit margin 15% Sales $10,000,000 Interest payments $1,200,000 Avg. assets $15,000,000 Equity $11,000,000 Avg. working capital $800,000 Dividend payout rate 35%

A) 10%.B) 8%.C) 9%.

A) cross-sectional analysis. B) longitudinal analysis.C) common-size analysis.

Assets Liabilities

A) Increase Increase

B) No effect Increase

C) Increase No effect

2007 2006 2005 Average debt $792,000 $800,000 $820,000 Average equity $215,000 $294,000 $364,000 Return on assets 5.9% 6.6% 7.2% Quick ratio 0.3 0.5 0.6 Sales $1,650,000 $1,452,000 $1,304,000 Cost of goods sold $1,345,000 $1,176,000 $1,043,000

A) Leverage has declined.B) Return on equity has improved.C) Gross profit margin has improved.

Page 72: Financial Reporting and Analysis_The Income Statement, Balance Sheet, and Cash Flow Statement

Question 28 - #97874

Given the following information about a company:

Receivables turnover = 10 times. Payables turnover = 12 times. Inventory turnover = 8 times.

What are the average receivables collection period, the average payables payment period, and the average inventory processing period respectively?

Question 29 - #97059

The traditional DuPont equation shows ROE equal to:

Question 30 - #95657

Changes in asset lives and salvage value are changes in accounting:

Question 31 - #98064

Extraordinary items are:

Question 32 - #97816

An analyst has gathered the following information about a company:

Average Receivables Collection Period

Average Payables Payment Period

Average Inventory Processing Period

A) 37 30 52

B) 37 30 46

C) 37 45 46

A) EBIT/sales × sales/assets × assets/equity × (1 – tax rate).B) net income/assets × sales/equity × assets/sales.C) net income/sales × sales/assets × assets/equity.

A) estimates and no specific disclosures are required. B) principle and specific disclosures are required.C) estimates and specific disclosures are required.

A) unusual or infrequent.B) unusual and infrequent.C) reported above the line.

Balance Sheet Assets

Cash 100Accounts Receivable 750Marketable Securities 300Inventory 850Property, Plant & Equip 900Accumulated Depreciation (150)

Page 73: Financial Reporting and Analysis_The Income Statement, Balance Sheet, and Cash Flow Statement

What is the inventory turnover ratio?

Question 33 - #98089

As a general rule, revenue is normally recognized when it is:

Question 34 - #98008

Which of the following statements regarding first in, first out (FIFO) is least accurate?

Question 35 - #96458

When a U.S. company pays dividends to its stockholders, which type of cash flow does this represent?

Question 36 - #98045

BWT, Inc. shows the following data in its financial statements at the end of the year.

Total Assets 2750

Liabilities and EquityAccounts Payable 300Short-Term Debt 130Long-Term Debt 700Common Stock 1000Retained Earnings 620

Total Liab. and Stockholder's equity 2750

Income Statement Sales 1500COGS 1100Gross Profit 400SG&A 150Operating Profit 250Interest Expense 25Taxes 75Net Income 150

A) 1.59.B) 1.29.C) 0.77.

A) earned.B) measurable.C) realizable and earned.

A) Ending inventory consists of the cost of the most recent purchases.B) Cost of goods sold consists of the costs of the first purchases.C) Items sold are a mix of the cost of the purchases.

A) Operating.B) Investing.C) Financing.

Page 74: Financial Reporting and Analysis_The Income Statement, Balance Sheet, and Cash Flow Statement

Assume all securites were outstanding at the beginning of the year:

6.125% convertible bonds, convertible into 33 shares of common stock. Issue price $1,000, 100 bonds outstanding. 6.25% convertible preferred stock, $100 par, 2,315 shares outstanding. Convertible into 3.3 shares of common stock, Issue price $100. 8% convertible preferred stock, $100 par, 2,572 shares outstanding. Convertible into 5 common shares, Issue price $80. 9,986 warrants are outstanding with an exercise price of $38. Each warrant is convertible into 1 share of common. Average market price of common is $52.00 per share. Common shares outstanding at the beginning of the year were 40,045. Net Income for the period was $200,000, while the tax rate was 40%.

Part 1) What were the preferred dividends paid this whole year?

Part 2) What was the after-tax interest charge?

Part 3) How many new shares had to be issued to facilitate warrant conversion?

Part 4)

What were the basic and diluted EPS for the year?

Question 37 - #97517

Mark Industries' income statement and related notes for the year ended December 31 are as follows (in $):

During the year:

Wages Payable increased $100,000. Accumulated Depreciation increased $2,500,000. Interest Payable decreased $200,000. Income Taxes Payable increased $500,000. Dividends of $100,000 were declared and paid.

Under U.S. GAAP, Mark Industries’ cash flow from operations (CFO) for the year ended December 31 was:

A) $14,469.B) $20,576.C) $35,045.

A) $6,215.B) $2,450.C) $3,675.

A) 13,665.B) 2,689.C) 9,986.

Basic EPS Diluted EPS

A) $3.97 $3.06

B) $4.12 $3.06

C) $4.12 $2.95

Sales 42,000,000 Cost of Goods Sold (32,000,000) Wages Expense (1,500,000) Depreciation Expense (2,500,000) Interest Expense (1,000,000) Income Tax Expense (2,000,000) Net Income 3,000,000

Page 75: Financial Reporting and Analysis_The Income Statement, Balance Sheet, and Cash Flow Statement

Question 38 - #97762

In applying the treasury stock method, if warrants allow the purchase of 1 million shares at $42 per share when the average price is $56 per share, how many shares will be added to the firm’s weighted average number of shares outstanding?

Question 39 - #97420

An analyst contemplates using the indirect method to create the projected statement of cash flows. She decides to research the differences between the direct and indirect methods. Which of the following is least likely a component of the statement of cash flows under the direct method?

Question 40 - #97670

An analyst has gathered the following information about a firm:

Net sales of $500,000. Cost of goods sold = $250,000. EBIT of $150,000. EAT of $90,000.

What is this firm’s operating profit margin?

Question 41 - #97992

An analyst has gathered the following information about Zany Corp.

Net income of $200,000 for the year ended December 31, 2004.

During 2004, 50,000 common shares were outstanding.

Zany has 10,000 shares of 7%, $50 par convertible preferred stock outstanding, each convertible into two shares of common.

5,000 warrants are outstanding with an exercise price of $24. Each warrant is convertible into one common share.

The average market price per common share during 2004 was $20.

Calculate Zany's basic and diluted earnings per share (EPS) for 2004.

A) $5,900,000.B) $4,800,000.C) $4,400,000.

A) 1,000,000.B) 420,000.C) 250,000.

A) Payment of dividends.B) Net income.C) Property, Plant, & Equipment.

A) 18%.B) 30%.C) 50%.

Basic EPS Diluted EPS

A) $3.30 $2.86

Page 76: Financial Reporting and Analysis_The Income Statement, Balance Sheet, and Cash Flow Statement

Question 42 - #97305

Common size balance sheets express all balance sheet items as a percentage of:

Question 43 - #97409

Consider the following:

Which of these statements regarding a common-size cash flow statement is (are) CORRECT?

Question 44 - #97895

Are the quick ratio and the debt-to-capital ratio used primarily to assess a company’s ability to meet short-term obligations?

Question 45 - #93954

Slovac Company purchased a machine that has an estimated useful life of eight years for $7,500. Its salvage value is estimated at $500.

What is the depreciation expense for the second year, assuming Slovac uses the double-declining balance method of depreciation?

Question 46 - #98043

On January 1, 20X7, Omega Corporation paid $45,000 to renew its property insurance for 3 years. What amount of insurance expense should Omega report for the year-ended December 31, 20X7 and what is the balance of Omega’s prepaid insurance account on December 31, 20X8?

B) $3.30 $2.00

C) $4.00 $2.86

A) sales.B) assets.C) equity.

Statement #1: One approach to presenting a common-size cash flow statement is to express each inflow of cash as a percentage of total cash inflows and each outflow of cash as a percentage of total cash outflows.

Statement #2: Expressing each line item of the cash flow statement as a percentage of revenue is useful in forecasting future cash flows.

A) Only statement #1 is correct.B) Only statement #2 is correct.C) Both statements are correct.

Quick ratio Debt-to-capital ratio

A) Yes Yes

B) No Yes

C) Yes No

A) $1,438.B) $1,406.C) $1,875.

Page 77: Financial Reporting and Analysis_The Income Statement, Balance Sheet, and Cash Flow Statement

Question 47 - #97869

Young Distributors, Inc. issued convertible bonds two years ago, and those bonds are the only potentially dilutive security Young has issued. In 20X5, Young’s basic earnings per share (EPS) and diluted EPS were identical, but in 20X4 they were different. Which of the following factors is least likely to explain the difference between basic and diluted EPS? The:

Question 48 - #97414

The Beeline Company has the following balance sheet and income statement.

The cash flow from operations for 2004 is:

Question 49 - #98052

Insurance expense Prepaid insurance

A) $15,000 $15,000

B) $15,000 $30,000

C) $45,000 $15,000

A) bonds were redeemed by Young Distributors at the beginning of 2005.B) bonds were antidilutive in 2005 but not in 2004.C) average market price of Young common stock increased in 20X5.

Beeline Company Balance Sheet As of December 31, 2004

2003 2004 2003 2004 Cash $50 $60 Accounts payable $100 $150Accounts receivable 100 110 Long-term debt 400 300Inventory 200 180 Common stock 50 50

Retained earnings 400 500 Fixed assets (gross) 800 900 Total liabilities and equity $950 $1,000Less: Accumulated depreciation 200 250Fixed assets (net) 600 650 Total assets $950 $1,000

Beeline Company Income Statement For year ended December 31, 2004

Sales $1,000Less: COGS 600Depreciation 50Selling, general, and administrative expenses 160Interest expense 23 Income before taxes $167Less tax 67 Net income $100

A) $260.B) $210.C) $100.D) $150.E)F)

Page 78: Financial Reporting and Analysis_The Income Statement, Balance Sheet, and Cash Flow Statement

Cash collection is a critical event for income recognition under the:

Question 50 - #97416

Given the following information, what is the adjustment to net income when calculating cash flow from operations using the indirect method?

Increase in accounts payable of $25. Sold one share of stock for $15. Paid dividends of $10 to shareholders. Depreciation expense of $100. Increase in inventory of $20.

Question 51 - #97669

Given the following income statement:

What are the gross profit margin and operating profit margin?

Question 52 - #97399

An analyst has gathered the following information about a company:

Cost-Recovery Method Installment Method

A) No Yes

B) Yes Yes

C) Yes No

A) -$50.B) +$105.C) -$95.

Net Sales 200Cost of Goods Sold 55Gross Profit 145Operating Expenses 30Operating Profit (EBIT) 115Interest 15Earnings Before Taxes (EBT) 100Taxes 40Earnings After Taxes (EAT) 60

Gross Profit Margin Operating Profit Margin

A) 2.630 1.226

B) 0.725 0.575

C) 0.379 0.725

Income Statement 2005 Sales $908ExpensesCOGS $512Depreciation 6Selling, General & Admin. 129

Page 79: Financial Reporting and Analysis_The Income Statement, Balance Sheet, and Cash Flow Statement

Note: the dividend payout ratio equals 45%.

What is the net increase or decrease in cash?

Question 53 - #97417

John Stone, CFA, is an investment advisor specializing in the preparation of company and industry reports for high net worth customers at Learmon Brothers. Currently, Stone is preparing a report on Soft Corporation, a rapidly growing software company. The explosive growth of this company was financed primarily by an initial public offering in which 3,000,000 shares were issued at a price of $20 per share on June 27, 2004. Soft Corporation received additional capital when employee stock options for 1,000,000 shares at a price of $10 were exercised on January 1, 2005. Stone realizes the importance of cash flow on a company's financial health and would like to include a projected statement of cash flows for 2005. Soft Corporation financial statements are presented in Tables 1 and 2. Included are the projected statements for the year ending December 31, 2005.

Interest 53Total expenses 700

Pre-tax income 208Taxes 83Net income $125

Balance Sheet

Assets 2004 2005 Liabilities 2004 2005Cash 60 80 Accts. Payable 100 75Accts. Rec. 140 155 Wages payable 80 85Inventories 47 72 Bonds 65 80Fixed Assets 120 160 Common Stock 40 70Accum. Depr. (29) (35) Retained Earnings 53 122Total 338 432 338 432

A) -$15.B) +$15.C) -$20.D)E) +$20.F)

Table 1 Soft Corporation Balance Sheets

as of December 31(in millions) Actual 2004 Projected 2005

Cash $24.0 $26.0 Accounts Receivable 17.0 24.0 Inventory 100.0 150.0 PP&E 100.0 125.0 Accumulated depreciation (30.0) (35.0)

Total Assets $211.0 $290.0 Accounts payable $91.0 $101.0 Long-term debt 20.0 40.0 Common stock 80.0 90.0 Retained earnings 20.0 59.0

Total liabilities and equity $211.0 $290.0

Table 2 Soft Corporation Income Statement

for Years Ended December 31 (in millions except per share data)

Actual 2004 Projected 2005Sales $80.0 $198.0 COGS (38.0) (90.0)

Gross profit $42.0 $108.0 SG&A (13.0) (30.0)

Page 80: Financial Reporting and Analysis_The Income Statement, Balance Sheet, and Cash Flow Statement

Part 1) Stone decides to use the direct method to compute Soft Corporation's projected operating cash collections. Using this method, which of the following is Soft Corporation's projected operating cash collections for the year ending December 31, 2005 (in millions)?

Part 2) Stone decides to use the direct method to compute Soft Corporation's projected cash inputs. Under this method, what will Soft Corporation's projected cash outflow inputs into the manufacturing process be for the year ending December 31, 2005 (in millions)?

Question 54 - #97748

An analyst compiled the following information from Hampshire, Inc.’s financial activities in the most recent year:

Net income was $2,800,000. 100,000 shares of common stock were outstanding on January 1. The average market price per share for the year was $250. 10,000 shares of 6%, $1,000 par value preferred shares were outstanding the entire year. 10,000 warrants, which allow the holder to purchase 10 shares of common stock for each warrant held at a price of $150 per common share, were outstanding the entire year. 30,000 shares of common stock were issued on September 1.

Hampshire, Inc.’s diluted earnings per share are closest to:

Question 55 - #100936

Information about a company’s revenue recognition policies is most likely disclosed in:

Depreciation (3.0) (5.0)Operating expenses $(16.0) $(35.0)

Interest expense $(4.0) $(5.0) Pretax Income 22.0 68.0 Income tax expense (7.0) (25.0)Net income $15.0 $43.0 EPS $2.0 $4.3 Average shares outstanding (millions) 7.5 10.0 Dividends per share $0.1 $0.4

A) 191.0.B) 1.0.C) 198.0.D) 224.0.E)F)

A) -80.0.B) +90.0.C) +140.0.D)E)F) -130.0.

A) $14.67.B) $18.38.C) $20.00.

A) the financial statement notes.B) the standard auditor’s report.C) Management’s Discussion and Analysis.

Page 81: Financial Reporting and Analysis_The Income Statement, Balance Sheet, and Cash Flow Statement

Question 56 - #94897

Which of the following transactions would least likely be reported in the cash flow statement as investing cash flows?

Question 57 - #98067

During 2007, Topeka Corporation entered into the following transactions:

Transaction #1 – Interest on a certificate of deposit owned by Topeka was credited to Topeka’s investment account.

Transaction #2 – Topeka sold 10,000 shares of common stock at $30 that had been repurchased by Topeka last year for $20.

Should Topeka recognize the results of these transactions as income on the income statement for the year ended December 31, 2007?

Question 58 - #97861

Kendall Company’s net income for 20X4 is $830,000 with 200,000 shares outstanding. Kendall has 1,000 6% convertible bonds (each bond $1,000 face value and convertible into 20 common shares) outstanding for the entire year. Kendall’s tax rate is 40%. What is Kendall Company’s diluted earnings per share for 20X4?

Question 59 - #97321

When calculating cash flow from operations (CFO) using the indirect method which of the following is most accurate?

Question 60 - #98060

Football Contractors, Inc. has contracted to build a stadium for the City of Washburn. The contract price is $100 million and costs are estimated at $60 million. Costs are not assured, however, because there is a material risk, which Football Contractors has assumed, that ground water problems might slow construction and increase costs by as much as $40 million. In 2004, the first year of the agreement, Football Contractors, Inc. billed $30 million, received a $20 million payment, and incurred $15 million in costs. For 2004 Football Contractors, Inc. should recognize revenue from the City of Washburn transaction in the amount of:

Question 61 - #97902

A) Purchase of plant and equipment used in the manufacturing process with financing provided by the seller.B) Principal payments received from loans made to others.C) Sale of held-to-maturity securities for cash.

A) Neither should be recognized.B) Both should be recognized.C) Only one should be recognized.

A) $4.15.B) $3.94.C) $3.77.

A) The indirect method requires an additional schedule to reconcile net income to cash flow.B) When recognizing a gain on the sale of fixed assets, the amount is a deduction to operating cash flows.C) In using the indirect method, each item on the income statement is converted to its cash equivalent.

A) $30 million.B) $20 million.C) $0.

Page 82: Financial Reporting and Analysis_The Income Statement, Balance Sheet, and Cash Flow Statement

What type of ratio is revenue divided by average working capital and what type of ratio is average total assets divided by average total equity?

Question 62 - #97387

Juniper Corp. has the following transactions in 2005.

Juniper’s equipment with a book value of $55,000 was sold for $85,000 cash. A parcel of land was purchased for $100,000 worth of Juniper common stock.

ABC company paid Juniper preferred dividends of $40,000.

Juniper declared and paid a $100,000 cash dividend.

Using the indirect method, what is cash flow from financing (CFF) for Juniper for 2005?

Question 63 - #97879

Juniper Corp’s stock transactions during the year 20X4 were as follows:

January 1 540,000 shares issued and outstanding March 1 50 percent stock dividend July 1 180,000 treasury shares reacquired October 1 60,000 treasury shares reissued

When computing for earnings per share (EPS) computation purposes, what was Juniper’s weighted average number of shares outstanding during 20X4?

Question 64 - #97760

Selected information from Feder Corp.’s financial activities for the year is as follows:

Net income was $7,650,000.

1,100,000 shares of common stock were outstanding on January 1.

The average market price per share was $62.

Dividends were paid during the year.

The tax rate was 40%.

Revenue / Average working capital

Average total assets / Average total equity

A) Activity ratio Liquidity ratio

B) Activity ratio Solvency ratio

C) Profitability ratio Solvency ratio

A) -$60,000.B) -$15,000.C) -$115,000.D)E)F) -$100,000.

A) 930,000.B) 870,000.C) 735,000.

Page 83: Financial Reporting and Analysis_The Income Statement, Balance Sheet, and Cash Flow Statement

10,000 shares of 6% $1,000 par value preferred shares convertible into common shares at a rate of 20 common shares for each preferred share were outstanding for the entire year.

70,000 options, which allow the holder to purchase 10 shares of common stock at an exercise price of $50 per common share, were outstanding the entire year.

Feder Corp.’s diluted earnings per share (EPS) was closest to:

Question 65 - #97858

Quad Associates, Inc.’s net income for 2005 was $892,000 with 400,000 shares outstanding. The tax rate was 40 percent. Quad had 2,000 six percent $1,000 par value convertible bonds that were issued in 2004. Each bond was convertible into 40 shares of common stock. Quad, Inc.’s diluted earnings per share (Diluted EPS) for 2005 was closest to:

Question 66 - #98065

Would an increase in the cost of raw materials used in the production of inventory and would an increase in marketing expenses result in lower gross profit?

Question 67 - #97278

Carpenter Corporation reported the following statement of shareholders’ equity as of December 31, 2006:

During 2007, Carpenter:

earned net income of $1,700,000. declared dividends of $300,000. $75,000 of the dividends remain unpaid. purchased held-to-maturity securities for $100,000. The securities have a fair value of $110,000 at year-end. purchased available-for-sale securities for $250,000. The securities have a fair value of $225,000 at year-end. translated the financial statements of a foreign subsidiary and calculated a $90,000 unrealized gain. purchased treasury stock for $75,000. The stock was valued at $60,000 when issued.

Calculate Carpenter’s retained earnings and accumulated other comprehensive income as of December 31, 2007.

A) $5.87.B) $4.91.C) $5.32.

A) $2.23.B) $2.41.C) $2.01.

Increase in raw materials cost

Increase in marketing expense

A) No Yes

B) Yes No

C) Yes Yes

Common stock at par $600,000 Additional paid-in-capital 900,000 Treasury stock (200,000) Retained earnings 10,500,000 Accumulated other comprehensive income 450,000

$12,250,000

Retained earnings Accumulated other comprehensive income

Page 84: Financial Reporting and Analysis_The Income Statement, Balance Sheet, and Cash Flow Statement

Question 68 - #97412

Which of the following is CORRECT about the consideration of depreciation in the operations section of a cash flow statement?

Question 69 - #97905

Which type of a capital structure contains no dilutive securities?

Question 70 - #97783

Assume a firm with a debt to equity ratio of 0.50 and debt equal to $35 million makes a commitment to acquire raw materials with a present value of $12 million over the next 3 years. For purposes of analysis the best estimate of the debt to equity ratio should be:

Question 71 - #94804

Dart Corporation engaged in the following transactions earlier this year:

With respect to these transactions, should Dart report transaction #1 as a financing cash flow and/or transaction #2 as an investing cash flow?

Question 72 - #97826

Barracuda Corporation, a U.S. corporation, owns a subsidiary located in Germany. The German subsidiary’s financial statements are maintained in euros. If the euro recently appreciated relative to the U.S. dollar, how would the unrealized translation gain affect Barracuda’s retained earnings and total stockholders’ equity?

A) $11,900,000 $65,000

B) $11,900,000 $515,000

C) $12,125,000 $515,000

Direct Method Indirect Method

A) Does not consider Considers

B) Does not consider Does not consider

C) Considers Considers

A) Basic.B) Simple.C) Complex.

A) 0.500.B) 0.671.C) 0.573.

Transaction #1: Retired long-term debenture bonds with a face amount of $10 million by issuing 500,000 shares of common stock to the bondholders.

Transaction #2: Borrowed $5 million from a bank and used the proceeds to purchase equipment used in the manufacturing process.

A) Neither should be reported as such.B) Only one should be reported as such.C) Both should be reported as such.

Page 85: Financial Reporting and Analysis_The Income Statement, Balance Sheet, and Cash Flow Statement

Question 73 - #97962

Consider the following information on the past year’s operating performance and current capital structure for the following two companies:

Based on the information above, which of the companies has a complex capital structure?

Question 74 - #97994

At the beginning of 2004, the Alaska Corporation had 2 million shares of common stock outstanding and no preferred stock. At the end of August, 2004, Alaska issued 600,000 new shares of common stock. If Alaska reported net income equal to $8.8 million, what was the firm’s earnings per share for 2004?

Question 75 - #98001

Retrospective presentation is least likely required for a change from:

Question 76 - #97930

Wells Incorporated reported the following common size data for the year ended December 31, 20X7:

Retained earnings Total stockholders' equity

A) No effect Increase

B) No effect No effect

C) Increase Increase

Supple Moves Perfect Collection Paid no dividends Paid common & pref. div.

Ave. Stock Price of $42.00 Ave. Stock Price of $22.00 Positive net income Positive net income

110,000 warrants with an exercise price of $50.00 Convertible debt with an 8.0% coupon, conversion ratio at 10.0. 150,000 options outstanding with an exercise price of $19.50

A) Perfect Collection only.B) Supple Moves only.C) Supple Moves and Perfect Collection.

A) $3.38.B) $3.67.C) $4.00.

A) zero salvage value to positive salvage value.B) percentage-of-completion to completed contract revenue recognition.C) LIFO to average cost inventory valuation.

Income Statement %Sales 100.0Cost of goods sold 58.2Operating expenses 30.2Interest expense 0.7Income tax 5.7Net income 5.2

Page 86: Financial Reporting and Analysis_The Income Statement, Balance Sheet, and Cash Flow Statement

For 20X6, Wells reported sales of $183,100,000 and for 20X7, sales of $215,600,000. At the end of 20X6, Wells’ total assets were $75,900,000 and common equity was $37,800,000. At the end of 20X7, total assets were $95,300,000. Calculate Wells’ current ratio and return on equity ratio for 20X7.

Question 77 - #100934

Under the general principles of accrual accounting, revenue is recognized when:

Question 78 - #97329

The net income for Miller Bat Company was $3 million for the year ended December 31, 2004. Additional information is as follows:

Depreciation on fixed assets $1,500,000

Gain from cash sales of land 200,000

Increase in accounts payable 300,000

Dividends paid on preferred stock 400,000

The net cash provided by operating activities in the statement of cash flows for the year ended December 31, 2004 is:

Question 79 - #97959

At the beginning of 2004, Osami Corporation had 1.4 million shares of common stock outstanding and no preferred stock. At the end of August 2004, Osami issued 1.2 million new shares of common stock. If Osami reported net income equal to $7.2 million, what were its earnings per share (EPS) for 2004?

Balance sheet % %Cash 4.8 Accounts payable 15.0Accounts receivable 14.9 Accrued liabilities 13.8Inventory 49.4 Long-term debt 23.2Net fixed assets 30.9 Common equity 48.0Total assets 100.00 Total liabilities & equity 100.0

Current ratio Return on equity

A) 2.4 26.4%

B) 4.6 25.2%

C) 2.4 26.8%

A) earned, and expenses are recognized when incurred.B) cash is received, and expenses are recognized when cash is paid.C) the good or service is delivered or cash is received, whichever is earlier.

A) $4,200,000.B) $4,600,000.C) $4,500,000.D) $4,800,000.E)F)

A) $4.00.B) $3.33.C) $2.77.

Page 87: Financial Reporting and Analysis_The Income Statement, Balance Sheet, and Cash Flow Statement

Question 80 - #97908

Which of the following reasons is least likely a valid limitation of ratio analysis?

Question 81 - #98006

The Gaffe Company had net income of $1,500,000. Gaffe paid preferred dividends of $5 on each of the 100,000 preferred shares. Each preferred share is convertible into 20 common shares. There are 1 million Gaffe common shares outstanding. In addition to the common and preferred stock, Gaffe has $25 million of 4% bonds outstanding. If Gaffe's tax rate is 40%, what is its diluted earnings per share?

Question 82 - #98019

Red Oak Corporation is a furniture manufacturer located in Canada. Red Oak is financed with a combination of debt and equity. The debt consists of unsecured zero-coupon bonds that mature in 20 years. For income tax purposes, interest on the bonds is deductible when accrued. Red Oak’s equity consists of common stock and preferred stock. No dividends have ever been paid on Red Oak’s common stock; however, dividends are paid quarterly to the preferred shareholders. Should the accrued interest on the zero-coupon bonds and the dividends paid to the preferred shareholders be reported as a nonoperating component of Red Oak’s net income?

Question 83 - #97435

A firm has net sales of $3,500, earnings after taxes (EAT) of $1,000, depreciation expense of $500, cost of goods sold (COGS) of $1,500, and cash taxes of $500. Also, inventory decreased by $100, and accounts receivable increased by $300. What is the firm's cash flow from operations?

Question 84 - #97502

The main difference between the current ratio and the quick ratio is that the quick ratio excludes:

Question 85 - #97251

A) It is difficult to find comparable industry ratios.B) Determining the target or comparison value for a ratio is difficult.C) Calculation of ratios involves a large degree of subjectivity.

A) $1.00.B) $0.33.C) $0.50.

Accrued interest Preferred dividends

A) Yes Yes

B) No Yes

C) Yes No

A) $1,300.B) $1,200.C) $1,800.

A) inventory.B) cost of goods sold.C) assets.

Page 88: Financial Reporting and Analysis_The Income Statement, Balance Sheet, and Cash Flow Statement

At the beginning of the year, Alpha Corporation purchased 10,000 shares of Beta Corporation for $20 per share. During the year, Beta paid a $2,000 cash dividend to Alpha. At the end of the year, Beta’s stock was selling for $22 per share. What amount should Alpha recognize in its year-end income statement if the investment is treated as an available-for-sale security and what amount should be recognized in the income statement if the investment is treated as a trading security?

Question 86 - #98075

The Better Building Company has a contract to build a building for $100 million. The estimate of the cost of the project is $75 million. In the first year of the project, BB had costs of $30 million. The Better Building Company’s reported profit for the first year of the contract, using the percentage-of-completion method, is:

Question 87 - #97072

Assume that Q-Tell Incorporated is in the communications industry, which has an average receivables turnover ratio of 16 times. If the Q-Tell’s receivables turnover is less than that of the industry, Q-Tell’s average receivables collection period is most likely:

Question 88 - #97952

The SSP Company had 5 million shares outstanding on January 1. On February 15 the board of directors approved a 3:2 stock split, effective April 1. What is the weighted average number of shares outstanding for the SSP Company for year-end?

Question 89 - #97716

For the year ended December 31, 2007, Milan Company reported the following financial information:

Ignoring taxes, calculate Milan’s net income and comprehensive income for 2007.

Available-for-sale Trading security

A) $2,000 $20,000

B) $2,000 $22,000

C) $0 $22,000

A) $0.B) $10 million.C) $20 million.

A) 25 days.B) 20 days.C) 12 days.

A) 6,875,000 shares.B) 7,500,000 shares.C) 5,625,000 shares.

Gross profit from sales $600,000 Operating expenses 100,000 Unrealized loss from foreign currency translation 30,000 Dividends received from available-for-sale securities 15,000 Increase in minimum pension liability 45,000 Interest expense 25,000 Acquired treasury stock for $25,000 more than original book value 75,000 Unrealized gain from available-sale-securities 20,000

Net income Comprehensive income

Page 89: Financial Reporting and Analysis_The Income Statement, Balance Sheet, and Cash Flow Statement

Question 90 - #98084

An oil exploration company has been contracted to dig 100 exploratory holes for $200,000. The cost to complete this job is estimated to be $150,000, but the company doesn’t recognize any of the $50,000 profit until the job is completed. Which revenue recognition method is being used?

Question 91 - #98053

CPP Corporation has a contract to build a custom test chamber for a client for $100,000. CPP Corporation uses the percentage-of-completion method for accounting and estimates the total costs for the project to be equal to $80,000. CPP Corporation has promised to complete the project within three years. At year-end the customer has paid $60,000, equaling the total amount billed for the year, and total costs incurred to date are $40,000. On the income statement, net income for the year-end will be:

Question 92 - #97070

Using the following data, find the return on equity (ROE).

Question 93 - #97775

When calculating earnings per share (EPS) for firms with complex capital structures, stock options are ordinarily considered to be:

Question 94 - #97982

A firm has a weighted average number of 20,000 common shares selling at an average of $10 throughout the year and 11,000, 10%, $100 par value preferred shares. If the firm earns $210,000 after taxes, what is its Basic EPS?

A) $490,000 $2,000

B) $490,000 $435,000

C) $40,000 $44,000

A) Completed contract method.B) Cost recovery method.C) Percentage-of-completion method.

A) $20,000.B) -$10,000.C) $10,000.

Net Income Total Assets Sales Equity $2 $10 $10 $8

A) 100%.B) 25%.C) 20%.

A) potentially dilutive securities.B) antidilutive securities.C) derivative securities.

A) $7.50 / share.B) $10.50 / share.C) $5.00 / share.

Page 90: Financial Reporting and Analysis_The Income Statement, Balance Sheet, and Cash Flow Statement

Question 95 - #97303

Pacific, Inc.’s financial information includes the following, with “change” referring to the difference from the prior year (in $ millions):

Pacific, Inc.’s cash flow from operations (CFO) in millions was:

Question 96 - #97821

According to the Financial Accounting Standards Board, what is the appropriate measurement basis for equipment used in the manufacturing process and inventory that is held for sale?

Question 97 - #98082

Is the following statement an accurate description of the account format and report format of presenting a balance sheet?

“The account format presents the asset, liability, and equity accounts in a single column while a report format presents the assets, liabilities, and equity in multiple columns.”

Question 98 - #122498

Selected information from the most recent cash flow statement of Thibault Company appears below:

Net Income 27 Change in Accounts Receivable +4 Change in Accounts Payable +1 Change in Inventory +5 Loss on sale of equipment -8 Gain on sale of real estate +4 Change in Retained Earnings +21 Dividends declared and paid +4

A) $27.B) $23.C) $15.

Equipment Inventory

A) Historical cost Historical cost

B) Fair value Lower of cost or market

C) Historical cost Lower of cost or market

Account format Report format

A) Yes Yes

B) No No

C) No Yes

Cash collections €8,900Cash paid to suppliers (€3,700)Cash operating expenses (€1,500)Cash taxes paid (€2,400)Cash from operating activities €1,300

Page 91: Financial Reporting and Analysis_The Income Statement, Balance Sheet, and Cash Flow Statement

Thibault’s reinvestment ratio for this period is closest to:

Question 99 - #97413

An analyst has gathered the following information about a company:

What is the cash flow from operations?

Question 100 - #97772

Examples of potentially dilutive securities include all of the following EXCEPT:

Cash paid for plant and equipment (€2,600)Cash interest received €700Cash dividends received €600Cash from investing activities (€1,300) Cash received from debt issuance €2,000Cash interest paid (€400)Cash dividends paid (€600)Cash from financing activities €1,000 Total change in cash €1,000

A) 0.50.B) 0.75.C) 1.00.

Income Statement for the Year 2004Sales $1,500ExpensesCOGS $1,300Depreciation 30Int. Expenses 40

Total expenses 1,370Income from cont. op. 130

Gain on sale 30Income before tax 160Income tax 64Net Income $96

Additional Information:Dividends paid $30Common stock sold 20Equipment purchased 50Bonds issued 80Fixed asset sold for (original cost of $100 with accumulated depreciation of $70) 60Accounts receivable decreased by 30Inventory decreased by 20Accounts payable increased by 20Wages payable decreased by 10

A) $170.B) $135.C) $156.D) $150.E)F)

Page 92: Financial Reporting and Analysis_The Income Statement, Balance Sheet, and Cash Flow Statement

Question 101 - #98061

Which of the following statements regarding making changes in accounting principles is least accurate?

Question 102 - #97830

An analyst has gathered the following information about a firm:

Quick ratio of 0.25. Cash ratio of 0.20. $2 million in marketable securities. $10 million in cash.

What is their receivables balance?

Question 103 - #97931

A firm with a capital structure consisting of only common stock and non-convertible bonds is said to have a:

Question 104 - #97404

How does decreasing accounts payable turnover affect a company’s cash flow from financing activities and is this source of cash sustainable?

Question 105 - #97846

Moulding Company’s net income was $13,820,000 with 2,600,000 shares outstanding. The average share price for the year was $58.00. Moulding had 10,000 options to purchase 10 shares each at $40 per share outstanding the entire year. Moulding Company’s diluted earnings per share are closest to:

A) non-convertible bonds.B) convertible preferred stock.C) options.

A) Changes in accounting estimates are now treated the same as changes in accounting principles.

B) A change in accounting principle is a change from one generally accepted accounting principle to another generally accepted principle. The firm making the change must justify the change.

C) The general rule is retrospective application.

A) 5 million.B) 3 million.C) 2 million.

A) simple capital structure.B) non-diluted capital structure.C) straight capital structure.

Financing cash flow Sustainable source

A) Increase No

B) No impact Yes

C) No impact No

A) $3.71.

Page 93: Financial Reporting and Analysis_The Income Statement, Balance Sheet, and Cash Flow Statement

Question 106 - #97320

Eagle Company’s financial statements for the year ended December 31, 2005 were as follows (in $ millions):

Cash flow from operations (CFO) for Eagle Company for the year ended December 31, 2005 was (in $ millions).

Question 107 - #97911

An analyst has collected the following data about a firm:

Receivables turnover = 20 times. Inventory turnover = 16 times. Payables turnover = 24 times.

What is the cash conversion cycle?

Question 108 - #97335

Common size balance sheets express all balance sheet accounts as a percentage of:

B) $5.32.C) $5.25.

Income Statement Sales 150 Cost of Goods Sold (48) Wages Expense (56) Interest Expense (12) Depreciation (22) Gain on Sale of Equipment 6 Income Tax Expense ( 8) Net Income 10

Balance Sheet 12-31-04 12-31-05

Cash 32 52 Accounts Receivable 18 22 Inventory 46 44 Property, Plant & Equip. (net) 182 160 Total Assets 278 278 Accounts Payable 28 33 Long-term Debt 145 135 Common Stock 70 70 Retained Earnings 35 40 Total Liabilities & Equity 278 278

A) $41.B) $37.C) $22.D)E) $29.F)

A) Not enough information is given.B) 26 days.C) 56 days.

Page 94: Financial Reporting and Analysis_The Income Statement, Balance Sheet, and Cash Flow Statement

Question 109 - #97944

As of December 31, 2007, Manhattan Corporation had a quick ratio of 2.0, current assets of $15 million, trade payables of $2.5 million, and receivables of $3 million, and inventory of $6 million. How much were Manhattan’s current liabilities?

Question 110 - #97935

Firebird Company reported the following financial information at the end of 2007:

Calculate Firebird’s current assets and working capital.

Question 111 - #95536

Which of the following should be classified as cash flows from investing (CFI) by Elegant, Inc., which reports under U.S. GAAP?

Question 112 - #97934

Eagle Manufacturing Company reported the following selected financial information for 2007:

A) total assets.B) stockholders equity.C) total liabilities.

A) $12.0 million.B) $4.5 million.C) $7.5 million.

in millions Merchandise inventory $240 Minority interest 70 Cash and equivalents 275 Accounts receivable 1,150 Accounts payable 225 Property & equipment 2,160 Accrued expenses 830 Current portion of long-term debt 120 Long-term debt 1,570 Retained earnings 4,230

Current assets Working capital

A) $1,665 million $420 million

B) $1,665 million $490 million

C) $1,735 million $490 million

A) Interest received by Elegant, Inc. on a bond Elegant, Inc. purchased from an outside investor. B) Dividends received by Elegant, Inc. from an investment in another firm.C) Elegant's payment to purchase equipment to be used in its business.

Accounts payable turnover 5.0Cost of goods sold $30 millionAverage inventory $3 millionAverage receivables $8 millionTotal liabilities $35 million

Page 95: Financial Reporting and Analysis_The Income Statement, Balance Sheet, and Cash Flow Statement

Assuming 365 days in the calendar year, calculate Eagle's sales for the year.

Question 113 - #97687

Which of the following is a measure of a firm's liquidity?

Question 114 - #98066

Do gains and losses, as well as expenses appear on the income statement?

Question 115 - #97787

Based on the following data, how many shares of common stock should be used to calculate diluted earnings per share?

Net income of $1,500,000, tax retention rate of 60% 1,000,000 shares of common are outstanding at the beginning of the year. 10,000, 6% convertible bonds with each bond convertible into 20 shares of common stock were issued at par ($100) on June 30th of this year. The firm has 100,000 warrants outstanding all year with an exercise price of $25 per share. The average stock price for the period is $20, and the ending stock price is $30.

Question 116 - #95134

Which of the following best describes a ratio that measures a firm’s ability to acquire long-term assets with cash flows from operations, and a performance ratio, respectively?

Interest expense $2 millionCash conversion cycle 13.5 days

A) $58.4 million.B) $52.3 million.C) $57.8 million.

A) Equity Turnover.B) Cash Ratio.C) Net Profit Margin.

A) Both appear on the income statement.B) Only expenses appear on the income statement.C) Only gains and losses appear on the income statement.

A) 1,266,667.B) 1,100,000.C) 1,000,000.

Acquire assets with CFO Performance ratio

A) Investing and financing ratio Cash-to-income ratio

B) Reinvestment ratio Debt payment ratio

C) Reinvestment ratio Cash-to-income ratio

Page 96: Financial Reporting and Analysis_The Income Statement, Balance Sheet, and Cash Flow Statement

© 2010 Kaplan Schweser

Page 97: Financial Reporting and Analysis_The Income Statement, Balance Sheet, and Cash Flow Statement

Kaplan Schweser Printable Exams - Test Management Exam 20

Test ID#: 7789112

Question 1 - #97985

Your answer: A was incorrect. The correct answer was C) Percentage-of-completion method.

The percentage-of-completion method is appropriate in this case because payment is assured when dealing with the U.S. government, and cost and price estimates are assumed reliable due to the ongoing and routine nature of the contract.

This question tested from Session 8, Reading 32, LOS b

Question 2 - #98058

Your answer: A was incorrect. The correct answer was B) unusual in nature and infrequent.

Extraordinary items are unusual and infrequent items reported below the line net of taxes. “Below the line” means after net income from continuing operations but before net income.

- Discontinued operations are reported below the line net of taxes.

- Unusual or infrequent items are unusual or infrequent, but not both. They appear (a separate line item) as a component of net income from continuing operations that must be removed if not deemed to be a component of persistent income. They are reported above the line before taxes.

- Changes in accounting principle are reported below the line net of taxes.

- Accounting errors go directly to retained earnings.

This question tested from Session 8, Reading 32, LOS f

Question 3 - #97966

Your answer: A was correct!

The January 1 balance is adjusted retroactively for the reverse stock split and 320,000 / 2 = 160,000 shares are treated as outstanding from January 1. Issuance of stock is included from the date of issuance. The weighted average shares are computed by multiplying the share amounts by the number of months the shares were outstanding, then adding these amounts and dividing the sum by 12.

Oregon’s weighted average shares = 2,370,000 / 12 = 197,500.

This question tested from Session 8, Reading 32, LOS g

Question 4 - #98080

January 1: initial shares 160,000 × 12 = 1,920,000July 1: Smith acquisition 60,000 × 6 = 360,000October 1: cash issuance 30,000 × 3 = 90,000Total: 2,370,000

Page 98: Financial Reporting and Analysis_The Income Statement, Balance Sheet, and Cash Flow Statement

Your answer: A was incorrect. The correct answer was C) presents the net equity of each asset by subtracting its related liability.

A classified balance sheet groups assets and liabilities by subcategories. It distinguishes between current and noncurrent assets and current and noncurrent liabilities. The assets and related liabilities are reported separately, they are not netted.

This question tested from Session 8, Reading 33, LOS b

Question 5 - #97831

Your answer: A was incorrect. The correct answer was B) $5.45.

Baltimore’s basic earnings per share (EPS) (net income / weighted average shares outstanding) for 2004 was $4,200,000 / 750,000 = $5.60.

To calculate diluted EPS, we use the treasury stock method to account for the warrants:

Number of common shares created if options are exercised = 10,000 × 10 = 100,000 Cash inflow if warrants are exercised = $40 × 100,000 = $4,000,000 Shares purchased with these funds = $4,000,000 / 50 = 80,000 Net increase in shares outstanding = 100,000 – 80,000 = 20,000

Diluted EPS = $4,200,000 / (750,000 + 20,000) = $5.45.

This question tested from Session 8, Reading 32, LOS h

Question 6 - #97752

Your answer: A was incorrect. The correct answer was B) company having issued warrants, convertible securities, or options.

A complex structure contains potentially dilutive securities such as options warrants or convertible securities. Where as simple capital structures contain no potentially dilutive securities and contains only common stock and non-convertible securities.

This question tested from Session 8, Reading 32, LOS g

Question 7 - #98004

Your answer: A was correct!

The cost recovery method is used when the costs to provide goods or services are not known. Under this method, sales are recognized when cash is received, but no gross profit is recognized until all of the cost of goods sold is collected.

This question tested from Session 8, Reading 32, LOS b

Question 8 - #97822

Your answer: A was correct!

Dilutive securities such as convertibles and options are found in a complex capital structure and always decrease EPS. Convertibles and options may also be antidilutive, which will increase EPS hence the name antidilutive. The

Page 99: Financial Reporting and Analysis_The Income Statement, Balance Sheet, and Cash Flow Statement

only way to know if a security is dilutive or antidilutive is to compare the basic EPS to diluted EPS. If the diluted EPS is higher than the basic EPS then the security is antidilutive and should not be included when determining diluted EPS.

This question tested from Session 8, Reading 32, LOS h

Question 9 - #94899

Your answer: A was incorrect. The correct answer was B) not be reported on the statement of cash flows because it is a non-cash event.

No cash is involved in a stock split--shares are exchanged for shares.

This question tested from Session 8, Reading 34, LOS b

Question 10 - #97797

Your answer: A was correct!

Antidilutive securities increase EPS if exercised or converted to common stock.

This question tested from Session 8, Reading 32, LOS h

Question 11 - #93535

Your answer: A was incorrect. The correct answer was B) Selling stock of the company.

Selling stock of the company would be a financing cash flow.

This question tested from Session 8, Reading 34, LOS a

Question 12 - #97896

Your answer: A was incorrect. The correct answer was B) 4 days.

Cash conversion cycle = average receivables collection period + average inventory processing period – payables payment period

= 95 + 183 – 274 = 4 days

This question tested from Session 8, Reading 35, LOS d

Question 13 - #97698

Your answer: A was correct!

ROE = profit margin × asset turnover × A/E = 0.08 × 1.2 × 1.1 = 0.1056 RR = (1 - 0.45) = 0.55 g = ROE × RR = 0.1056 × 0.55 = 0.0581

This question tested from Session 8, Reading 35, LOS g

Page 100: Financial Reporting and Analysis_The Income Statement, Balance Sheet, and Cash Flow Statement

Question 14 - #97690

Your answer: A was incorrect. The correct answer was B) 7.35%.

ROE = tax burden × interest burden × EBIT margin × asset turnover × financial leverage

tax burden = net income/EBT EBT = EBIT - I = 2,000,000 - 900,000 = 1,100,000 net income = (EBT)(1-t) = (1,100,000)(1 - 0.35) = 715,000 tax burden = 715,000/1,100,000 = 0.65

interest burden = EBT/EBIT = 1,100,000/2,000,000 = 0.55

EBIT margin = EBIT/revenue = 2,000,000/16,000,000 = 0.125

asset turnover = revenue/total assets = 16,000,000/12,300,000 = 1.301

financial leverage = total assets/total equity = 12,300,000/7,000,000 = 1.757

ROE = 0.65 × 0.55 × 0.125 × 1.301 × 1.757 = 0.1021

Alternatively, ROE = [(EBIT - I)(1-t)]/equity = [(2,000,000 - 900,000)(1 - 0.35)]/7,000,000 = 0.1021

Sustainable growth = ROE (1 – dividend payout rate) = 0.1021 × 0.72 = 7.35%.

This question tested from Session 8, Reading 35, LOS g

Question 15 - #97300

Your answer: A was incorrect. The correct answer was B)

Cash paid for insurance = insurance expense + change in prepaid insurance, so insurance expense = cash paid for insurance – change in prepaid insurance. Insurance expense for 2007 is equal to $925,000 [($750,000 cash paid for insurance – (–$175,000)]. Interest expense for 2007 is equal to $950,000 ($900,000 cash interest paid + $50,000 increase in interest payable).

This question tested from Session 8, Reading 34, LOS e

Question 16 - #97975

Your answer: A was correct!

Number of average shares:

1/1 5,500 shares issued (includes 10% stock dividend on 6/1) × 12 = 66,000 7/1 1,000 shares repurchased × 6 months = 6,000 66,000 − 6,000 = 60,000

60,000 shares / 12 months = 5,000 average shares

Preferred dividends = ($10)($1,000) = $10,000

Basic EPS = [$15,000(NI) – $10,000(preferred dividends)] / 5,000 shares = $5,000 / 5,000 shares = $1/share

This question tested from Session 8, Reading 32, LOS g

$925,000 $950,000

Page 101: Financial Reporting and Analysis_The Income Statement, Balance Sheet, and Cash Flow Statement

Question 17 - #95772

Your answer: A was incorrect. The correct answer was C) Straight Line.

$90 − $10 million = $80 million; $80 million / 8 = $10 million depreciation per year under Straight Line depreciation.

This question tested from Session 8, Reading 32, LOS d

Question 18 - #96532

Your answer: A was incorrect. The correct answer was B) 0.267.

There are many different ways to illustrate ROE one of which is:

ROE = (net profit margin)(asset turnover)(equity multiplier)

0.18 = (0.15)(4.5)(equity multiplier)

0.18 ÷ [(0.15)(4.5)] = equity multiplier

0.18 ÷ 0.675 = equity multiplier

0.18 ÷ 0.675 = 0.267

This question tested from Session 8, Reading 35, LOS f

Question 19 - #97893

Your answer: A was incorrect. The correct answer was B) $0.782.

Nichols basic EPS (net income / weighted average common shares outstanding) was:

$978,000 / 1,250,000 = $0.782.

Because the exercise price of the warrants is higher than the average share price, the warrants are antidilutive and are excluded from diluted EPS. Because there were no other potentially dilutive securities, Nichols' diluted EPS in 20X6 is the same as basic EPS.

This question tested from Session 8, Reading 32, LOS h

Question 20 - #97996

Your answer: A was incorrect. The correct answer was C) 37,000.

The end-of-period weighted average number of common shares outstanding is the number of shares outstanding during the year weighted by the portion of the year they were outstanding. Dividends and splits are applied to all shares issued or repurchased and all original or adjusted shares outstanding prior to the split or dividend. Step 1) Apply the 04/01/06 dividend to the beginning of year shares:

Adjusted shares = 1.05 × 20,000 = 21,000

Step 2) Apply the 10/01/06 split to the adjusted beginning-of-year shares and the repurchase.

Page 102: Financial Reporting and Analysis_The Income Statement, Balance Sheet, and Cash Flow Statement

Adjusted beginning-of-year shares = 42,000 (= 2 × 21,000) Adjusted repurchase = 10,000 (= 2 × 5,000)

Step 3) Compute the weighted average number of shares.

42,000(12/12) - 10,000(6/12) = 37,000 shares

This question tested from Session 8, Reading 32, LOS g

Question 21 - #97827

Your answer: A was correct!

Diluted EPS = adjusted earnings after conversion (EAC) / weighted average plus potential common shares outstanding.

Step 1: Calculate Adjusted EAC

preferred dividends = convertible preferred dividends = (0.08)(90)(500) = 3,600

convertible debt interest = (60,000)(0.06)(1 – 0.40) = 2,160

adjusted EAC = (30,000 – 3,600 + 3,600 + 2,160) = $32,160

Step 2: Calculate Weighted average plus potential common shares outstanding.

Step 3: Calculate Diluted EPS

Diluted EPS = 32,160 / 13,600 = $2.36.

This question tested from Session 8, Reading 32, LOS h

Question 22 - #98014

Your answer: A was incorrect. The correct answer was C)

adjusted EAC: net income - preferred dividends+ dividends on convertible preferred stock+ after-tax interest on convertible debt= adjusted earnings available for common shares

weighted average common shares = 5,000shares from conversion of convertible preferred stock = (500 × 4) = 2,000shares from conversion of convertible bonds = (60 × 110) = 6,600weighted ave. plus potential common shares outst. = 13,600

$1.00 $0.80

Page 103: Financial Reporting and Analysis_The Income Statement, Balance Sheet, and Cash Flow Statement

This question tested from Session 8, Reading 32, LOS h

Question 23 - #93588

Your answer: A was incorrect. The correct answer was B) Dividends paid to stockholders.

Dividends paid to stockholders are considered cash flow relating to financing activity. However, U.S. GAAP requires interest paid to bondholders to be considered an operating activity.

This question tested from Session 8, Reading 34, LOS a

Question 24 - #97691

Your answer: A was incorrect. The correct answer was C) 9%.

Return on equity (ROE) = net profit margin × asset turnover × leverage = (0.15)(0.67)(1.364) = 0.137.

The sustainable growth = (1 – dividend rate)(ROE) = (0.65)(0.137) = 8.9%.

This question tested from Session 8, Reading 35, LOS g

Question 25 - #119452

Your answer: A was correct!

Comparing a company’s ratios with those of its competitors is known as cross-sectional analysis.

This question tested from Session 8, Reading 35, LOS c

Question 26 - #98033

Your answer: A was incorrect. The correct answer was C)

When a firm recognizes revenue before cash is collected, equity increases (retained earnings) and assets increase (accounts receivable). Liabilities would not be affected.

This question tested from Session 8, Reading 33, LOS c

Question 27 - #97118

Your answer: A was incorrect. The correct answer was B) Return on equity has improved.

Leverage increased as measured by the debt-to-equity ratio from 2.25 in 2005 to 3.68 in 2007. Gross profit margin declined from 20.0% in 2005 to 18.5% in 2007. Return on equity has improved since 2005. One measure of ROE is ROA × financial leverage. Financial leverage (assets / equity) can be derived by adding 1 to the debt-to-equity ratio. In 2005, ROE was 23.4% [7.2% ROA × (1 + 2.25 debt-to-equity)]. In 2007, ROE was 27.6% [5.9% ROA × (1 + 3.68 debt-to-equity)].

This question tested from Session 8, Reading 35, LOS e

Question 28 - #97874

Increase No effect

Page 104: Financial Reporting and Analysis_The Income Statement, Balance Sheet, and Cash Flow Statement

Your answer: A was incorrect. The correct answer was B)

Average receivables collection period = (365 / 10) = 36.5 or 37

Average payables payment period = (365 / 12) = 30.4 or 30

Average inventory processing period = (365 / 8) = 45.6 or 46

This question tested from Session 8, Reading 35, LOS d

Question 29 - #97059

Your answer: A was incorrect. The correct answer was C) net income/sales × sales/assets × assets/equity.

Profit margin × asset turnover × financial leverage. Although net income/assets × sales/equity × assets/sales also yields ROE, it is not the DuPont equation.

This question tested from Session 8, Reading 35, LOS f

Question 30 - #95657

Your answer: A was correct!

Changes in asset lives and salvage value are changes in accounting estimates and are not considered changes in accounting principle. No specific disclosures are required.

This question tested from Session 8, Reading 32, LOS f

Question 31 - #98064

Your answer: A was incorrect. The correct answer was B) unusual and infrequent.

Extraordinary items are unusual and infrequent, reported below the line separate from income from continuing operations on the income statement, and would include such items as: foreign government confiscation, earthquake damages, losses from volcanic eruptions, etc.

This question tested from Session 8, Reading 32, LOS f

Question 32 - #97816

Your answer: A was incorrect. The correct answer was B) 1.29.

Inventory turnover = 1,100(COGS) / 850(inventory) = 1.29

This question tested from Session 8, Reading 35, LOS d

Question 33 - #98089

Your answer: A was incorrect. The correct answer was C) realizable and earned.

Under the accrual concept, revenue is recognized when the earnings process is completed (earned) and ultimate realization (cash receipt) is assured.

37 30 46

Page 105: Financial Reporting and Analysis_The Income Statement, Balance Sheet, and Cash Flow Statement

This question tested from Session 8, Reading 32, LOS b

Question 34 - #98008

Your answer: A was incorrect. The correct answer was C) Items sold are a mix of the cost of the purchases.

With FIFO the cost of the items sold are the first purchased, with LIFO the cost of the items last purchased represent the first to be sold, and with the weighted average cost method the cost of the items sold are a mix of purchases.

This question tested from Session 8, Reading 32, LOS d

Question 35 - #96458

Your answer: A was incorrect. The correct answer was C) Financing.

Dividends paid to stockholders are considered cash outlays from financing according to U.S. GAAP.

This question tested from Session 8, Reading 34, LOS a

Question 36 - #98045

Part 1) Your answer: A was incorrect. The correct answer was C) $35,045.

(0.0625)(100)(2,315) = 14,469

(0.08)(100)(2,572) = 20,576

14,469 + 20,576 = 35,045

This question tested from Session 8, Reading 32, LOS h

Part 2) Your answer: A was incorrect. The correct answer was C) $3,675.

(0.06125)(1,000)(100)

(6,125)(1 − 0.4) = 3,675

This question tested from Session 8, Reading 32, LOS h

Part 3) Your answer: A was incorrect. The correct answer was B) 2,689.

9,986 × $38 = $379,468

$379,468 / $52 = 7,297 common shares

9,986 − 7,297 = 2,689 new common shares

This question tested from Session 8, Reading 32, LOS h

Part 4) Your answer: A was incorrect. The correct answer was B)

Page 106: Financial Reporting and Analysis_The Income Statement, Balance Sheet, and Cash Flow Statement

Basic EPS = Net income − preferred dividends / Wt Average shares of common = ($200,000 − $35,045) / 40,045 = 164,955/40,405 = $4.12

Diluted EPS:

(100 bonds)(33 common shares/bond) = 3,300 common shares

(2,315 preferred shares)(3.3) = 7,640

(2,572 preferred shares)(5) = 12,860

7,640 + 12,860 = 20,500 common shares from preferred

[($200,000 − $35,045) + $35,045 + $3,675] / (40,045 + 3,300 + 20,500 + 2,689)

= $203,675 / 66,534 shares = $3.06

This question tested from Session 8, Reading 32, LOS h

Question 37 - #97517

Your answer: A was correct!

Using the indirect method, net income is adjusted by adding back depreciation (a non-cash expense) and changes in working capital: the increase in wages payable and the increase in income taxes payable are sources of cash, and the decrease in interest payable is a use of cash. Dividends paid are financing cash flows under U.S. GAAP.

CFO = $3,000,000 + $2,500,000 + $100,000 + $500,000 - $200,000 = $5,900,000.

This question tested from Session 8, Reading 34, LOS e

Question 38 - #97762

Your answer: A was incorrect. The correct answer was C) 250,000.

The treasury stock method would allow the 1 million additional shares to be partially offset by the number of shares that could be repurchased with the amount of money received for those shares. In this case, the 1 million shares issued would be offset by (1,000,000 × $42 / $56) or 750,000 shares.

This question tested from Session 8, Reading 32, LOS h

Question 39 - #97420

Your answer: A was incorrect. The correct answer was B) Net income.

Property, Plant, & Equipment and payment of dividends are components of the statement of cash flows under both the direct and indirect methods. Net income is the first figure under the indirect method, but it is not a part of the statement of cash flows under the direct method. The correct response is net income.

This question tested from Session 8, Reading 34, LOS e

Question 40 - #97670

$4.12 $3.06

Page 107: Financial Reporting and Analysis_The Income Statement, Balance Sheet, and Cash Flow Statement

Your answer: A was incorrect. The correct answer was B) 30%.

Operating profit margin = (EBIT / net sales) = ($150,000 / $500,000) = 30%

This question tested from Session 8, Reading 35, LOS d

Question 41 - #97992

Your answer: A was correct!

Basic EPS = (net income − preferred dividends) / number of common shares = (200,000 − 35,000) / 50,000 = $3.30 per share

The preferred shares are converted into 20,000 common shares, the firm does not pay preferred dividends. Diluted EPS = 200,000 / (50,000 + 20,000) = $2.86 per share. The warrants are out of the money at a stock price of $20.

This question tested from Session 8, Reading 32, LOS h

Question 42 - #97305

Your answer: A was incorrect. The correct answer was B) assets.

Common size balance sheets express all balance sheet items as a percentage of assets. Note that common size income statements express all income statement items as a percentage of sales.

This question tested from Session 8, Reading 33, LOS h

Question 43 - #97409

Your answer: A was incorrect. The correct answer was C) Both statements are correct.

A cash flow statement can be presented in common-size format by expressing each line item as a percentage of total revenue or by expressing each inflow of cash as a percentage of total cash inflows and each outflow as a percentage of total cash outflows. Expressing each line item of the cash flow statement as a percentage of revenue is useful in forecasting future cash flows since revenue usually drives the forecast.

This question tested from Session 8, Reading 34, LOS g

Question 44 - #97895

Your answer: A was incorrect. The correct answer was C)

The quick ratio is a liquidity ratio. Liquidity ratios are used to measure a firm’s ability to meet its short-term obligations. The debt-to-capital ratio is a solvency ratio. Solvency ratios are used to measure a firm’s ability to meet its longer-term obligations.

This question tested from Session 8, Reading 35, LOS d

Question 45 - #93954

Your answer: A was incorrect. The correct answer was B) $1,406.

Yes No

Page 108: Financial Reporting and Analysis_The Income Statement, Balance Sheet, and Cash Flow Statement

double-declining balance depreciation rate = 2 × 1/8 = ¼ or 25%

first year deprecation will be $7,500 × 0.25 = $1,875

second year deprecation will be ($7,500 − $1,875) × 0.25 = $1,406

This question tested from Session 8, Reading 32, LOS d

Question 46 - #98043

Your answer: A was correct!

At the beginning of 20X7, the prepaid insurance account (asset) will have a balance of $45,000. Insurance expense will be recognized at a rate of $15,000 per year. At the end of 20X8, one year’s insurance remains; thus, the balance of the prepaid insurance account will equal $15,000 ($45,000 beginning balance – $15,000 insurance expense for 20X7 – $15,000 insurance expense for 20X8).

This question tested from Session 8, Reading 33, LOS c

Question 47 - #97869

Your answer: A was incorrect. The correct answer was C) average market price of Young common stock increased in 20X5.

Average stock price is not a factor in determining whether convertible bonds are dilutive or antidilutive.

If Young redeemed the bonds, they would have no potentially dilutive securities outstanding in 20X5 and diluted EPS, if the company reported it, would equal basic EPS. Basic and diluted EPS would also be equal in 20X5 if the bonds were antidilutive in that year.

This question tested from Session 8, Reading 32, LOS h

Question 48 - #97414

Your answer: A was incorrect. The correct answer was B) $210.

Cash flow from operations (CFO) calculated using the indirect method is: net income (100) + depreciation (50) – increase in accounts receivable (10) + decrease in inventory (20) + increase in accounts payable (50) = $210.

This question tested from Session 8, Reading 34, LOS e

Question 49 - #98052

Your answer: A was incorrect. The correct answer was B)

Recognition of income depends on cash collected under both methods.

This question tested from Session 8, Reading 32, LOS b

Question 50 - #97416

Your answer: A was incorrect. The correct answer was B) +$105.

Yes Yes

Page 109: Financial Reporting and Analysis_The Income Statement, Balance Sheet, and Cash Flow Statement

Using the indirect method, the increase in accounts payable is a source of cash from operations (+25), depreciation expense is a non-cash expense added back in computing cash from operations (+100), and increase in inventory is a use of cash from operations (-20) = 25 + 100 - 20 = 105. The sale of stock and the dividends paid are financing cash flows that are not included in net income, so they do not require adjustment when calculating CFO.

This question tested from Session 8, Reading 34, LOS e

Question 51 - #97669

Your answer: A was incorrect. The correct answer was B)

Gross profit margin = gross profit / net sales = 145 / 200 = 0.725

Operating profit margin = EBIT / net sales = 115 / 200 = 0.575

This question tested from Session 8, Reading 35, LOS d

Question 52 - #97399

Your answer: A was incorrect. The correct answer was E) +$20.

There are two ways to approach this problem. The easy way is to just take the difference in cash between the two years: 80 – 60 = $20

The other way is to create a statement of cash flows:

CFO = Net Income (125) – (increase in Accounts Receivable) (15) – (increase in Inventory) (25) + (Depreciation) (6) – (decrease in Accounts Payable) (25) + (increase in Wages Payable) (5) = $71.

CFI = Fixed assets increased by $40 representing a use of cash = -$40.

CFF = (issuance of Bonds) (15) + (issuance/sale of Common Stock) (30) – Dividends (56) = -$11

Net increase in cash = 71 – 40 –11 = $20.

This question tested from Session 8, Reading 34, LOS e

Question 53 - #97417

Part 1) Your answer: A was correct!

Under the direct method, cash collections are found by subtracting the change in accounts receivable from total sales. This is done because an increase in receivables indicates sales that were made on credit. In the case of Soft Corporation, the calculation is as follows:

198.0 – (24.0 – 17.0) = 191.0

Therefore, the correct answer is 191.0. Financing activities and expenses are not included in cash collections from operating activities.

This question tested from Session 8, Reading 34, LOS e

0.725 0.575

Page 110: Financial Reporting and Analysis_The Income Statement, Balance Sheet, and Cash Flow Statement

Part 2) Your answer: A was incorrect. The correct answer was F) -130.0.

Under the direct method cash inputs = -COGS + decrease in inventory + increase in accounts payable. The calculation for Soft Corporation’s projected cash inputs is as follows:

-90.0 − (150.0 − 100.0) + (101.0 − 91.0) = -130.0

This question tested from Session 8, Reading 34, LOS e

Question 54 - #97748

Your answer: A was correct!

To compute Hampshire’s basic EPS ((net income – preferred dividends) / weighted average common shares outstanding), the weighted average common shares must be computed. 100,000 shares were outstanding from January 1, and 30,000 shares were issued on September 1, so the weighted average is 100,000 + (30,000 × 4 / 12) = 110,000. Basic EPS is ($2,800,000 – (10,000 × $1,000 × 0.06)) / 110,000 = $20.00.

If the warrants were exercised, cash inflow would be 10,000 × $150 × 10 = $15,000,000 for 10 × 10,000 = 100,000 shares. Using the treasury stock method, the number of Hampshire shares that can be purchased with the cash inflow (cash inflow / average share price) is $15,000,000 / $250 = 60,000. The number of shares that would be created is 100,000 – 60,000 = 40,000. Diluted EPS is $2,200,000 / (110,000 + 40,000) = $14.67.

This question tested from Session 8, Reading 32, LOS h

Question 55 - #100936

Your answer: A was correct!

Revenue recognition policies are disclosed in the footnotes to the financial statements.

This question tested from Session 8, Reading 32, LOS b

Question 56 - #94897

Your answer: A was correct!

The purchase of plant and equipment with financing provided by the seller is a non-cash transaction. Non-cash transactions are disclosed separately in a note or supplementary schedule to the cash flow statement.

This question tested from Session 8, Reading 34, LOS b

Question 57 - #98067

Your answer: A was incorrect. The correct answer was C) Only one should be recognized.

Interest earned on the CD is recognized as interest income. The gain on the sale of treasury stock is not reported on the income statement but is relected on the statement of changes in stockholders’ equity and on the balance sheet. The sale proceeds simply increase equity and increase cash.

This question tested from Session 8, Reading 32, LOS a

Page 111: Financial Reporting and Analysis_The Income Statement, Balance Sheet, and Cash Flow Statement

Question 58 - #97861

Your answer: A was incorrect. The correct answer was B) $3.94.

Kendall’s basic EPS is $830,000 / 200,000 = $4.15. To compute diluted EPS, bond interest paid net of taxes is added to net income, and the number of shares that would be issued in the conversion is added to the denominator. Kendall’s diluted EPS = [$830,000 + (1,000 × $1,000 × 0.06) × (1 – 0.4)] / (200,000 + 20,000) = $3.94. Since diluted EPS is less than basic EPS, we know that the bonds are dilutive and should be considered in calculating diluted EPS.

This question tested from Session 8, Reading 32, LOS h

Question 59 - #97321

Your answer: A was incorrect. The correct answer was B) When recognizing a gain on the sale of fixed assets, the amount is a deduction to operating cash flows.

When recognizing a gain on the sale of fixed assets, the amount is a deduction to operating cash flows. This is because the gain would be double counted in the investing section and in net income. Therefore, the gain must be removed from net income. The direct method of cash flow calculation converts the income statement items to their cash equivalents, not the indirect method. Also, depreciation is added to net income in order to calculate CFO using the indirect method.

This question tested from Session 8, Reading 34, LOS e

Question 60 - #98060

Your answer: A was incorrect. The correct answer was C) $0.

The completed contract method is used when a reliable estimate of the total costs cannot be determined until the contract is finished. Because of the significant uncertainty surrounding the ground water costs, the completed contract method should be used in this transaction, and no revenue should be recognized in 2004 or any later year until the contract is completed or the cost uncertainty is resolved.

This question tested from Session 8, Reading 32, LOS b

Question 61 - #97902

Your answer: A was incorrect. The correct answer was B)

Revenue divided by average working capital, also known as the working capital turnover ratio, is an activity ratio. Average total assets divided by average total equity, also known as the financial leverage ratio, is a solvency ratio.

This question tested from Session 8, Reading 35, LOS d

Question 62 - #97387

Your answer: A was incorrect. The correct answer was F) -$100,000.

The only item involving cash flow from financing (CFF) was the payment of a cash dividend by Juniper. The sale of equipment affects cash flow from investing (CFI), the purchase of land has no effect on cash, and the preferred dividends received are cash flow from operations.

Activity ratio Solvency ratio

Page 112: Financial Reporting and Analysis_The Income Statement, Balance Sheet, and Cash Flow Statement

This question tested from Session 8, Reading 34, LOS e

Question 63 - #97879

Your answer: A was incorrect. The correct answer was C) 735,000.

The January 1 balance is adjusted retroactively for the stock dividend and (540,000 × 1.5) = 810,000 shares are treated as outstanding from January 1. The weighted average number of shares is computed by multiplying the shares by the number of months held, as follows:

Weighted average shares was (8,820,000 / 12) = 735,000 shares.

This question tested from Session 8, Reading 32, LOS g

Question 64 - #97760

Your answer: A was incorrect. The correct answer was C) $5.32.

Feder’s basic earnings per share ((net income – preferred dividends) / weighted average shares outstanding) was (($7,650,000 – ($1,000 × 10,000 × 0.06)) / 1,100,000 =) $6.41.

If the convertible preferred stock was converted to common stock at January 1, (10,000 × 20 =) 200,000 additional common shares would have been issued, dividends on the preferred stock would not have been paid, and Diluted EPS would have been ($7,650,000 / (1,100,000 + 200,000) = $5.88. Because $5.88 is less than basic EPS of $6.41, the preferred shares are dilutive.

Using the treasury stock method, if the options were exercised cash inflow would be (70,000 × 10 × $50 =) $35,000,000. The number of Feder shares that can be purchased with the inflow (cash inflow divided by the average share price) is ($35,000,000 / $62 =) 564,516.

The number of shares that would have been created is (700,000 – 564,516 =) 135,484. Diluted EPS was ($7,650,000 / (1,100,000 + 135,484) =) $6.19. Because this is less than the EPS of $6.41, the options are dilutive.

Combining the calculations, Diluted EPS was (($7,650,000) / (1,100,000 + 200,000 + 135,484) = $5.32.

This question tested from Session 8, Reading 32, LOS h

Question 65 - #97858

Your answer: A was incorrect. The correct answer was C) $2.01.

Quad’s basic EPS (net income / weighted average common shares outstanding) was $892,000 / 400,000 = $2.23.

Diluted EPS is calculated under the assumption that the convertible bonds are converted into common stock, the bond interest net of tax is restored to net income, and the additional common shares are added to the denominator of the equation. Quad’s diluted EPS was [$892,000 + (2,000 × $1,000 × 0.06)(1 − 0.40)] / [400,000 + (2,000 × 40)] = $2.01. Since diluted EPS is less than basic EPS, we know that the bonds are dilutive and should be considered in calculating diluted EPS.

This question tested from Session 8, Reading 32, LOS h

January 1 Initial shares (810,000 × 12) = 9,720,000July 1 Reacquired shares (-180,000 × 6) = 1,080,000October 1 Reissued shares (60,000 × 3) = 180,000

8,820,000

Page 113: Financial Reporting and Analysis_The Income Statement, Balance Sheet, and Cash Flow Statement

Question 66 - #98065

Your answer: A was incorrect. The correct answer was B)

Gross profit is equal to sales minus cost of goods sold. Cost of goods sold includes the direct costs of producing a product or service such as raw materials, direct labor, and overhead (fixed costs). Thus, an increase in raw materials costs will result in higher cost of goods sold and lower gross profit. Marketing expenses are considered operating expenses (SG&A), not in cost of goods sold.

This question tested from Session 8, Reading 32, LOS a

Question 67 - #97278

Your answer: A was incorrect. The correct answer was B)

As of December 31, 2007, Carpenter’s retained earnings is $11,900,000 [$10,500,000 beginning balance + $1,700,000 net income – $300,000 dividends declared]. Accumulated other comprehensive income is $515,000 [$450,000 beginning balance – $25,000 unrealized loss from available for sale securities ($225,000 fair value – $250,000 cost) + $90,000 unrealized translation gain]. There is no impact on retained earnings or accumulated other comprehensive income from unrealized gains and losses on held-to-maturity securities since the securities are not reported at fair value on the balance sheet. The purchase of treasury stock does not affect comprehensive income because it is a transaction with shareholders.

This question tested from Session 8, Reading 33, LOS g

Question 68 - #97412

Your answer: A was correct!

The indirect method must add back depreciation expense because the starting point is net income. Since the direct method does not begin with net income it does not need to consider non-cash expenses such as depreciation.

This question tested from Session 8, Reading 34, LOS e

Question 69 - #97905

Your answer: A was incorrect. The correct answer was B) Simple.

A complex capital structure contains potentially dilutive securities such as options, warrants, or convertible securities. There is no basic capital structure but there are basic earnings per share which does NOT consider the effects of any dilutive securities in the computation of EPS.

This question tested from Session 8, Reading 32, LOS g

Question 70 - #97783

Your answer: A was incorrect. The correct answer was B) 0.671.

The original debt / equity ratio = 35 / 70 = 0.5. Now adjust the numerator but not the denominator. Why? You have commitments (liabilities) but no new equity because (non-current) liabilities and assets are increased by the same

Yes No

$11,900,000 $515,000

Page 114: Financial Reporting and Analysis_The Income Statement, Balance Sheet, and Cash Flow Statement

amount. D/E = (35 + 12) / 70 = 0.671.

This question tested from Session 8, Reading 35, LOS d

Question 71 - #94804

Your answer: A was incorrect. The correct answer was B) Only one should be reported as such.

Retiring bonds by issuing common stock to the bondholders is a non-cash transaction and is disclosed separately in a note or supplementary schedule to the cash flow statement, rather than as a financing cash flow. The cash borrowed for the equipment purchase is a financing inflow and the cash cost of the equipment is reported as an investing cash flow in the cash flow statement. Had a bond been issued to the seller of the equipment, it would be treated as a non-cash transaction and reported only in the notes to the cash flow statement.

This question tested from Session 8, Reading 34, LOS b

Question 72 - #97826

Your answer: A was correct!

Unrealized foreign currency translation gains and losses are not reported in the income statement; thus, retained earnings are unaffected. However, unrealized foreign currency gains and losses are included in comprehensive income. Comprehensive income includes all changes in equity except those that result from transactions with shareholders. So, the translation gain increases stockholders’ equity by increasing comprehensive income.

This question tested from Session 8, Reading 32, LOS i

Question 73 - #97962

Your answer: A was incorrect. The correct answer was C) Supple Moves and Perfect Collection.

A complex capital structure is one that has potentially dilutive elements. Here, Supple Moves and Perfect Collection both meet this criteria. (The warrants for Supple Moves will be dilutive if the average stock prices were over $50.00.)

This question tested from Session 8, Reading 32, LOS g

Question 74 - #97994

Your answer: A was incorrect. The correct answer was C) $4.00.

EPS = earnings available to common shareholders divided by the weighted average number of common shares outstanding. With no preferred shareholders, all of net income is available to the common shareholders. The weighted average number of shares outstanding equals the original 2 million shares plus 4/12 of the additional 600,000 shares. The 4/12 weight is used because the new shares were only outstanding 4 months of the year. Thus, EPS = $8.8 million / [2 million + (4/12)(600,000)] = 8.8/2.2 = $4.00.

This question tested from Session 8, Reading 32, LOS g

Question 75 - #98001

Your answer: A was correct!

Page 115: Financial Reporting and Analysis_The Income Statement, Balance Sheet, and Cash Flow Statement

Changes in accounting principle require retrospective presentation. A change in the salvage value of an asset is a change in accounting estimate, which does not apply retrospectively.

This question tested from Session 8, Reading 32, LOS f

Question 76 - #97930

Your answer: A was incorrect. The correct answer was C)

The current ratio is equal to 2.4 [(4.8% cash + 14.9% accounts receivable + 49.4% inventory) / (15.0% accounts payable + 13.8% accrued liabilities)]. This ratio can be calculated from the common size balance sheet because the percentages are all on the same base amount (total).

Return on equity is equal to net income divided by average total equity. Since this ratio mixes an income statement item and a balance sheet item, it is necessary to convert the common-size inputs to dollars. Net income is $11,211,200 ($215,600,000 × 5.2%) and average equity is $41,772,000 [($95,300,000 × 48.0%) + $37,800,000] / 2. Thus, 2007 ROE is 26.8% ($11,211,200 net income / $41,772,000 average equity).

This question tested from Session 8, Reading 35, LOS d

Question 77 - #100934

Your answer: A was correct!

The principle of accrual accounting is that revenue is recognized when earned, and expenses are recognized when incurred.

This question tested from Session 8, Reading 32, LOS b

Question 78 - #97329

Your answer: A was incorrect. The correct answer was B) $4,600,000.

$3,000,000 + $1,500,000 − $200,000 + $300,000 = $4,600,000.

This question tested from Session 8, Reading 34, LOS e

Question 79 - #97959

Your answer: A was correct!

The new shares were only outstanding 4 months of the year. Thus, the weighted average number of shares outstanding is [1.4 + (4/12)(1.2)] million = 1.8 million shares. So basic EPS = $7.2 million / 1.8 million = $4.00.

This question tested from Session 8, Reading 32, LOS g

Question 80 - #97908

Your answer: A was incorrect. The correct answer was C) Calculation of ratios involves a large degree of subjectivity.

There is not a great deal of subjectivity involved in calculating ratios. The mechanical formulas for the calculations

2.4 26.8%

Page 116: Financial Reporting and Analysis_The Income Statement, Balance Sheet, and Cash Flow Statement

are fairly standard and objective for the activity, liquidity, solvency, and profitability ratios, for instance. On the other hand, determining the target or comparison value for a ratio is difficult as it requires some range of acceptable values and that introduces an element of subjectivity. Conclusions cannot be made from viewing one set of ratios as all ratios must be viewed relative to one another in order to make meaningful conclusions. It can be difficult to find comparable industry ratios, especially when analyzing companies that operate in multiple industries.

This question tested from Session 8, Reading 35, LOS b

Question 81 - #98006

Your answer: A was incorrect. The correct answer was C) $0.50.

The preferred shares are convertible into 100,000 × 20 = 2 million common shares. They are dilutive since:

This question tested from Session 8, Reading 32, LOS h

Question 82 - #98019

Your answer: A was incorrect. The correct answer was C)

Since Red Oak is a nonfinancial firm, the accrued interest is considered a nonoperating activity, related to how the firm is financed. Dividends paid to preferred shareholders do not affect net income.

This question tested from Session 8, Reading 32, LOS e

Question 83 - #97435

Your answer: A was correct!

This question tested from Session 8, Reading 34, LOS e

Basic EPS = $1,000,000 = $1.00 1,000,000

Diluted EPS = $1,500,000 = $0.50 which is less. 3,000,000

Yes No

Indirect Method

EAT +1,000Depreciation +500Change in Inv. + 100 a sourceChange in Accts. Rec. (300) a use CFO 1,300

Direct Method

Net Sales +3,500Change in Accts. Rec. (300) a useCOGS (1,500)Cash Taxes (500) Change in Inv. +100 a source CFO 1,300

Page 117: Financial Reporting and Analysis_The Income Statement, Balance Sheet, and Cash Flow Statement

Question 84 - #97502

Your answer: A was correct!

Current ratio = (current assets / current liabilities) = [cash + marketable securities + receivables + inventory] / current liabilities

Quick ratio = [cash + marketable securities + receivables] / current liabilities

This question tested from Session 8, Reading 35, LOS d

Question 85 - #97251

Your answer: A was incorrect. The correct answer was B)

Unrealized gains and losses from trading securities are recognized in the income statement while unrealized gains and losses from available-for-sale securities bypass the income statement and are reported as other comprehensive income, a component of stockholders’ equity. Cash dividends are recognized in the income statement for both trading and available-for-sale securities. Thus, Alpha will recognize only the $2,000 dividend if the shares are considered available-for-sale and will recognize $22,000 ($2,000 dividend + $20,000 unrealized gain) if the shares are considered trading securities.

This question tested from Session 8, Reading 33, LOS f

Question 86 - #98075

Your answer: A was incorrect. The correct answer was B) $10 million.

Reported profit (in millions) = ($30 / $75)($100 − 75) = $10.

This question tested from Session 8, Reading 32, LOS b

Question 87 - #97072

Your answer: A was correct!

Average receivables collection period = 365 / receivables turnover, which is 22.81 days for the industry (= 365 / 16). If Q-Tell’s receivables turnover is less than 16, its average days collection period must be greater that 22.81 days.

This question tested from Session 8, Reading 35, LOS e

Question 88 - #97952

Your answer: A was incorrect. The correct answer was B) 7,500,000 shares.

Stock splits and stock dividends are applied to all shares that existed at the beginning of the period and shares that were issued or repurchased during the period, but prior to the split or dividend. For SSP, the 5 million beginning-of-year shares outstanding are adjusted to 7.5 million shares (5.0 × 3/2) as a result of the 3:2 split.

This question tested from Session 8, Reading 32, LOS g

$2,000 $22,000

Page 118: Financial Reporting and Analysis_The Income Statement, Balance Sheet, and Cash Flow Statement

Question 89 - #97716

Your answer: A was incorrect. The correct answer was B)

Net income is equal to $490,000 ($600,000 gross profit – $100,000 operating expenses + $15,000 dividends received – $25,000 interest expense). Comprehensive income includes all transactions that affect stockholders’ equity except transactions with shareholders. Thus, comprehensive income is equal to $435,000 ($490,000 net income – $30,000 unrealized loss from foreign currency translation – $45,000 increase in minimum pension liability + $20,000 unrealized gain on available-for-sale securities). The treasury stock purchase is a transaction with shareholders and is not included in either comprehensive income or net income.

This question tested from Session 8, Reading 32, LOS i

Question 90 - #98084

Your answer: A was correct!

The completed contract method doesn't recognize revenue and expense until the contract is completed. The percentage-of-completion method would have recognized a portion of the $50,000 profit prior to completion.

This question tested from Session 8, Reading 32, LOS b

Question 91 - #98053

Your answer: A was incorrect. The correct answer was C) $10,000.

Under the percentage-of-completion method, one-half of the total revenue is recognized because one-half of the costs have been incurred ($40,000 / $80,000). Therefore, revenue will be equal to $50,000, expenses are $40,000, and net income will be $10,000.

This question tested from Session 8, Reading 32, LOS b

Question 92 - #97070

Your answer: A was incorrect. The correct answer was B) 25%.

Net Income / Equity = ROE 2 / 8 = 25%

This question tested from Session 8, Reading 35, LOS f

Question 93 - #97775

Your answer: A was correct!

Dilutive securities are securities that decrease EPS if they are exercised or converted to common stock. When the exercise price is less than the average market price, stock options are considered to be dilutive, Stock options, warrants, convertible debt, and convertible preferred stock are examples of potentially dilutive securities.

This question tested from Session 8, Reading 32, LOS h

$490,000 $435,000

Page 119: Financial Reporting and Analysis_The Income Statement, Balance Sheet, and Cash Flow Statement

Question 94 - #97982

Your answer: A was incorrect. The correct answer was C) $5.00 / share.

(210,000 − 110,000) / 20,000 = $5 share

This question tested from Session 8, Reading 32, LOS g

Question 95 - #97303

Your answer: A was incorrect. The correct answer was B) $23.

Using the indirect method, cash flow from operations is net income less increase in accounts receivable, plus increase in accounts payable, less increase in inventory, plus loss on sale of equipment, less gain on sale of real estate. 27 – 4 + 1 – 5 + 8 – 4 = $23 million.

This question tested from Session 8, Reading 34, LOS e

Question 96 - #97821

Your answer: A was incorrect. The correct answer was C)

Equipment is reported in the balance sheet at historical cost less accumulated depreciation. Inventory is reported in the balance sheet at the lower of cost or market.

This question tested from Session 8, Reading 33, LOS e

Question 97 - #98082

Your answer: A was incorrect. The correct answer was B)

The account format follows the traditional ledger account, assets on the left hand side and liabilities and equity on the right hand side. With a report format, the assets, liabilities, and equity are presented in a single column.

This question tested from Session 8, Reading 33, LOS b

Question 98 - #122498

Your answer: A was correct!

The reinvestment ratio is CFO divided by cash paid for long-term assets: €1,300 / €2,600 = 0.5. (Note that on this cash flow statement, CFI includes interest and dividends received and CFF includes interest paid, which is acceptable under IFRS.)

This question tested from Session 8, Reading 34, LOS h

Question 99 - #97413

Your answer: A was incorrect. The correct answer was C) $156.

Historical cost Lower of cost or market

No No

Page 120: Financial Reporting and Analysis_The Income Statement, Balance Sheet, and Cash Flow Statement

This question tested from Session 8, Reading 34, LOS e

Question 100 - #97772

Your answer: A was correct!

Preferred stock and bonds are only considered to be potentially dilutive if they are convertible. Options are always considered to be potentially dilutive.

This question tested from Session 8, Reading 32, LOS h

Question 101 - #98061

Your answer: A was correct!

Changes in accounting estimates are not treated the same as changes in principles. Changes in principles are treated retrospectively, whereas changes in accounting estimates are accounted for in the current and future periods. Both remaining statements are accurate.

This question tested from Session 8, Reading 32, LOS f

Question 102 - #97830

Your answer: A was incorrect. The correct answer was B) 3 million.

Cash ratio = (cash + marketable securities) / current liabilities

0.20 = ($10,000,000 + $2,000,000) / current liabilities

current liabilities = $12,000,000 / 0.2 = $60,000,000

Quick ratio = [cash + marketable securities + receivables] / $60,000,000

0.25 = [$10,000,000 + $2,000,000 + receivables] / $60,000,000

($60,000,000)(0.25) = $12,000,000 + receivables

$15,000,000 = $12,000,000 + receivables

$15,000,000 − $12,000,000 = receivables

$3,000,000 = receivables

This question tested from Session 8, Reading 35, LOS d

Net Income +$96Depreciation +30Gain on sale of asset -30Accts. Rec. +30Inventory +20Accts. Payable +20Wage/Pay -10

CFO +$156

Page 121: Financial Reporting and Analysis_The Income Statement, Balance Sheet, and Cash Flow Statement

Question 103 - #97931

Your answer: A was correct!

A simple capital structure is one that contains no securities that have the potential to dilute a firm’s earnings per share. For example, convertible bonds, convertible preferred stock, options, and warrants have the potential to dilute earnings per share upon conversion or exercise.

This question tested from Session 8, Reading 32, LOS g

Question 104 - #97404

Your answer: A was incorrect. The correct answer was C)

Decreasing accounts payable turnover saves cash by delaying payments to suppliers. The result is an operating source of cash, not a financing source. Decreasing accounts payable turnover is not a sustainable source of cash flow because suppliers will refuse to extend credit, at some point, if payment is slower and slower.

This question tested from Session 8, Reading 34, LOS g

Question 105 - #97846

Your answer: A was incorrect. The correct answer was C) $5.25.

Moulding’s basic EPS (net income / weighted average common shares outstanding) was $13,820,000 / 2,600,000 = $5.32.

Using the treasury stock method to compute diluted EPS, if the options were exercised, cash inflow would be 10,000 × 10 × $40 = $4,000,000. Based on the average share price of $58.00, the number of Moulding shares that can be purchased with the cash flow is $4,000,000 / $58 = 68,966. The number of shares that would have been created is 100,000 – 68,966 = 31,034. Diluted EPS was $13,820,000 / (2,600,000 + 31,034) = $5.25.

This question tested from Session 8, Reading 32, LOS h

Question 106 - #97320

Your answer: A was incorrect. The correct answer was E) $29.

Using the indirect method:

This question tested from Session 8, Reading 34, LOS e

Question 107 - #97911

No impact No

Add: Net Income $10 Add: Depreciation Expense 22 Less: Gain from Sale of Equip. (6) Less: Increase in Accounts Receivable (4) Add: Decrease in Inventory 2 Add: Increase in Accounts Payable 5 Cash flow from operations (CFO) 29

Page 122: Financial Reporting and Analysis_The Income Statement, Balance Sheet, and Cash Flow Statement

Your answer: A was incorrect. The correct answer was B) 26 days.

Cash conversion cycle = receivables collection period + inventory processing period – payables payment period.

Receivables collection period = (365 / 20) = 18 Inventory processing period = (365 / 16) = 23 Payables payment period = (365 / 24) = 15 Cash conversion cycle = 18 + 23 – 15 = 26

This question tested from Session 8, Reading 35, LOS d

Question 108 - #97335

Your answer: A was correct!

Common size statements normalize balance sheet and income statements and allow the analyst to make easier comparisons of different sized firms. A common size balance sheet expresses all balances as a percentage of total assets.

This question tested from Session 8, Reading 33, LOS h

Question 109 - #97944

Your answer: A was incorrect. The correct answer was B) $4.5 million.

Manhattan’s quick assets were equal to $9 million ($15 million current assets – $6 million inventory). Given a quick ratio of 2.0, quick assets were twice the current liabilities. Thus, the current liabilities must have been $4.5 million ($9 million quick assets / 2.0 quick ratio).

This question tested from Session 8, Reading 35, LOS d

Question 110 - #97935

Your answer: A was incorrect. The correct answer was B)

Current assets are equal to $1,665 ($275 cash and equivalents + $1,150 accounts receivable + $240 inventory). Working capital (current assets minus current liabilities) is equal to $490 ($1,665 current assets – $225 accounts payable – $830 accrued expenses – $120 current portion of long-term debt).

This question tested from Session 8, Reading 33, LOS d

Question 111 - #95536

Your answer: A was incorrect. The correct answer was C) Elegant's payment to purchase equipment to be used in its business.

Purchases of equipment are considered to be cash flows from investing. Interest paid or received and dividends received are considered to be cash flows from operations under U.S. GAAP.

This question tested from Session 8, Reading 34, LOS a

Question 112 - #97934

$1,665 million $490 million

Page 123: Financial Reporting and Analysis_The Income Statement, Balance Sheet, and Cash Flow Statement

Your answer: A was correct!

Set up the cash conversion cycle formula and solve for the missing variable, sales. Days in payables is equal to 73 [365 / 5 accounts payable turnover]. Days in inventory is equal to 36.5 [365 / ($30 million COGS / $3 million average inventory)]. Given the cash conversion cycle, days in inventory, and days in payables, calculate days in receivables of 50 [13.5 days cash conversion cycle + 73 days in payables – 36.5 days in inventory]. Given days in receivables of 50 and average receivables of $8 million, sales are $58.4 million [($8 million average receivables / 50 days) × 365].

This question tested from Session 8, Reading 35, LOS d

Question 113 - #97687

Your answer: A was incorrect. The correct answer was B) Cash Ratio.

Equity turnover and net profit margin are each measures of a company's operating performance.

This question tested from Session 8, Reading 35, LOS d

Question 114 - #98066

Your answer: A was correct!

Gains and losses result from, transactions that are not a part of the firm’s normal business operations. Expenses are amounts that are incurred to generate revenue; thus, expenses result from the firm’s ongoing operations. Both are included on the income statement.

This question tested from Session 8, Reading 32, LOS a

Question 115 - #97787

Your answer: A was incorrect. The correct answer was B) 1,100,000.

First, Check for dilution: Basic EPS = 1,500,000 / 1,000,000 = 1.50

Warrants: anti-dilutive since the average stock price is less than the exercise price

Convertible bonds: numerator impact = (# bonds) × (par value) × (interest rate) × (tax retention rate) × (0.5 for 1/2 year outstanding) = (10,000) × (100) × (0.06) × (0.6) × (0.5) = 18,000, so the numerator = 1,518,000 Denominator impact: increase in average shares = [(# bonds) × (conversion factor) × (# months outstanding)] / 12 = (1,200,000 / 12 = 100,000) so, the denominator = 1,100,000 and EPS with conversion = 1,518,000 / 1,100,000 = 1.38, which is less than 1.50. The bonds are dilutive and the diluted EPS calculation should use 1,100,000 shares of common stock in the denominator. The warrants are out of the money based on the average price of $20.

This question tested from Session 8, Reading 32, LOS h

Question 116 - #95134

Your answer: A was incorrect. The correct answer was C)

The reinvestment ratio measures a firm’s ability to acquire long-term assets with cash flows from operations. In contrast, the investing and financing ratio, which is more comprehensive, measures the firm’s ability to purchase

Reinvestment ratio Cash-to-income ratio

Page 124: Financial Reporting and Analysis_The Income Statement, Balance Sheet, and Cash Flow Statement

assets, satisfy debts, and pay dividends.

The cash-to-income ratio measures the ability to generate cash from a firm’s operations and is a performance ratio for cash flow analysis purposes. The debt payment ratio measures the firm’s ability to satisfy long-term debt with cash flow from operations but it is more of a coverage ratio than a performance ratio.

This question tested from Session 8, Reading 34, LOS h

© 2010 Kaplan Schweser

Page 125: Financial Reporting and Analysis_The Income Statement, Balance Sheet, and Cash Flow Statement

Kaplan Schweser Printable Exams - 2011 CFA Level 1

You can print this page by going to file -> print in your internet browser.

Question 1 - #97793

Protocol, Inc.’s net income for 2005 was $4,800,000. Protocol had 800,000 shares of common stock outstanding for the entire year. The tax rate was 40 percent. The average share price in 2005 was $37.00. Protocol had 5,000 8 percent $1,000 par value convertible bonds that were issued in 2004. Each bond is convertible into 25 shares of common stock. Protocol, Inc.’s basic and diluted earnings per share for 2005 were closest to:

Question 2 - #97784

Which of the following statements is CORRECT regarding the reporting of earnings per share (EPS)?

Question 3 - #97071

An analyst has gathered the following information about a company:

Basic EPS Diluted EPS

A) $6.00 $4.92

B) $5.19 $4.92

C) $6.00 $5.45

A) Diluted EPS must be less than or equal to basic EPS.

B) The EPS when antidilutive securities are converted into shares of common stock is less than basic EPS.

C) Basic EPS can be less than diluted EPS.

Balance Sheet AssetsCash 100Accounts Receivable 750Marketable Securities 300Inventory 850Property, Plant & Equip 900Accumulated Depreciation (150)

Total Assets 2750

Liabilities and EquityAccounts Payable 300Short-Term Debt 130Long-Term Debt 700

Page 126: Financial Reporting and Analysis_The Income Statement, Balance Sheet, and Cash Flow Statement

What is the ROE?

Question 4 - #97865

Valuable Corp.’s basic earnings per share (EPS) and diluted EPS for the year are different. Given this information, which of the following statements is least accurate?

Question 5 - #97422

Determine the cash flow from operations given the following table.

Common Equity 1000Retained Earnings 620

Total Liab. and Stockholder's equity 2750

Income Statement Sales 1500COGS 1100Gross Profit 400SG&A 150Operating Profit 250Interest Expense 25Taxes 75Net Income 150

A) 10.7%.B) 9.9%.C) 9.3%.

A) Diluted EPS is less than basic EPS.B) Valuable Corp.'s capital structure may include both options and warrants.C) All of Valuable's potentially dilutive securities are antidilutive.

Item AmountCash payment of dividends $30Sale of equipment $25Net income $25Purchase of land $15Increase in accounts payable $20Sale of preferred stock $25Increase in deferred taxes $5Profit on sale of equipment $15

A) $35.B) $20.C) $45.

Page 127: Financial Reporting and Analysis_The Income Statement, Balance Sheet, and Cash Flow Statement

Question 6 - #93537

Which of the following is least likely a cash flow in the calculation of cash flow from operations under U.S. GAAP?

Question 7 - #97044

An analyst has gathered the following information about a company.

The total asset turnover is 1.2.

The after-tax profit margin is 10%.

The financial leverage multiplier is 1.5.

Given this information, the company’s return on equity is:

Question 8 - #97346

Darth Corporation’s most recent income statement shows net sales of $6,000, and Darth’s marginal tax rate is 40%. The total expenses reported were $3,200, all of which were paid in cash. In addition, depreciation expense was reported at $800. A further examination of the most recent balance sheets reveals that accounts receivable during that period increased by $1,000. The cash flow from operating activities reported by Darth should be:

Question 9 - #98026

The “All Faiths” church is building a new church for $2 million on land acquired several years ago. The contractor estimates the cost at $1.3 million and the project is to be completed over a 2-year period with the payments split evenly between the 2 years. During the first year, the total costs incurred were $700,000. During the second year the contractor experienced cost overruns and costs incurred were $1.0 million. Using the percentage-of-completion method, how much revenue and income should the contractor recognize in the second year of the project?

A) Dividends received.B) Interest income.C) Interest paid.D)E) Dividends paid.F)

A) 18%.B) 12%.C) 9%.

A) $1,200.B) $2,200.C) $1,000.

Revenue Income

A) $923,077 -$76,923

Page 128: Financial Reporting and Analysis_The Income Statement, Balance Sheet, and Cash Flow Statement

Question 10 - #98013

Which of the following items regarding the corporate income statement is most accurate?

Question 11 - #97828

In calculating the numerator for diluted earnings per share, the dividends on convertible preferred stock are:

Question 12 - #97304

What is the impact on accounts receivable if sales exceed cash collections and what is the impact on accounts payable if cash paid to suppliers exceeds purchases?

Question 13 - #97256

When the market value of an investment in a debt security is less than its carrying value, how should the investor report the investment on the balance sheet if the security is classified as held-to-maturity and what amount should be reported if the security is classified as available-for-sale?

B) $1,076,923 $376,923

C) $1,000,000 $0

A)Examples of extraordinary items include expropriations of property and equipment by foreign governments, losses from earthquakes and tornados, and gains from the sale of investments in subsidiaries.

B)If a corporation disposes of a business segment that is separable from the company's core business activities, the results of the discontinued segment are reported as a separate line item below income from continuing operations on a pre-tax basis.

C) Unusual or infrequent items appear in the income statement of a corporation as a component of net income from continuing operations.

A) added to earnings available to common shareholders with an adjustment for taxes.B) subtracted from earnings available to common shareholders without an adjustment for taxes.C) added to earnings available to common shareholders without an adjustment for taxes.

A) Both accounts payable and accounts receivable will increase.B) Only accounts receivable will increase.C) Only accounts payable will increase.

Held-to-maturity Available-for-sale

A) Amortized cost Amortized cost

B) Amortized cost Fair value

C) Fair value Fair value

Page 129: Financial Reporting and Analysis_The Income Statement, Balance Sheet, and Cash Flow Statement

Question 14 - #94227

On January 1, 2004, JME purchased a truck that cost $24,000. The truck had an estimated useful life of 5 years and $4,000 salvage value. The amount of depreciation expense recognized in 2006 assuming that JME uses the double declining balance method is:

Question 15 - #98007

Which of the following is least likely reported net of tax on the income statement under U.S. GAAP?

Question 16 - #97964

The following information pertains the QRK Company:

One million shares of common stock outstanding at the beginning of 2005. 200,000 shares issued on the last day of March. 500,000 shares issued on the last day of June. 800,000 shares issued on the last day of September.

What is the number of shares that should be used to compute 2005 earnings per share for the QRK Company?

Question 17 - #97458

Capital Corp.’s activities in the year 2005 included the following:

At the beginning of the year, Capital purchased a cargo plane from Aviation Partners for $10 million in exchange for $2 million cash, $3 million in Capital Corp. bonds and $5 million in Capital Corp. preferred stock.

Interest of $150,000 was paid on the bonds, and dividends of $250,000 were paid on the preferred stock.

At the end of the year, the cargo plane was sold for $12,000,000 cash to Standard Company. Proceeds from the sale were used to pay off the $3 million in bonds held by Aviation Partners.

On Capital Corp.’s Statement of Cash Flow for the year ended December 31, 2005, cash flow from investments (CFI) related to the above activities is:

A) $5,760.B) $4,000. C) $3,456.

A) Income from discontinued operations.B) Interest expense.C) Extraordinary items.

A) 2.5 million.B) 1.6 million.C) 1.9 million.

A) $10,000,000.B) $9,750,000.

Page 130: Financial Reporting and Analysis_The Income Statement, Balance Sheet, and Cash Flow Statement

Question 18 - #97899

The following data applies to the XTC Company:

Sales = $1,000,000. Receivables = $260,000. Payables = $600,000. Purchases = $800,000. COGS = $800,000. Inventory = $400,000. Net Income = $50,000. Total Assets = $800,000. Debt/Equity = 200%.

What is the average collection period, the average inventory processing period, and the payables payment period for XTC Company?

Question 19 - #97993

Last year, the AKB Company had net income equal to $5 million. Combined state and local taxes were 45%. The firm paid $1 million to holders of its 1 million common shares and $250,000 to 100,000 preferred shareholders. What was AKB's earnings per share (EPS) last year?

Question 20 - #97998

Jersey, Inc.’s financial information included the following for its year ended December 31:

160,000 shares of common stock were outstanding for the entire year. 18,000 shares of 10%, $100 par value cumulative preferred stock were outstanding for the entire year. Common stock dividends paid during the current year were $240,000. All preferred stock dividends were paid for the current year. Net income was $720,000.

Basic earnings per share for Jersey, Inc. for the year ended December 31 are closest to:

C) $6,750,000.D) $6,600,000.E)F)

Average Collection Period

Average Inventory Processing Period

Payables Payments Period

A) 55 days 195 days 231 days

B) 95 days 183 days 274 days

C) 45 days 45 days 132 days

A) $4.75.B) $2.25.C) $2.50.

Page 131: Financial Reporting and Analysis_The Income Statement, Balance Sheet, and Cash Flow Statement

Question 21 - #97307

Maverick Company reported the following financial information for 2007:

Calculate Maverick’s cost of goods sold and cash paid to suppliers for 2007.

Question 22 - #97411

When using the indirect method for computing cash flow from operating activities, a change in accounts payable will require which of the following?

Question 23 - #97819

Anti-dilutive securities should:

Question 24 - #98027

Which of the following is NOT a requirement for revenue recognition to occur?

A) $4.50.B) $3.38.C) $2.81.

in millionsBeginning accounts receivable $180 Ending accounts receivable 225 Sales 11,000 Beginning inventory 2,000 Ending inventory 2,300 Purchases 8,100 Beginning accounts payable 1,600 Ending accounts payable 1,200

Cost of goods sold Cash paid to suppliers

A) $7,800 million $7,100 million

B) $7,800 million $8,500 million

C) $3,800 million $8,500 million

A) A negative (positive) adjustment to net income when accounts payable increases (decreases).B) A negative adjustment to net income regardless of whether accounts payable increases or decreases.C) A positive (negative) adjustment to net income when accounts payable increases (decreases).

A) be used in calculating basic EPS but not diluted EPS.B) be used in calculating diluted EPS but not basic EPS.C) not be used in calculating basic or diluted EPS.

Page 132: Financial Reporting and Analysis_The Income Statement, Balance Sheet, and Cash Flow Statement

Question 25 - #97765

Cassie Hamilton is an analyst with Pacers Worldwide, an investment banking firm. She just received the following information (as of year-end) for Trotters Diversified:

Average common shares outstanding of 5.0 million. Average market price for common stock of $35.00 per share. Net income of $9.0 million. Common stock dividends paid of $1.2 million. Preferred dividends paid (on convertible preferred stock noted below) of $1.5 million. Tax rate of 40%. 500,000 shares of cumulative convertible preferred stock with $30 par value and 10% dividend. Each preferred share is convertible into 5 common shares. 10,000 convertible $1,000 par bonds with a 6.0% coupon, each convertible into 8 shares of common stock. 400,000 stock options recently issued with an exercise price of $32.00 per share.

Part 1) In the denominator of the basic EPS calculation, Hamilton should include how many shares related to the convertible bonds?

Part 2) Hamilton correctly calculates diluted EPS at approximately:

Question 26 - #97967

Ajax Company has a simple capital structure. Which of the following will NOT be found on its balance sheet?

Question 27 - #97940

Peterson Painting Company is a commercial painting contractor. At the beginning of 20X7, Peterson’s net working capital was $350,000. The following transactions occurred during 20X7:

A) Cash must have been received.B) Earning activities are substantially completed.C) Transactions giving rise to revenue should be arms-length.

A) 80,000.B) 0.C) 10,000.

A) $1.23.B) $1.19.C) $1.50.

A) 6%, $50 par value callable bond.B) 3%, $100 par value convertible bond.C) 10%, secured mortgage bond denominated in Swiss francs.

Performed services on credit $150,000Purchased office equipment for cash 10,000Recognized salaries expense 54,000

Page 133: Financial Reporting and Analysis_The Income Statement, Balance Sheet, and Cash Flow Statement

Calculate Peterson’s working capital at the end of 20X7 and the change in cash for the year 20X7.

Question 28 - #97757

An analyst has gathered the following information about a company:

Purchased paint supplies on on credit 25,000Consumed paint supplies 20,000Paid salaries 50,000Collected accounts receivable 157,000Recognized straight-line depreciation expense 2,000Paid accounts payable 15,000

Working capital Change in cash

A) $414,000 $82,000

B) $416,000 $80,000

C) $416,000 $82,000

Balance Sheet AssetsCash 100Accounts Receivable 750Marketable Securities 300Inventory 850Property, Plant & Equip 900Accumulated Depreciation (150)

Total Assets 2750

Liabilities and EquityAccounts Payable 300Short-Term Debt 130Long-Term Debt 700Common Stock 1000Retained Earnings 620

Total Liab. and Stockholder's equity 2750

Income Statement Sales 1500COGS 1100Gross Profit 400SG&A 150Operating Profit 250Interest Expense 25Taxes 75Net Income 150

Page 134: Financial Reporting and Analysis_The Income Statement, Balance Sheet, and Cash Flow Statement

What is the current ratio?

Question 29 - #94989

The correct set of cash flow treatments as they relate to interest paid according to U.S. generally accepted accounting principles (GAAP) and International Accounting Standards (IAS) GAAP is:

Question 30 - #98024

Assume that the exercise price of an option is $10, and the average market price of the stock is $13. Assuming 999 options are outstanding during the entire year, what is the number of shares to be added to the denominator of the diluted earnings per share (EPS)?

Question 31 - #97971

If a reliable estimate of total costs of the contract does not exist, which of the following revenue recognition methods should be used?

Question 32 - #98015

When the cost of goods and services used are recognized as an expense in the same period that its generated revenue is recognized, which of the following principle(s) is (are) being described?

A) 2.67.B) 4.65.C) 0.22.

U.S. GAAP IAS GAAP

A) CFO CFO or CFF

B) CFF CFF

C) CFO or CFF CFO

A) 768.B) 231.C) 999.

A) Cost recovery method.B) Completed contract method.C) Percentage-of-completion method.

A) The matching and accrual principles.B) The matching principle for revenue and expense recognition.C) The accrual and expense recognition principles.

Page 135: Financial Reporting and Analysis_The Income Statement, Balance Sheet, and Cash Flow Statement

Question 33 - #97745

Matrix, Inc.’s common size income statement for the years ended December 31, 20X1 and 20X2 included the following information (percent of net sales):

Analysis of this data indicates that from 20X1 to 20X2:

Question 34 - #97437

Paragon Company's operating profits are $100,000, interest expense is $25,000, and earnings before taxes are $75,000. What is Paragon's interest coverage ratio?

Question 35 - #93593

Interest payments, either as part of a coupon payment or to creditors, are considered which type of cash flow under U.S. GAAP?

Question 36 - #97969

An analyst has gathered the following information about a company:

110,000 shares of common outstanding at the beginning of the year. The company repurchases 20,000 of its own common shares on July 1. Net income is $300,000 for the year. 10,000 shares of existing 10 percent cumulative $100 par preferred outstanding that is not in arrears at the beginning or ending of the year. The company also has $1 million in 10 percent callable bonds outstanding. The company has declared a $0.50 dividend on the common.

20X1 20X2Sales 100 100 Cost of Goods Sold (55) (60)Gross Profit 45 40 Selling General & Administrative (5) (5)Depreciation (7) (8)Operating Profit (EBIT) 33 37 Interest Expense (15) (7)Earnings before Taxes 18 30 Income Tax Expense (6) (10)Earnings after Taxes 12 20

A) interest expense per dollar of sales declined.B) cost of goods sold increased.C) the effective tax rate increased.

A) 1 time.B) 4 times.C) 3 times.

A) Financing.B) Investing.C) Operating.

Page 136: Financial Reporting and Analysis_The Income Statement, Balance Sheet, and Cash Flow Statement

What is the company's basic Earnings Per Share?

Question 37 - #119451

To study trends in a company’s cost of goods sold (COGS), an analyst should standardize COGS by dividing it by:

Question 38 - #98002

The Kammel Building Company has a contract to build a building for $100 million. The estimate of the cost of the project is $75 million. In the first year of the project, Kammel had costs of $30 million. Kammel’s reported profit for the first year of the contract, using the completed contract method, is:

Question 39 - #98047

The calculation of the income recognized in the third year of a five-year construction contract accounted for using the percentage-of-completion method includes the ratio of:

Question 40 - #97910

An analyst has collected the following data about a firm:

Receivables turnover = 10 times. Inventory turnover = 8 times. Payables turnover = 12 times.

What is the average receivables collection period, the average inventory processing period, and the average payables payment period? (assume 360 days in a year)

A) $3.00.B) $2.00.C) $1.00.

A) sales.B) net income.C) prior year COGS.

A) $15 million.B) $10 million.C) $0.

A) costs incurred in year 3 to total estimated costs.B) costs incurred in year 3 to total billings. C) total costs incurred to total estimated cost.

Receivables Collection Period

Inventory Processing Period

Payables Payment Period

A) 36 days 45 days 30 days

Page 137: Financial Reporting and Analysis_The Income Statement, Balance Sheet, and Cash Flow Statement

Question 41 - #94938

What is the difference between the direct and the indirect method of calculating cash flow from operations?

Question 42 - #97309

Adams Co.'s common sized balance sheet shows that:

Current Liabilities = 20% Equity = 45% Current Assets = 45% Total Assets = $2,000

What are Adams' long-term debt to equity ratio and working capital?

Question 43 - #97334

Convenience Travel Corp.’s financial information for the year ended December 31, 2004 included the following:

The only asset owned by Convenience Travel in 2005 was a corporate jet airplane. The airplane was being depreciated over a 15-year period on a straight-line basis at a rate of $1,000,000 per year. On December 31, 2005 Convenience Travel sold the airplane for $10,000,000 cash. Net income for the year ended December 31, 2005 was $12,000,000. Based on the above information, and ignoring taxes, what is cash flow from operations (CFO) for Convenience Travel for the year ended December 31, 2005?

B) 30 days 30 days 60 days

C) 45 days 36 days 30 days

A)The indirect method starts with gross income and adjusts to cash flow from operations, while the direct method starts with gross profit and flows through the income statement to calculate cash flows from operations.

B) Balance sheet items are not included in the cash flow from operations for the direct method, while they are included for the indirect method.

C) The direct method starts with sales and follows cash as it flows through the income statement, while the indirect method starts with net income and adjusts for non-cash charges and other items.

Debt to Equity Working Capital

A) 0.78 $250

B) 1.22 $500

C) 0.78 $500

Property Plant & Equipment $15,000,000Accumulated Depreciation 9,000,000

A) $13,000,000.B) $11,000,000.C) $12,000,000.D)

Page 138: Financial Reporting and Analysis_The Income Statement, Balance Sheet, and Cash Flow Statement

Question 44 - #97421

An analyst contemplates using the indirect methods to create the projected statement of cash flows. She decides to research the differences between the direct and indirect methods. Which of the following statements is most accurate? Under the:

Question 45 - #97956

Zimmer Co. had the following common shares outstanding:

January 1, 2003: 50,000 October 1, 2003: Issued 20,000 shares March 1, 2004: Issued a 10% stock dividend July 1, 2004: Declared a 2 for 1 stock split October 1, 2004: Repurchased 30,000 shares

Calculate the weighted average number of common shares outstanding for 2003 and 2004.

Question 46 - #97068

What is the net income of a firm that has a return on equity of 12%, a leverage ratio of 1.5, an asset turnover of 2, and revenue of $1 million?

Question 47 - #97766

During 2004, Covax Corp. reported net income of $2.4 million and 2 million shares of common stock. Covax paid cash dividends of $14,000 to its preferred shareholders and $30,000 to its common shareholders. In 2004, Covax issued 900, $1,000 par, 5.5 percent bonds for $900,000. Each bond is convertible to 50 shares of common stock. Assume the tax rate is 40%. Compute Covax’s basic and diluted EPS.

E)F) $8,000,000.

A) indirect method, depreciation must be added to net income, because it is a non-cash expense.B) direct method, depreciation must be added to cash collections because it is a non-cash expense.C) indirect method, changes in accounts receivable are already included in the net income figure.

2003 2004

A) 55,000 146,500

B) 55,000 124,500

C) 10,000 124,000

A) $40,000.B) $360,000.C) $36,000.

Basic EPS Diluted EPS

Page 139: Financial Reporting and Analysis_The Income Statement, Balance Sheet, and Cash Flow Statement

Question 48 - #98031

Walsh Furniture has purchased a machine with a 7-year useful life for $250,000. At the end of its life it will have an estimated salvage value of $15,000. Using the double-declining balance (DDB) method, depreciation expense in year 2 is closest to:

Question 49 - #97392

To convert an indirect statement of cash flows to a direct basis, the analyst would:

Question 50 - #97981

Washington, Inc.’s stock transactions during the year 20X4 were as follows:

What was Washington’s weighted average number of shares outstanding during 20X4, for earnings per share (EPS) computation purposes?

Question 51 - #96768

Summit Co. has provided the following information for its most recent reporting period:

A) $1.19 $1.18

B) $1.19 $1.22

C) $1.22 $1.22

A) $58,750.B) $51,020.C) $71,430.

A) reduce cost of goods sold by any decreases in accounts payable.B) reduce cost of goods sold by any decreases in inventory.C) increase cost of goods sold by any depreciation that was included.

January 1 720,000 shares issued and outstandingMay 1 2 for 1 stock split occurred

A) 1,500,000.B) 1,666,667.C) 1,440,000.

Beginning Figures Ending Figures Average Figures Sales $ 5,000,000 EBIT $ 800,000 Interest Expense $ 160,000 Taxes $ 256,000

Page 140: Financial Reporting and Analysis_The Income Statement, Balance Sheet, and Cash Flow Statement

What is Summit Co.’s total asset turnover and return on equity?

Question 52 - #97679

Lightfoot Shoe Company reported sales of $100 million for the year ended 20X7. Lightfoot expects sales to increase 10% in 20X8. Cost of goods sold is expected to remain constant at 40% of sales and Lightfoot would like to have an average of 73 days of inventory on hand in 20X8. Forecast Lightfoot’s average inventory for 20X8 assuming a 365 day year.

Question 53 - #98056

An analyst has gathered the following data pertaining to Hegel Company’s construction projects, which began during 2002:

Part 1)

If Hengel used the completed contract method, what amount of gross profit (loss) would Hengel report in its 2002 income statement for:

Part 2) If Hengel used the percentage-of-completion method, what amount of gross profit (loss) would Hengel report in its 2002 income statement?

Assets $ 3,500,000 $ 4,000,000 $ 3,750,000 Equity $ 1,700,000 $ 2,000,000 $ 1,850,000

Total Asset Turnover Return on Equity

A) 1.25 20.8%

B) 1.33 20.8%

C) 1.33 15.8%

A) $8.0 million.B) $22.0 million.C) $8.8 million.

Project 1 Project 2Contract price $420,000 $300,000Costs incurred in 2002 240,000 280,000Estimated costs to complete 120,000 40,000Billed to customers during 2002 150,000 270,000Received from customers during 2002 90,000 250,000

Project 1 Project 2

A) $0 $0

B) ($20,000) $0

C) $0 ($20,000)

Page 141: Financial Reporting and Analysis_The Income Statement, Balance Sheet, and Cash Flow Statement

Question 54 - #97374

An examination of the cash receipts and payments of Xavier Corporation reveals the following:

Under U.S. GAAP, Xavier’s reported cash flow from operations will be:

Question 55 - #97808

Advantage Corp.'s capital structure was as follows:

During 2005, Advantage paid dividends of $3 per share on its preferred stock. The preferred shares are convertible into 20,000 shares of common stock. The 8% bonds are convertible into 30,000 shares of common stock. Net income for 2005 was $850,000. Assume the income tax rate is 30%.

Calculate Advantage's basic and diluted earnings per share (EPS) for 2005.

Question 56 - #98063

A) $22,500.B) $20,000.C) $(20,000).

Cash paid to suppliers for purchase of merchandise $5,000Cash received from customers 14,000Cash paid for purchase of equipment 22,000Dividends paid 2,000 Cash received from issuance of preferred stock 10,000Interest received on short-term investments 1,000 Wages paid 4,000 Repayment of loan to the bank 5,000 Cash from sale of land 12,000

A) -$5,000.B) $6,000.C) $5,000.

December 31, 2005 December 31, 2004Outstanding shares of stock:

Common 110,000 110,000

Convertible Preferred 10,000 10,000

8% Convertible Bonds $1,000,000 $1,000,000

Basic EPS Diluted EPS

A) $6.31 $5.66

B) $7.45 $5.66

C) $7.45 $6.26

Page 142: Financial Reporting and Analysis_The Income Statement, Balance Sheet, and Cash Flow Statement

A video rental store with a large inventory of newly released movies is attempting to determine an appropriate method of depreciation for its movies for rental. As well, it is trying to determine an appropriate method of determining the cost of its inventory of movies for sale. Which of the following treatments is most appropriate for the movies for rental and movies for sale?

Question 57 - #97863

Given the following income statement and balance sheet for a company:

What is the average receivables collection period?

Movies for rental Movies for sale

A) Straight-line depreciation Last-in, first-out

B) Accelerated depreciation Last-in, first-out

C) Accelerated depreciation First-in, first-out

Balance Sheet Assets Year 2003 Year 2004Cash 500 450Accounts Receivable 600 660Inventory 500 550Total CA 1300 1660Plant, prop. equip 1000 1250Total Assets 2600 2,910

LiabilitiesAccounts Payable 500 550Long term debt 700 1102Total liabilities 1200 1652

EquityCommon Stock 400 538Retained Earnings 1000 720Total Liabilities & Equity 2600 2,910

Income Statement Sales 3000Cost of Goods Sold (1000)Gross Profit 2000SG&A 500Interest Expense 151EBT 1349Taxes (30%) 405Net Income 944

A) 80.3 days.B) 60.6 days.C) 76.7 days.

Page 143: Financial Reporting and Analysis_The Income Statement, Balance Sheet, and Cash Flow Statement

Question 58 - #97268

Consider the following statements.

With respect to these statements:

Question 59 - #97382

Which of the following statements regarding depreciation expense in the cash flow statements is CORRECT? Depreciation is added back to net income when determining CFO using:

Question 60 - #97389

Selected information from Rockway, Inc.’s U.S. GAAP financial statements for the year ended December 31, included the following (in $):

Using the direct method, cash provided or used by operating activities(CFO) in the year 2005 was:

Statement #1: Par value is a nominal dollar value assigned to shares of stock in a corporation’s charter.Statement #2: The par value of common stock represents the amount the corporation received when the

stock was issued.

A) both statements are correct.B) only statement #1 is correct.C) only statement #2 is correct.

A) either the direct or indirect methods.B) the indirect method.C) the direct method.

2004 2005 Sales 17,000,000 21,000,000Cost of Goods Sold 11,000,000 15,000,000Interest Paid 800,000 1,000,000Current Income Taxes Paid 700,000 1,000,000Accounts Receivable 3,000,000 2,500,000Inventory 2,400,000 3,000,000Property, Plant & Equip. 2,000,000 16,000,000Accounts Payable 1,000,000 1,400,000Long-term Debt 8,000,000 9,000,000Common Stock 4,000,000 5,000,000

A) $5,300,000.B) $6,300,000.C) $3,500,000.D) $4,300,000.E)F)

Page 144: Financial Reporting and Analysis_The Income Statement, Balance Sheet, and Cash Flow Statement

Question 61 - #97803

The primary difference between basic EPS and diluted EPS is that:

Question 62 - #97958

Robinson Company had 1 million shares outstanding at the beginning of the year. On April 1, Robinson issued an additional 300,000 shares. On July 1, Robinson issued 200,000 more shares. What is Robinson's weighted average number of shares outstanding for the calculation of earnings per share?

Question 63 - #97978

For a firm with a simple capital structure, all of the following are necessary to measure basic earnings per share (EPS) EXCEPT:

Question 64 - #97892

Given the following income statement and balance sheet for a company:

A) diluted EPS includes the potential effects of convertible securities while basic EPS does not. B) proprietors and partners report basic EPS but corporations report diluted EPS.

C) extraordinary items and discontinued operations are omitted from basic EPS but included in diluted EPS.

A) 1,500,000 shares.B) 1,325,000 shares.C) 1,200,000 shares.

A) dividends paid to common shareholders.B) the timing and number of shares issued or repurchased during the year.C) dividends paid to preferred shareholders.

Balance Sheet Assets Year 2003 Year 2004Cash 500 450Accounts Receivable 600 660Inventory 500 550Total CA 1300 1660Plant, prop. equip 1000 1250Total Assets 2600 2910

LiabilitiesAccounts Payable 500 550Long term debt 700 700Total liabilities 1200 1652

EquityCommon Stock 400 400

Page 145: Financial Reporting and Analysis_The Income Statement, Balance Sheet, and Cash Flow Statement

What is the operating profit margin?

Question 65 - #97711

Sampson Corp. had 500,000 shares of common stock outstanding at the beginning of the year. The average market price was $20.

On April 1, Sampson issued 100,000 shares of $1000 par value 10 percent preferred stock. On July 1, Sampson issued 200,000 warrants to purchase 10 shares of common stock each at $22 per share. On October 1, Sampson repurchased 60,000 of common stock as treasury stock for $15 per share.

The weighted average common shares outstanding Sampson should use to compute basic earnings per share (EPS) was:

Question 66 - #98044

Which of the following statements regarding the methods of revenue recognition is most accurate?

Question 67 - #100935

Guidance from the U.S. Securities and Exchange Commission regarding the criteria for revenue recognition least

Retained Earnings 1260 1260Total Liabilities & Equity 2600 2910

Income Statement Sales 3000Cost of Goods Sold (1000)Gross Profit 2000SG&A 500Interest Expense 151EBT 1349Taxes (30%) 405Net Income 944

A) 0.50.B) 0.45.C) 0.67.

A) 485,000.B) 515,000.C) 600,000.

A) The percentage-of-completion method generally results in lower retained earnings than the completed contract method.

B) The completed contract method is used when the selling price or cost estimates are unreliable.

C) The completed contract method, in comparison to the percentage-of-completion method, will generally result in higher net income.

Page 146: Financial Reporting and Analysis_The Income Statement, Balance Sheet, and Cash Flow Statement

likely specifies that there must be:

Question 68 - #119450

The only section of the statement of cash flows that must be adjusted to convert a statement of cash flows from the indirect to the direct method is:

Question 69 - #97279

Ascot Corporation has 4 million shares of common stock authorized, 2.4 million shares of common stock issued, and 1.8 million shares of common stock outstanding. How many shares of treasury stock does Ascot own and is the treasury stock reported as an asset in Ascot’s balance sheet?

Question 70 - #98091

Which revenue recognition method is used when the payment is assured and revenue is earned as costs are incurred?

Question 71 - #98073

When an unreliable estimate of costs exists and ultimate payment is assured, which of the following revenue recognition methods should be used?

A) evidence of an arrangement between the buyer and the seller.B) reasonable assurance that the product will be delivered or the service will be rendered.C) a determined or determinable price.

A) cash flows from investing.B) cash flows from operations.C) cash flows from financing.

Treasury shares Reported as an asset

A) 600,000 Yes

B) 1.6 million No

C) 600,000 No

A) Installment sales method.B) Cost recovery method.C) Percentage-of-completion method.

A) Percentage-of-completion method.B) Completed contract method.C) Cost recovery method.

Page 147: Financial Reporting and Analysis_The Income Statement, Balance Sheet, and Cash Flow Statement

Question 72 - #97350

Determine the cash flow from investing given the following table:

Question 73 - #98005

The Widget Company had net income of $1 million for the period. There were 1 million shares of widget common stock outstanding for the entire period. If there are 100,000 options outstanding with an exercise price of $40, what is the diluted earnings per share for Widget common stock if the average price per share over the period was $50?

Question 74 - #100931

When evaluating the differences between two revenue recognition policies, an analyst should view the policy as more conservative which:

Question 75 - #97914

A firm’s financial statements reflect the following:

What are the firm’s current ratio, quick ratio, and cash ratio?

Item AmountCash payment of dividends $30Sale of equipment $25Net income $25Purchase of land $15Increase in accounts payable $20Sale of preferred stock $25Increase in deferred taxes $5

A) $10.B) -$5.C) -$10.

A) $1.00.B) $0.98.C) $0.99.

A) recognizes revenue later.B) results in less leverage on the balance sheet.C) is more dependent on management estimates.

Current liabilities $4,000,000 Cash $400,000 Inventory $1,200,000 Accounts receivable $800,000 Short-term investments $2,000,000 Long-term investments $800,000 Accounts payable $2,500,000

Page 148: Financial Reporting and Analysis_The Income Statement, Balance Sheet, and Cash Flow Statement

Question 76 - #95144

Holden Company’s fixed asset footnote included the following:

During 20X7, Holden sold machinery for a gain of $100,000. The machinery had an original cost of $500,000 and its accumulated depreciation was $240,000. At the end of 20X7, Holden purchased machinery at a cost of $1,000,000. Holden paid $400,000 cash. The balance was financed by the seller at 8% interest. Depreciation expense was $2,080,000 for the year ended 20X7.

Calculate Holden’s cash flow from investing activities for the year ended 20X7.

Question 77 - #97811

Securities that would decrease earnings per share (EPS) if they were exercised and converted to common stock are called:

Question 78 - #97951

A company has convertible preferred stock outstanding. In the computation of diluted earnings per share, common shares issued when convertible preferred stock is converted are added to the denominator of the basic EPS equation, and the numerator is:

Question 79 - #97302

On January 1, 2008, Tenant Company leased office space from Landlord Inc. for 5 years at $75,000 per month. On that same date, Tenant made the following payments to Landlord:

Current Ratio Quick Ratio Cash Ratio

A) 0.8 0.6 1.1

B) 1.1 0.6 0.8

C) 1.1 0.8 0.6

A) $360,000 inflow.B) $40,000 outflow.C) $300,000 outflow.

A) dilutive securities.B) synthetic securities.C) antidilutive securities.

A) not adjusted.B) adjusted by adding back non-convertible preferred stock dividends.C) adjusted by adding back convertible preferred stock dividends.

First month’s rent $75,000

Page 149: Financial Reporting and Analysis_The Income Statement, Balance Sheet, and Cash Flow Statement

The leasehold improvements include build-out costs to install office walls, restrooms, and a kitchen. Tenant allocates the cost of the leasehold improvements over the lease term using the straight-line method. What amount of total lease expense should Tenant report for the year ended 2008 and what is the balance of all of the lease related assets on December 31, 2008, assuming the lease payments are made on the first day of each month?

Question 80 - #97991

In accounting for long-term construction contracts, the percentage-of-completion method is preferable to the completed contract method when:

Question 81 - #97989

An analyst gathered the following information about a company:

01/01/04 - 50,000 shares issued and outstanding at the beginning of the year 04/01/04 - 5% stock dividend 10/01/04 - 10% stock dividend

What is the company’s weighted average number of shares outstanding at the end of 2004?

Question 82 - #96534

When the return on equity equation (ROE) is decomposed using the original DuPont system, what three ratios comprise the components of ROE?

Last month’s rent 75,000 Security deposit 100,000 Lease improvements 1,500,000

Lease expense Lease related assets

A) $1,200,000 $1,375,000

B) $1,200,000 $1,200,000

C) $375,000 $1,375,000

A) the contracts are of a relatively short duration (less than one year). B) lack of dependable cost estimates cause forecasts to be doubtful.

C) estimates of the costs to complete and the extent of progress toward completion are reasonably dependable.

A) 57,500.B) 57,750.C) 55,000.

A) Net profit margin, asset turnover, asset multiplier.B) Net profit margin, asset turnover, equity multiplier.C) Gross profit margin, asset turnover, equity multiplier.

Page 150: Financial Reporting and Analysis_The Income Statement, Balance Sheet, and Cash Flow Statement

Question 83 - #97684

Bluff, Inc.’s stock transactions during the year were as follows:

January 1 90,000 common shares outstanding.

April 1 20% stock dividend is declared and issued.

October 1 10,000 shares are reacquired as treasury stock.

What is Bluff’s weighted average number of shares outstanding during the year?

Question 84 - #97873

An analyst has gathered the following data about a company:

Average receivables collection period of 37 days. Average payables payment period of 30 days. Average inventory processing period of 46 days.

What is their cash conversion cycle?

Question 85 - #98134

Galaxy Corporation manufactures custom motorcycles. Galaxy finances the motorcycles over 36 months for customers who make a minimum down payment of 10%. Historically, Galaxy has experienced bad debt losses equal to 1% of sales. Galaxy also provides a 24 month unlimited warranty on all new motorcycles. In the past, warranty expense has averaged 3% of sales. Ignoring taxes, how does the recognition of bad debt expense and warranty expense at the time of sale affect Galaxy’s liabilities?

Question 86 - #97032

If a firm has a net profit margin of 0.05, an asset turnover of 1.465, and a leverage ratio of 1.66, what is the firm's ROE?

A) 105,500.B) 98,000.C) 101,000.

A) 113 days.B) 53 days.C) 45 days.

Bad debt expense Warranty expense

A) No effect No effect

B) No effect Increase

C) Increase Increase

Page 151: Financial Reporting and Analysis_The Income Statement, Balance Sheet, and Cash Flow Statement

Question 87 - #97291

A firm has a cash conversion cycle of 80 days. The firm's payables turnover goes from 11 to 12, what happens to the firm's cash conversion cycle? It:

Question 88 - #97949

Zachary Company’s warrants issued in 2000 are Zachary’s only outstanding potentially dilutive security. In 2005, EPS and Dilutive EPS differed for the first time. A possible explanation for the change is the:

Question 89 - #97682

Which of the following items is NOT in the numerator of the quick ratio?

Question 90 - #95041

Consider the following:

Which of these arguments support the use of the indirect method for presenting cash flow from operating activities in the cash flow statement?

A) 12.16%.B) 3.18%.C) 5.87%.

A) shortens.B) lengthens.C) may shorten or lengthen.

A) year-end market price of Zachary increased.B) average market price of Zachary increased.C) average market price of Zachary decreased.

A) Cash.B) Receivables.C) Inventory.

Argument #1: The indirect method presents a firm’s operating cash receipts and payments and is thus more consistent with the objectives of the cash flow statement.

Argument #2: The indirect method provides more information than the direct method and is more useful to analysts in estimating future operating cash flows.

A) Neither argument.B) Argument #2 only.C) Argument #1 only.

Page 152: Financial Reporting and Analysis_The Income Statement, Balance Sheet, and Cash Flow Statement

Question 91 - #97443

The Red Company’s balance sheet as of December 31, 2004 was as follows:

Red’s interest expense was $900,000 and income tax expense was $1,000,000 in 2004. Red prepares its Statements of Cash Flows using the direct method.

The other cash outflows section of Cash Flow from Operations (CFO) for 2004 would total:

Question 92 - #97867

Earnings before interest and taxes (EBIT) is also known as:

Question 93 - #97306

Murray Company reported the following revenues and expenses for the year ended 2007:

Dec. 31, 2003 Dec. 31, 2004Cash $1,500,000 $1,900,000Accounts Receivable 3,000,000 3,400,000Inventory 2,300,000 2,500,000Property, Plant & Equipment 16,700,000 19,700,000Less Accumulated Depreciation (5,300,000) (8,200,000)Total Assets $18,200,000 $19,300,000 Accounts Payable $2,100,000 $1,900,000Interest Payable 800,000 1,200,000Income Taxes Payable 1,000,000 800,000Notes Payable 2,700,000 2,900,000Deferred Income Taxes 2,600,000 2,900,000Common Stock 1,000,000 1,000,000Retained Earnings 8,000,000 8,600,000

$18,200,000 $19,300,000

A) $1,700,000.B) $2,100,000.C) $1,900,000.D)E)F) $1,400,000.

A) operating profit.B) gross profit.C) earnings before income taxes.

Sales revenue $200,000 Wage expense 89,000 Insurance expense 17,000 Interest expense 10,400 Depreciation expense 50,000

Page 153: Financial Reporting and Analysis_The Income Statement, Balance Sheet, and Cash Flow Statement

Following are the related balance sheet accounts:

Calculate cash collections and cash expenses.

Question 94 - #97946

Selected information from Jupiter Corp.’s financial activities in the year 20X5 is as follows:

Net income is $18,300,000. 115,000 shares of common stock were outstanding on January 1. The average market price per share was $150 in 20X5. 200 warrants, which each allow the holder to purchase 100 shares of common stock at an exercise price of $100 per common share, were outstanding the entire year. 60,000 shares of common stock were issued on April 1. 45,000 shares of common stock were purchased by the company as treasury stock on October 1.

Jupiter Corp.’s diluted earnings per share for 20X5 are closest to:

Question 95 - #122496

Under accrual accounting, revenues are recognized in the same period in which the associated:

Question 96 - #97963

A 12 percent $100,000 convertible bond was issued on October 1, 2004. It is dilutive and can be converted into 18,000 shares. The effective income tax rate for the year was 40%. What adjustments should be made to calculate diluted earnings per share?

2007 2006 Unearned revenue $15,600 $13,200 Wages payable 5,400 6,600 Prepaid insurance 1,200 0 Interest payable 500 1,600 Accumulated depreciation 95,000 45,000

Cash collections Cash expenses

A) $202,400 $119,900

B) $202,400 $58,100

C) $197,600 $119,900

A) $117.75.B) $123.02.C) $159.13.

A) cash is collected.B) expenses are incurred.C) invoices are billed.

Page 154: Financial Reporting and Analysis_The Income Statement, Balance Sheet, and Cash Flow Statement

Question 97 - #97868

Are the following statements about common-size financial statements correct or incorrect?

Statement #1 – Expressing financial information in a common-size format enables the analyst to make better comparisons between two firms of similar size that operate in different industries.

Statement #2 – Common-size financial statements can be used to highlight the structural changes in the firm’s operating results and financial condition that have occurred over time.

With respect to these statements:

Question 98 - #98074

Suppose that JPK, Inc., paid dividends of $80,000 to its preferred shareholders and $40,000 to its common shareholders during 2004. The company had 20,000 shares of common stock issued and outstanding on January 1, 2004, issued 7,000 more shares on June 1, 2004, and paid a 10% stock dividend on August 1, 2004. Assuming that JPK had $150,000 in net income, what is the firm’s basic earnings per share (EPS) for 2004?

Question 99 - #97736

For the year ended December 31, 2007, Cobra Company reported the following financial information:

Ignoring taxes, calculate Cobra’s net income and comprehensive income for 2007.

Interest added to the numerator

Shares added to the denominator

A) $3,000 4,500

B) $3,000 18,000

C) $1,800 4,500

A) both are correct.B) both are incorrect.C) only one is correct:

A) $2.71.B) $2.91.C) $2.64.

Revenue $100,000 Cost of goods sold 40,000 Operating expenses 20,000 Unrealized gain from foreign currency translation 5,000 Unrealized loss on cash flow hedging derivatives 3,000 Dividends paid to common shareholders 7,500 Realized gain on sale of equipment 1,000

Net income Comprehensive income

Page 155: Financial Reporting and Analysis_The Income Statement, Balance Sheet, and Cash Flow Statement

Question 100 - #97439

An analyst has gathered the following information about a company:

What is the cash flow from financing?

Question 101 - #98109

Duster Company reported the following financial information at the end of 2007:

A) $40,000 $43,000

B) $41,000 $43,000

C) $41,000 $2,000

Income Statement for the YearSales $1,500ExpensesCOGS $1,300Depreciation 20Goodwill 10Int. Expenses 40

Total expenses 1,370Income from cont. op. 130

Gain on sale 30Income before tax 160Income tax 64Net Income $96

Additional Information:Dividends paid $30Common stock sold 20Equipment purchased 50Bonds issued 80Fixed asset sold for (original cost of $100 with accumulated depreciation of $70) 60

Accounts receivable decreased by 30Inventory decreased by 20Accounts payable increased by 20Wages payable decreased by 10

A) $110.B) $70.C) $130.

in millions Unearned revenue $240 Common stock at par 30

Page 156: Financial Reporting and Analysis_The Income Statement, Balance Sheet, and Cash Flow Statement

Calculate Duster’s liabilities and stockholders’ equity as of December 31, 2007.

Question 102 - #97916

A company has a receivables turnover of 10, an inventory turnover of 5, and a payables turnover of 12. The company’s cash conversion cycle is closest to:

Question 103 - #97829

An analyst has gathered the following information about Barnstabur, Inc., for the year:

Reported net income of $30,000. 5,000 shares of common stock and 2,000 shares of 8%, $90 par preferred stock outstanding during the whole year. During the year, Barnstabur issued at par, $60,000 of 6.0% convertible bonds, with each of the 60 bonds convertible into 110 shares of the Barnstabur common stock.

If Barnstabur’s effective tax rate is 40%, what will Barnstabur report for diluted earnings per share (EPS)?

Question 104 - #97069

Comparative income statements for E Company and G Company for the year ended December 31 show the following (in $ millions):

Capital in excess of par 440 Accounts payable 1,150 Treasury stock 2,000 Retained earnings 5,160 Accrued expenses 830 Accumulated other comprehensive loss 210 Long-term debt 1,570

Liabilities Stockholders' equity

A) $3,790 million $3,420 million

B) $3,790 million $7,420 million

C) $3,550 million $7,840 million

A) 30 days.B) 79 days.C) 37 days.

A) $1.53.B) $2.36.C) $1.66.

E Company G CompanySales 70 90Cost of Goods Sold (30) (40) Gross Profit 40 50

Page 157: Financial Reporting and Analysis_The Income Statement, Balance Sheet, and Cash Flow Statement

The financial risk of E Company, as measured by the interest coverage ratio, is:

Question 105 - #98137

Which of the following is NOT a category on the statement of cash flows? Cash flow from:

Question 106 - #97995

For an organization with a simple capital structure, the computation of earnings per share is least likely to consider:

Question 107 - #100937

Jerry Krome, CFA, is an equity analyst. The head of research at Krome’s firm composes a memo that contains the following statements:

To the extent that management has discretion over the firm’s revenue recognition, an analyst should consider policies that recognize revenue later to be more conservative than policies that recognize revenue sooner. When comparing the performance of companies, an analyst can use the information in the financial statement disclosures to adjust the financial statements for differences in revenue recognition policies.

With regard to the implications of revenue recognition policies for financial analysis, Krome should agree with:

Question 108 - #98071

Sales and Administration (5) (15)Depreciation (5) (10) Operating Profit 30 25Interest Expense (20) (5) Earnings Before Taxes 10 20Income Taxes (4) (8) Earnings after Taxes 6 12

A) higher than G Company's because its interest coverage ratio is less than G Company's, but at least one-third of G Company's.

B) lower than G Company's because its interest coverage ratio is at least three times G Company's.C) higher than G Company's because its interest coverage ratio is less than one-third of G Company's.

A) sales.B) financing.C) operations.

A) net income.B) the weighted average number of preferred shares outstanding.C) the weighted average number of common shares outstanding.

A) only one of these statements.B) both of these statements.C) neither of these statements.

Page 158: Financial Reporting and Analysis_The Income Statement, Balance Sheet, and Cash Flow Statement

A firm’s financial statements reflect the following:

Based on this information, what is the firm’s basic EPS?

Question 109 - #97825

In calculating the numerator for diluted Earnings Per Share, the interest on convertible debt is:

Question 110 - #95965

Jodi Lein, small business consultant, is currently working with RJ Landscaping, a sole proprietorship. She is trying to educate the owner on the importance of monitoring cash flows. Operating information as of the end of the most recent month appears below:

Cash from sale of truck of $7,000. Cash salaries paid of $17,000. Cash from customers of $45,000. Depreciation expense of $5,500. Interest on bank line of credit of $1,000. Cash paid to suppliers of $22,000. Other cash expenses, including rent, of $6,300. No taxes due.

Using this information, what is the cash flow from operations for the month?

Question 111 - #97960

A simple capital structure is least likely to include:

Net income $1,700,000 EBIT $2,900,000 Effective tax rate 35% Interest payments $285,000 Common equity $3,100,000 Total assets $6,600,000 Preferred dividends paid $1,100,000 Weighted avg. shares outstanding 523,000

A) $2.75.B) $3.25.C) $1.15.

A) subtracted from earnings available to common shareholders after an adjustment for taxes.B) added to earnings available to common shareholders after an adjustment for taxes.C) added to earnings available to common shareholders.

A) $11,200.B) -$300.C) -$1,300.

Page 159: Financial Reporting and Analysis_The Income Statement, Balance Sheet, and Cash Flow Statement

Question 112 - #96591

Kellen Harris is a credit analyst with the First National Bank. Harris has been asked to evaluate Longhorn Supply Company’s cash needs. Harris began by calculating Longhorn’s turnover ratios for 2007. After a discussion with Longhorn’s management, Harris decides to adjust the turnover ratios for 2008 as follows:

Longhorn’s expected cash conversion cycle for 2008, based on the expected changes in turnover and assuming a 365 day year, is closest to:

Question 113 - #97755

Lawson, Inc.’s net income for the year was $1,060,000 with 420,000 shares outstanding. Lawson has 2,000 shares of 8%, $1,000 par value convertible preferred stock that were outstanding the entire year. Each share of preferred is convertible into 50 shares of common stock. Lawson's diluted earnings per share are closest to:

Question 114 - #97715

According to the Financial Accounting Standards Board, what is the appropriate balance sheet treatment for available-for-sale securities and where are the unrealized gains and losses reported?

A) treasury stock.B) convertible bonds.C) callable preferred stock.

2007 Actual Turnover

Expected Increase / (Decrease)

Accounts receivable 5.0 10% Fixed asset 3.0 7% Accounts payable 6.0 (20%) Inventory 4.0 (5%) Equity 5.5 — Total asset 2.3 8%

A) 82 days.B) 86 days.C) 46 days.

A) $1.94.B) $2.14.C) $2.04.

Balance sheet Unrealized gains and losses

A) Fair value Other comprehensive income

B) Amortized cost Other comprehensive income

C) Fair value Net income

Page 160: Financial Reporting and Analysis_The Income Statement, Balance Sheet, and Cash Flow Statement

Question 115 - #97954

A firm has had the following numbers of shares outstanding during the year:

Based on this information, what is the weighted number of shares outstanding for the year?

Question 116 - #97894

Rushford Corp.’s net income is $16,500,000 with 300,000 shares outstanding. The tax rate is 40%. The average share price for the year was $372. Rushford has 50,000, 9%, $1,000 par value convertible bonds outstanding. Each bond is convertible into two shares of common stock.

Rushford Corp.’s basic and diluted earnings per share (EPS) are closest to:

Question 117 - #97813

Securities that improve basic per share earnings, or reduce per share losses, if they are exercised or converted to common stock are called:

Question 118 - #98020

Pinto Corporation is an automobile manufacturer located in North America. Pinto owns a 5 percent interest in one of its suppliers, Continental Supply Company. Each year, Pinto receives a cash dividend from Continental. Pinto’s engine supplier, National Supply Company, recently increased prices on goods sold to all customers due to higher labor costs. Should Pinto report the dividends received from Continental and the price increase from National as an operating or nonoperating component on its year-end income statement?

Beginning of year 10,000,000 shares Issued on April 1 500,000 shares Split 2 for 1 on July 1 Issued on October 1 100,000 shares Split 2 for 1 on December 31

A) 42,400,000.B) 20,780,000.C) 41,550,000.

Basic EPS Dilutied EPS

A) $65.63 $48.00

B) $55.00 $51.56

C) $55.00 $48.00

A) dilutive securities.B) embedded securities.C) antidilutive securities.

A) Both are nonoperating.B) Both are operating.

Page 161: Financial Reporting and Analysis_The Income Statement, Balance Sheet, and Cash Flow Statement

© 2010 Kaplan Schweser

C) Only one is operating.

Page 162: Financial Reporting and Analysis_The Income Statement, Balance Sheet, and Cash Flow Statement

Kaplan Schweser Printable Exams - Test Management Exam 21

Test ID#: 7790094

Question 1 - #97793

Your answer: A was incorrect. The correct answer was C)

Protocol’s basic EPS (net income / weighted average common shares outstanding) was $4,800,000 / 800,000 = $6.00. Diluted EPS is calculated under the assumption that the convertible bonds were converted into common stock, and the bond interest net of tax was restored to net income. The common shares from the conversion of the bonds are added to the denominator of the equation. Protocol’s Diluted EPS was [$4,800,000 + (5,000 × $1,000 × 0.08)(1 − 0.40)] / [800,000 + (5,000 × 25)] = $5.45.

This question tested from Session 8, Reading 32, LOS h

Question 2 - #97784

Your answer: A was correct!

Antidilutive securities are securities that would increase EPS if exercised or converted to common stock.

This question tested from Session 8, Reading 32, LOS h

Question 3 - #97071

Your answer: A was incorrect. The correct answer was C) 9.3%.

ROE = 150(NI) / [1000(common) + 620(RE)] = 150 / 1620 = 0.0926 or 9.3%

This question tested from Session 8, Reading 35, LOS f

Question 4 - #97865

Your answer: A was incorrect. The correct answer was C) All of Valuable's potentially dilutive securities are antidilutive.

If all of Valuable’s potentially dilutive securities were antidilutive, then EPS would equal diluted EPS.

This question tested from Session 8, Reading 32, LOS h

Question 5 - #97422

Your answer: A was correct!

$6.00 $5.45

Item AmountCash payment of dividends CFF -$30Sale of equipment CFI +$25Net income CFO +$25

Page 163: Financial Reporting and Analysis_The Income Statement, Balance Sheet, and Cash Flow Statement

CFO = 25(NI) + 20(AP) + 5(Def Tax) − 15(Equip Profit) = $35

This question tested from Session 8, Reading 34, LOS e

Question 6 - #93537

Your answer: A was incorrect. The correct answer was E) Dividends paid.

According to SFAS 95, dividends paid are treated as cash flow from financing.

This question tested from Session 8, Reading 34, LOS a

Question 7 - #97044

Your answer: A was correct!

ROE = profit margin × total asset turnover × financial leverage ROE = (0.1)(1.2)(1.5) = 0.18 or 18.0%

This question tested from Session 8, Reading 35, LOS f

Question 8 - #97346

Your answer: A was incorrect. The correct answer was C) $1,000.

Net income is ($6,000 – 3,200 – 800)(1 – 0.4) = $1,200. Adjustments to reconcile net income to cash flow from operating activities will require that depreciation ($800) be added back, and increase in accounts receivable ($1,000) be subtracted: $1,200 + 800 – 1,000 = $1,000.

This question tested from Session 8, Reading 34, LOS e

Question 9 - #98026

Your answer: A was correct!

During the first year, the revenue was 700,000 / 1,300,000 × 2,000,000 = 1,076,923

The total revenue for both years = $2,000,000

The second year revenue was 2,000,000 – 1,076,923 = $923,077

The second year income = revenues − costs = 923,077 – 1,000,000 = $-76,923

This question tested from Session 8, Reading 32, LOS b

Question 10 - #98013

Purchase of land CFI -$15Increase in accounts payable CFO +$20Sale of preferred stock CFF +$25Increase in deferred taxes CFO +$5Profit on sale of equipment CFO -$15

Page 164: Financial Reporting and Analysis_The Income Statement, Balance Sheet, and Cash Flow Statement

Your answer: A was incorrect. The correct answer was C) Unusual or infrequent items appear in the income statement of a corporation as a component of net income from continuing operations.

Explanations for incorrect answers are as follows:

The gain on the sale of a subsidiary is an unusual or infrequent item. The results of a discontinued segment are reported below the line, net of tax (after tax).

This question tested from Session 8, Reading 32, LOS f

Question 11 - #97828

Your answer: A was incorrect. The correct answer was C) added to earnings available to common shareholders without an adjustment for taxes.

Diluted EPS = [(Net income − Preferred dividends) + Convertible preferred dividends + (Convertible debt interest)(1 − t)] / [(Weighted average shares) + (Shares from conversion of conv. pfd shares) + (Shares from conversion of conv. debt) + (Shares issuable from stock options)]

This question tested from Session 8, Reading 32, LOS h

Question 12 - #97304

Your answer: A was incorrect. The correct answer was B) Only accounts receivable will increase.

If a firm sells more than it collects, accounts receivable will increase. If a firm pays suppliers more than it purchases, accounts payable will decrease.

This question tested from Session 8, Reading 34, LOS e

Question 13 - #97256

Your answer: A was incorrect. The correct answer was B)

Held-to-maturity securities are reported on the balance sheet at amortized cost while available-for-sale securities are reported at fair value. Amortized cost includes the amortization of a premium or discount that was created when the security was purchased.

This question tested from Session 8, Reading 33, LOS f

Question 14 - #94227

Your answer: A was incorrect. The correct answer was C) $3,456.

yr. 2004 = 24,000 × 2/5 = 9,600

yr. 2005 = (24,000 − 9,600) × 2/5 = 5,760

yr. 2006 = (24,000 − 9,600 − 5,760) × 2/5 = 3,456

This question tested from Session 8, Reading 32, LOS d

Amortized cost Fair value

Page 165: Financial Reporting and Analysis_The Income Statement, Balance Sheet, and Cash Flow Statement

Question 15 - #98007

Your answer: A was incorrect. The correct answer was B) Interest expense.

Interest expense would be considered an expense that is incurred from continuing operations and, therefore, is listed prior to subtracting the income tax expense on the income statement. Income from discontinued operations and extraordinary items are included on the income statement after the net income from continuing operations is reported and after the income tax expense from continuing operations is reported. Therefore, these latter accounts are reported net of tax.

This question tested from Session 8, Reading 32, LOS f

Question 16 - #97964

Your answer: A was incorrect. The correct answer was B) 1.6 million.

The weighted average number of common shares outstanding is the number of shares outstanding during the year weighted by the portion of the year they were outstanding. For the QRK Company, the weighted number of shares outstanding is the original one million shares plus 150,000 shares for the end-of-March issue (= 200,000 × 9/12), plus 250,000 shares for the end-of-June issue (= 500,000 × 6/12), plus 200,000 shares for the end-of-September issue (= 800,000 × 3/12), or 1.6 million shares.

This question tested from Session 8, Reading 32, LOS g

Question 17 - #97458

Your answer: A was correct!

Investing cash of $2 million was used to purchase the cargo plane. Proceeds from the sale of the plane were a source of $12 million of investing cash. Net CFI is $12 million − $2 million = $10 million. The interest payment is included in cash from operations (CFO) and the dividend payment in cash from financing (CFF). Redemption of the bonds is a use of cash from financing (CFF).

This question tested from Session 8, Reading 34, LOS e

Question 18 - #97899

Your answer: A was incorrect. The correct answer was B)

Receivables turnover = $1,000,000 / $260,000 = 3.840 Average collection period = 365 / 3.840 = 95.05 or 95 days

Inventory turnover = $800,000 / $400,000 = 2 Average inventory processing period = 365 / 2 = 183 days

Payables turnover ratio = $800,000 / $600,000 = 1.333 Payables payment period = 365 / 1.333 = 273.82 or 274 days

This question tested from Session 8, Reading 35, LOS d

Question 19 - #97993

Your answer: A was correct!

95 days 183 days 274 days

Page 166: Financial Reporting and Analysis_The Income Statement, Balance Sheet, and Cash Flow Statement

EPS = earnings available to common shareholders divided by the weighted average number of common shares outstanding. Earnings available to common shareholders is net income minus preferred dividends, or $4,750,000 (= $5 million – 250,000) for AKB.

This question tested from Session 8, Reading 32, LOS g

Question 20 - #97998

Your answer: A was incorrect. The correct answer was B) $3.38.

Jersey, Inc.’s basic EPS = (net income – preferred dividends) / (weighted average number of common shares outstanding) was ($720,000 - $180,000)/160,000 = $3.38.

This question tested from Session 8, Reading 32, LOS g

Question 21 - #97307

Your answer: A was incorrect. The correct answer was B)

Cost of goods sold is equal to $7,800 million ($2,000 million beginning inventory + $8,100 million purchases – $2,300 million ending inventory). Cash paid to suppliers is equal to $8,500 million (–$7,800 COGS – $300 million increase in inventory – $400 million decrease in accounts payable). Alternate solution: Cash paid to suppliers is equal to $8,500 million (–$8,100 million purchases – $400 decrease in accounts payable).

This question tested from Session 8, Reading 34, LOS e

Question 22 - #97411

Your answer: A was incorrect. The correct answer was C) A positive (negative) adjustment to net income when accounts payable increases (decreases).

A decrease in accounts payable represents an outflow. Hence, a negative adjustment will be required. Conversely, an increase represents an inflow and a positive adjustment.

This question tested from Session 8, Reading 34, LOS e

Question 23 - #97819

Your answer: A was incorrect. The correct answer was C) not be used in calculating basic or diluted EPS.

Antidilutive securities would increase EPS if exercised or converted to common stock.

This question tested from Session 8, Reading 32, LOS h

Question 24 - #98027

Your answer: A was correct!

Revenue from credit sales may be recognized when sales are on account.

Other conditions when revenues are also considered earned include when: revenue can be measured with reasonable accuracy, transactions are not subject to revocation, it is possible to measure the cost of provided

$7,800 million $8,500 million

Page 167: Financial Reporting and Analysis_The Income Statement, Balance Sheet, and Cash Flow Statement

goods (no significant contingent obligation), and there is assurance of payment (cash) or collectability.

This question tested from Session 8, Reading 32, LOS b

Question 25 - #97765

Part 1) Your answer: A was incorrect. The correct answer was B) 0.

The calculation for basic EPS excludes the impact of complex capital elements.

This question tested from Session 8, Reading 32, LOS h

Part 2) Your answer: A was incorrect. The correct answer was B) $1.19.

As we will show below, only the options and convertible preferred stock are dilutive.

First, calculate basic EPS to use as a benchmark to determine dilutive capital components. Basic EPS = (net income – preferred dividends) / weighted average common shares outstanding Here, preferred dividends = (0.5 shares × $30 par × 0.10 dividend) = $1.5 million = (9.0 – 1.5) / 5.0 = $1.50.

Now, check for dilutive elements.

options are dilutive because the exercise price is less than the stock price. There is no numerator impact from the options. The denominator impact = # options – [(# options × exercise price) / average stock price)] = 400,000 – [(400,000 × 32) / 35] = 34,286 or 0.034 million. To check whether the convertible preferred stock is dilutive we need to determine whether it decreases EPS. To the numerator, we add back the preferred dividend. The denominator impact = (# preferred shares × conversion rate) = 500,000 × 5 = 2,500,000, or 2.5 million. Then, EPS = (9.0 – 1.5 + 1.5) / (5.0 + 2.5) = $1.20. Thus the convertible preferred stock is dilutive.

To check whether the convertible bonds are dilutive we need to determine whether they decrease EPS. To the numerator, we add back the after-tax impact of the coupon, or (face value × coupon × (1 − t)), or (10,000 bonds × 1,000 par × 0.06 coupon × 0.6 ) = 360,000, or $0.360 million. The denominator impact = (# convertible bonds × conversion rate) = 10,000 × 8 = 80,000, or 0.080 million. Then, EPS = (9.0 – 1.5 + 0.360) / (5.0 + 0.080) = $1.55. Thus the bonds are antidilutive.

Finally, calculate dilutive EPS: Diluted EPS = (9.0 – 1.5 + 1.5) / (5.0 + 2.5 + 0.034) = approximately $1.19

This question tested from Session 8, Reading 32, LOS h

Question 26 - #97967

Your answer: A was incorrect. The correct answer was B) 3%, $100 par value convertible bond.

If convertible bonds exist, the firm has a complex capital structure.

This question tested from Session 8, Reading 32, LOS g

Question 27 - #97940

Your answer: A was incorrect. The correct answer was C) $416,000 $82,000

Page 168: Financial Reporting and Analysis_The Income Statement, Balance Sheet, and Cash Flow Statement

The change in cash was $82,000 ($157,000 collections – $10,000 from equipment purchase – $50,000 salaries paid – $15,000 for payables).

Working capital at the end of 20X7 is $416,000 ($350,000 beginning working capital + $150,000 increase in accounts receivable from services – $10,000 office equipment purchase – $54,000 salaries expense accrual – $20,000 consumed supplies).

Purchasing $25,000 of paint supplies on credit has no net effect on working capital (current assets and current liabilities increase). Consuming $20,000 of these supplies reduces working capital (current assets decrease). Salary expense reduces working capital by $54,000 when recognized (current liabilities increase). Paying $50,000 of these salaries has no net effect on working capital (current assets and current liabilities decrease). Collecting accounts receivable has no net effect on working capital (one current asset increases and another decreases). Recognizing depreciation does not affect working capital. Paying accounts payable has no net effect on working capital (current assets and current liabilities decrease).

This question tested from Session 8, Reading 33, LOS d

Question 28 - #97757

Your answer: A was incorrect. The correct answer was B) 4.65.

Current ratio = [100(cash) + 750(AR) + 300(marketable securities) + 850(inventory)] / [300(AP) + 130(short-term debt)] = (2,000 / 430) = 4.65

This question tested from Session 8, Reading 35, LOS d

Question 29 - #94989

Your answer: A was correct!

U.S. GAAP treats interest paid as CFO whereas IAS GAAP treats interest paid as either CFO or CFF.

This question tested from Session 8, Reading 34, LOS c

Transaction Amount Working capital CashPerformed services on credit $150,000 Increase A/RPurchased PP&E for cash 10,000 Decrease cash -$10,000Recognized salaries expense 54,000 Increase A/PPurchased paint supplies on on credit 25,000 Increase

inventories, increase A/P

Consumed paint supplies 20,000 Decrease inventories

Paid salaries 50,000 Decrease cash, decrease A/P

-$50,000

Collected accounts receivable 157,000 Increase cash, decrease A/R

+$157,000

Recognized straight-line depreciation expense 2,000Paid accounts payable 15,000 Decrease cash,

decrease A/P-$15,000

Page 169: Financial Reporting and Analysis_The Income Statement, Balance Sheet, and Cash Flow Statement

Question 30 - #98024

Your answer: A was incorrect. The correct answer was B) 231.

(999)(10) = 9,990 9,990 / 13 = 768 999 − 768 = 231

This question tested from Session 8, Reading 32, LOS h

Question 31 - #97971

Your answer: A was incorrect. The correct answer was B) Completed contract method.

The cost recovery method is used when future cash collections are not assured even after receipt of partial payments. Gross profit is not recognized until all of the cost of goods sold is collected.

The percentage-of-completion method is used when ultimate payment is assured and revenue is earned as costs are incurred. Profit is recognized corresponding to the percentage of costs incurred to the total estimated.

This question tested from Session 8, Reading 32, LOS b

Question 32 - #98015

Your answer: A was incorrect. The correct answer was B) The matching principle for revenue and expense recognition.

The accrual concept states that revenue is recognized when the earnings process is completed and cash receipt is assured.

This question tested from Session 8, Reading 32, LOS c

Question 33 - #97745

Your answer: A was correct!

On a common size income statement, all amounts are stated as a percentage of net sales. Dollars of interest expense per dollar of sales has declined from 0.15 to 0.07. The other interpretations listed are not necessarily correct. The volume of sales is not shown on this common-size income statement. COGS increased as a percentage of sales, but if sales volume decreased, COGS may have decreased as well. The company's effective tax rate (income tax expense / pretax income) can be calculated from a common-size income statement. Here the effective tax rate was 33% in both years.

This question tested from Session 8, Reading 35, LOS c

Question 34 - #97437

Your answer: A was incorrect. The correct answer was B) 4 times.

ICR = operating profit ÷ I = EBIT ÷ I = 100,000 ÷ 25000 = 4

This question tested from Session 8, Reading 35, LOS d

Page 170: Financial Reporting and Analysis_The Income Statement, Balance Sheet, and Cash Flow Statement

Question 35 - #93593

Your answer: A was incorrect. The correct answer was C) Operating.

Under U.S. GAAP, interest paid is an operating cash flow.

This question tested from Session 8, Reading 34, LOS a

Question 36 - #97969

Your answer: A was incorrect. The correct answer was B) $2.00.

Interest is already deducted from earnings.

This question tested from Session 8, Reading 32, LOS g

Question 37 - #119451

Your answer: A was correct!

In a common-size income statement, each income statement account is divided by sales. COGS is then production costs as a percentage of the sales price.

This question tested from Session 8, Reading 35, LOS c

Question 38 - #98002

Your answer: A was incorrect. The correct answer was C) $0.

Under the completed contract method, profit is only reported upon completion of the contract.

This question tested from Session 8, Reading 32, LOS b

Question 39 - #98047

Your answer: A was incorrect. The correct answer was C) total costs incurred to total estimated cost.

The percentage of completion method recognizes revenues in proportion to the proportion of expenses incurred. Using only the current year's costs produces an incorrect result if the estimated total cost has changed. Revenue recognized in any given year is costs to date divided by total estimated costs, minus revenue that has already been recognized.

This question tested from Session 8, Reading 32, LOS b

Question 40 - #97910

Your answer: A was correct!

Receivables collection period = 360 / 10 = 36 days

Page 171: Financial Reporting and Analysis_The Income Statement, Balance Sheet, and Cash Flow Statement

Inventory processing period = 360 / 8 = 45 days

Payables payment period = 360 / 12 = 30 days

This question tested from Session 8, Reading 35, LOS d

Question 41 - #94938

Your answer: A was incorrect. The correct answer was C) The direct method starts with sales and follows cash as it flows through the income statement, while the indirect method starts with net income and adjusts for non-cash charges and other items.

The main difference between the direct and indirect methods of calculating cash flows is the way that cash flow from operations is calculated. The direct method starts with sales and follows cash as it flows through the income statement, while the indirect method starts with income after taxes and adjusts backwards for non-cash and other items. Both methods will have the same result for operating cash flows. The direct and indirect method calculates the financing and investing cash flows the same way and both methods will result in the same cash flow figure.

This question tested from Session 8, Reading 34, LOS c

Question 42 - #97309

Your answer: A was incorrect. The correct answer was C)

If equity equals 45% of assets, and current liabilities equals 20%, then long-term debt must be 35%.

Long-Term Debt / Equity = 0.35 / 0.45 = 0.78

Working capital = CA − CL = 45% - 20% = 25% of assets

WC = 2,000(0.25) = $500

This question tested from Session 8, Reading 35, LOS d

Question 43 - #97334

Your answer: A was incorrect. The correct answer was F) $8,000,000.

Using the indirect method, CFO is net income increased by 2005 depreciation ($1,000,000) and decreased by the gain recognized on the sale of the plane [$10,000,000 sale price − ($15,000,000 original cost − $10,000,000 accumulated depreciation including 2005) = $5,000,000]. $12,000,000 + $1,000,000 − $5,000,000 = $8,000,000.

This question tested from Session 8, Reading 34, LOS e

Question 44 - #97421

Your answer: A was correct!

The indirect method begins with net income, which has already included all cash and non-cash expenses. Therefore, under the indirect method, depreciation must be added to net income, because it is a non-cash expense.

This question tested from Session 8, Reading 34, LOS e

0.78 $500

Page 172: Financial Reporting and Analysis_The Income Statement, Balance Sheet, and Cash Flow Statement

Question 45 - #97956

Your answer: A was correct!

For year 2003: 50,000 × 12 = 600,000 20,000 × 3 = 60,000 660,000/12 = 55,000

For year 2004: 70,000 × 1.1 × 2 = 154,000 × 12 = 1,848,000 (30,000) × 3 = (90,000) 1,758,000 / 12 = 146,500

This question tested from Session 8, Reading 32, LOS g

Question 46 - #97068

Your answer: A was correct!

The traditional DuPont system is given as:

ROE = (net profit margin)(asset turnover)(leverage ratio)

Solving for the net profit margin yields:

0.12 = (net profit margin) × (2) × (1.5)

0.04 = (net profit margin)

Recognizing that the net profit margin is equal to net income / revenue we can substitute that relationship into the above equation and solve for net income:

0.04 = net income / revenue = net income / $1,000,000

$40,000 = net income.

This question tested from Session 8, Reading 35, LOS f

Question 47 - #97766

Your answer: A was correct!

2004 Basic EPS:

2004 Diluted EPS:

This question tested from Session 8, Reading 32, LOS g

Page 173: Financial Reporting and Analysis_The Income Statement, Balance Sheet, and Cash Flow Statement

Question 48 - #98031

Your answer: A was incorrect. The correct answer was B) $51,020.

This question tested from Session 8, Reading 32, LOS d

Question 49 - #97392

Your answer: A was incorrect. The correct answer was B) reduce cost of goods sold by any decreases in inventory.

Decreases in inventory represent a source of cash so these would be subtracted from cost of goods sold. Any depreciation and/or amortization included in the cost of goods sold does not represent an actual use of cash, so this amount should be subtracted from cost of goods sold. Decreases in accounts payable represent a use of cash so these should be added to cost of goods sold.

This question tested from Session 8, Reading 34, LOS f

Question 50 - #97981

Your answer: A was incorrect. The correct answer was C) 1,440,000.

The January 1 balance is adjusted retroactively for the stock split and (720,000 × 2 =) 1,440,000 shares are treated as outstanding from January.

This question tested from Session 8, Reading 32, LOS g

Question 51 - #96768

Your answer: A was incorrect. The correct answer was B)

Total asset turnover = sales / average assets = 5,000,000 / 3,750,000 = 1.33

Return on equity = net income / average equity Net income = EBIT − interest − taxes = 800,000 − 160,000 − 256,000 = 384,000 ROE = 384,000 / 1,850,000 = 20.8%

This question tested from Session 8, Reading 35, LOS f

Question 52 - #97679

Your answer: A was incorrect. The correct answer was C) $8.8 million.

20X8 sales are expected to be $110 million [$100 million × 1.1] and COGS is expected to be $44 million [$110 million sales × 40%]. With 73 days of inventory on hand, average inventory is $8.8 million [($44 million COGS / 365) × 73 days].

Year 2 / Depreciable Life × Book Value at Beginning of the Year = Depreciation

1 0.2857 250,000 71,429 2 0.2857 178,571 51,020

1.33 20.8%

Page 174: Financial Reporting and Analysis_The Income Statement, Balance Sheet, and Cash Flow Statement

This question tested from Session 8, Reading 35, LOS h

Question 53 - #98056

Part 1) Your answer: A was incorrect. The correct answer was C)

No profit is recognized until the completion of the project, however losses are recognized. Project 2 has an expected loss of $20,000.

This question tested from Session 8, Reading 32, LOS b

Part 2) Your answer: A was incorrect. The correct answer was B) $20,000.

Under the percentage of completion method, $40,000 of profit is recognized for project 1. 120,000 + 240,000 = 360,000 total costs; 240,000 / 360,000 × 60,000 estimated profit = $40,000 profit.

Project 2 is running at a $20,000 loss. If the loss can be estimated the loss must be recognized at the time it is estimated. Total revenue for project 2 = 300,000 contract price − 320,000 total costs = -$20,000 estimated loss

40,000 (project 1) − 20,000 (project 2) = $20,000 gross profit in 2002

This question tested from Session 8, Reading 32, LOS b

Question 54 - #97374

Your answer: A was incorrect. The correct answer was B) $6,000.

Cash flow relating to operating activities includes cash paid to suppliers, cash received from customers, interest received, and wages paid. –5,000 + 14,000 + 1,000 + –4,000 = 6,000.

This question tested from Session 8, Reading 34, LOS a

Question 55 - #97808

Your answer: A was incorrect. The correct answer was B)

Basic EPS = net income − pref div / wt. ave. shares of common

[850,00 − (3 × 10,000)] / 110,000 = $7.45

Diluted EPS = [(net income − preferred dividends) + convertible preferred dividends + (convertible debt interest)(1 − t)] / [(weighted average shares) + (shares from conversion of conv. pfd shares) + (shares from conversion of conv. debt) + (shares issuable from stock options)]

[(850,000 − (3 × 10,000)) + 30,000 + (80,000)(1 − 0.3)] / [(110,000) + (20,000) + (30,000)] = $5.66.

This question tested from Session 8, Reading 32, LOS h

Question 56 - #98063

$0 ($20,000)

$7.45 $5.66

Page 175: Financial Reporting and Analysis_The Income Statement, Balance Sheet, and Cash Flow Statement

Your answer: A was incorrect. The correct answer was C)

With the movies for rental, a greater portion of the decrease in the value of newly released movies would reasonably be realized in the first year, given the rapid rate of obsolescence in view of the large number of movies available. Therefore, depreciating this pool of assets by a greater amount in the first year using an accelerated depreciation method better approximates economic depreciation than depreciating it straight line.

With the movies for sale, there are two methods available for accounting as inventory. FIFO is appropriate for inventory that has a limited shelf life and LIFO is appropriate for inventory that does not deteriorate with age. Because the movies have a very limited shelf life and will greatly deteriorate in value with age, especially after the first year, FIFO is the most appropriate method of accounting for the movies for sale.

This question tested from Session 8, Reading 32, LOS d

Question 57 - #97863

Your answer: A was incorrect. The correct answer was C) 76.7 days.

Average collection period = 365 / receivables turnover Receivables turnover = sales / average receivables = 3,000 / 630 = 4.76 Average receivables collection period = 365 / 4.76 = 76.65

This question tested from Session 8, Reading 35, LOS d

Question 58 - #97268

Your answer: A was incorrect. The correct answer was B) only statement #1 is correct.

The par value of common stock is the stated or nominal value assigned to the stock. Par value has no relationship to market value. The amount the corporation receives from the issuance of common stock is equal to the par value plus the additional paid-in-capital (proceeds in excess of par).

This question tested from Session 8, Reading 33, LOS g

Question 59 - #97382

Your answer: A was incorrect. The correct answer was B) the indirect method.

Depreciation is a non-cash expense. Only in the indirect method is depreciation added back to net income when determining CFO because net income is only used in the indirect method and not the direct method. The direct method instead starts with cash sales and works down the income statement.

This question tested from Session 8, Reading 34, LOS e

Question 60 - #97389

Your answer: A was incorrect. The correct answer was D) $4,300,000.

Cash provided or used by operating activities under the direct method is computed by adding cash inflows and subtracting cash inputs and cash outflows. Operating Cash inflows for Rockway Inc. for 2005 came from sales ($21,000,000) and decrease in accounts receivable ($3,000,000 − $2,500,000 = $500,000) for net cash inflows of ($21,000,000 + $500,000 =) $21,500,000. Operating cash inputs were cost of goods sold ($15,000,000), plus the increase in inventory ($3,000,000 − $2,400,000 = $600,000) less the increase in accounts payable, (which is a source of funds) ($1,000,000 − $1,400,000 = -$400,000) for net cash inputs of ($15,000,000 + $600,000 -

Accelerated depreciation First-in, first-out

Page 176: Financial Reporting and Analysis_The Income Statement, Balance Sheet, and Cash Flow Statement

$400,000 =) $15,200,000. Other operating cash outflows were interest paid ($1,000,000) and current income taxes paid ($1,000,000) totaling ($2,000,000). Cash provided by operations was ($21,500,000 − $15,200,000 − $2,000,000 =) $4,300,000. Changes in property, plant and equipment, long-term debt and common stock do not affect cash from operations.

This question tested from Session 8, Reading 34, LOS e

Question 61 - #97803

Your answer: A was correct!

The primary difference between basic EPS and diluted EPS is that diluted EPS includes the potential effects of convertible securities while basic EPS does not.

This question tested from Session 8, Reading 32, LOS h

Question 62 - #97958

Your answer: A was incorrect. The correct answer was B) 1,325,000 shares.

Weighted average shares = 1,000,000 + (0.75) 300,000 + (0.5) 200,000 = 1,325,000 shares

This question tested from Session 8, Reading 32, LOS g

Question 63 - #97978

Your answer: A was correct!

Basic EPS = earnings available to common shareholders divided by the weighted average number of common shares outstanding. Earnings available to common shareholders equals net income minus preferred dividends.

This question tested from Session 8, Reading 32, LOS g

Question 64 - #97892

Your answer: A was correct!

Operating profit margin = (EBIT / sales) = (1,500 / 3,000) = 0.5

This question tested from Session 8, Reading 35, LOS d

Question 65 - #97711

Your answer: A was correct!

Only the October 1 transaction affects the weighted average common shares outstanding because the April 1 transaction would not affect the number of shares outstanding and the July 1 transaction involves warrants which would not be included in the basic EPS calculation. The computation for basic EPS is [(500,000 × 12) − (60,000 × 3)] / 12 = 485,000.

This question tested from Session 8, Reading 32, LOS g

Page 177: Financial Reporting and Analysis_The Income Statement, Balance Sheet, and Cash Flow Statement

Question 66 - #98044

Your answer: A was incorrect. The correct answer was B) The completed contract method is used when the selling price or cost estimates are unreliable.

The completed contract method compared to the percentage-of-completion method will result in lower net income since revenue is recognized later. Hence, retained earnings will also be lower than the percentage-of-completion method.

This question tested from Session 8, Reading 32, LOS b

Question 67 - #100935

Your answer: A was incorrect. The correct answer was B) reasonable assurance that the product will be delivered or the service will be rendered.

One of the SEC’s criteria for revenue recognition is that the product has been delivered or the service has been rendered. The other criteria are evidence of an arrangement between the buyer and seller; the price has been determined or is determinable; and the seller is reasonably assured of collecting money.

This question tested from Session 8, Reading 32, LOS b

Question 68 - #119450

Your answer: A was incorrect. The correct answer was B) cash flows from operations.

The cash flows from investing activities and cash flows from financing activities sections of the statement of cash flows are the same for both the indirect and direct methods. Only the cash flows from operations section must be adjusted to convert the statement of cash flows from the indirect to the direct method.

This question tested from Session 8, Reading 34, LOS f

Question 69 - #97279

Your answer: A was incorrect. The correct answer was C)

Shares that were issued previously but are not outstanding are treasury shares (owned by the firm). Thus, there are 600,000 treasury shares (2.4 million issued – 1.8 million outstanding). Treasury shares are reported as a reduction in shareholders’ equity on the balance sheet. Treasury stock is not an asset.

This question tested from Session 8, Reading 33, LOS g

Question 70 - #98091

Your answer: A was incorrect. The correct answer was C) Percentage-of-completion method.

The installment sales method is used when the assurance of payment and estimated bad debts does not exist before cash is collected. Sales revenue and COGS are recognized only when cash is received.

The cost recovery method is used when future cash collections are not assured even after receipt of partial payments. Gross profit is not recognized until all of the cost of goods sold is collected.

This question tested from Session 8, Reading 32, LOS b

600,000 No

Page 178: Financial Reporting and Analysis_The Income Statement, Balance Sheet, and Cash Flow Statement

Question 71 - #98073

Your answer: A was incorrect. The correct answer was B) Completed contract method.

The key word is "unreliable." The completed contract method is used when cost estimates are unreliable. The percentage-of-completion method recognizes profit corresponding to the percentage of cost incurred to total estimated costs associated with long-term construction contracts. Percent-of-completion is used where contracts and cost estimates are reliable.

The cost recovery method is similar to the installment sales method but is more conservative. Sales are recognized when cash is received, but no gross profit is recognized until all of the cost of goods sold is collected.

This question tested from Session 8, Reading 32, LOS b

Question 72 - #97350

Your answer: A was correct!

CFI = Sale of Equipment (+25) + Purchase of Land (–15) = $10.

This question tested from Session 8, Reading 34, LOS e

Question 73 - #98005

Your answer: A was incorrect. The correct answer was B) $0.98.

Use the Treasury stock method Proceeds = 100,000 ($40) = $4,000,000 Shares assumed purchased with proceeds= $4,000,000/$50 = 80,000 shares Potential dilution = 100,000 – 80,000 = 20,000 shares Basic EPS = $1/share Diluted EPS = $1,000,000 / 1,020,000 = $0.98/share

This question tested from Session 8, Reading 32, LOS h

Question 74 - #100931

Your answer: A was correct!

Recognizing revenue later rather than sooner is considered more conservative. More aggressive (less conservative) revenue recognition can result in less leverage by increasing assets.

This question tested from Session 8, Reading 32, LOS b

Question 75 - #97914

Item AmountCash payment of dividends CFF -$30Sale of equipment CFI +$25Net income CFO +$25Purchase of land CFI -$15Increase in accounts payable CFO +$20Sale of preferred stock CFF +$25Increase in deferred taxes CFO +$5

Page 179: Financial Reporting and Analysis_The Income Statement, Balance Sheet, and Cash Flow Statement

Your answer: A was incorrect. The correct answer was C)

Current ratio = (0.4 + 2.0 + 0.8 + 1.2) / 4.0 = 1.1.

Quick ratio = (0.4 + 2.0 + 0.8) / 4.0 = 0.8.

Cash ratio = (0.4 + 2.0) / 4.0 = 0.6.

This question tested from Session 8, Reading 35, LOS d

Question 76 - #95144

Your answer: A was incorrect. The correct answer was B) $40,000 outflow.

Given the gain of $100,000 and book value of the machinery sold of $260,000 ($500,000 original cost – $240,000 accumulated depreciation), the proceeds from the sale of the machinery were $360,000 ($100,000 gain + $260,000 book value). For 20X7, CFI was an outflow of $40,000 ($360,000 sale proceeds – $400,000 machinery purchase). The $600,000 financed by the seller is a non-cash transaction and is reported in the notes to the cash flow statement.

This question tested from Session 8, Reading 34, LOS a

Question 77 - #97811

Your answer: A was correct!

Dilutive securities are securities that decrease EPS if they are exercised or converted to common stock. Stock options, warrants, convertible debt, and convertible preferred stock are examples of dilutive securities.

This question tested from Session 8, Reading 32, LOS h

Question 78 - #97951

Your answer: A was incorrect. The correct answer was C) adjusted by adding back convertible preferred stock dividends.

If convertible preferred stock is dilutive, the preferred dividends that would not have been paid if the preferred stock is converted must be added back to the numerator. Note that any nonconvertible preferred stock dividends are still subtracted from net income in the numerator.

This question tested from Session 8, Reading 32, LOS h

Question 79 - #97302

Your answer: A was correct!

Total annual lease expense is $1,200,000 [$75,000 monthly payment × 12 months) + ($1,500,000 lease improvements / 5 years)]. At the end of 2008, Tenant will report lease related assets of $1,375,000 [$75,000 prepaid rent + 100,000 deposit + $1,200,000 book value of leasehold improvements].

This question tested from Session 8, Reading 33, LOS h

1.1 0.8 0.6

Page 180: Financial Reporting and Analysis_The Income Statement, Balance Sheet, and Cash Flow Statement

Question 80 - #97991

Your answer: A was incorrect. The correct answer was C) estimates of the costs to complete and the extent of progress toward completion are reasonably dependable.

In accounting for long-term construction contracts, the percentage-of-completion method is preferable to the completed contract method when estimates of the costs to complete and the extent of progress toward completion are reasonably dependable.

This question tested from Session 8, Reading 32, LOS b

Question 81 - #97989

Your answer: A was incorrect. The correct answer was B) 57,750.

The weighted average number of common shares outstanding is the number of shares outstanding during the year weighted by the portion of the year they were outstanding. Dividends and splits are applied to all shares issued or repurchased and all original or adjusted shares outstanding prior to the split or dividend. Step 1) Apply the 04/01/04 dividend to the beginning-of-year shares: Adjusted shares = 1.05 × 50,000 = 52,500 Step 2) Apply the 10/01/04 dividend the adjusted beginning-of-year shares. Adjusted beginning of year shares = 57,750 (= 1.1 × 52,500). Step 3) Compute the weighted average number of shares. 57,750 × (12/12) = 57,750 shares.

This question tested from Session 8, Reading 32, LOS g

Question 82 - #96534

Your answer: A was incorrect. The correct answer was B) Net profit margin, asset turnover, equity multiplier.

The three ratios can be further decomposed as follows:

Net profit margin = net income/sales

Asset turnover = sales/assets

Equity multiplier = assets/equity

This question tested from Session 8, Reading 35, LOS f

Question 83 - #97684

Your answer: A was correct!

This question tested from Session 8, Reading 32, LOS g

Question 84 - #97873

Initial shares: 90,000 × 1.20 = 108,000– Reacquired treasury shares: 10,000 × 3/12 = –2,500

105,500

Page 181: Financial Reporting and Analysis_The Income Statement, Balance Sheet, and Cash Flow Statement

Your answer: A was incorrect. The correct answer was B) 53 days.

Cash conversion cycle = average receivables collection period + average inventory processing period – payables payment period = 37 + 46 – 30 = 53 days.

This question tested from Session 8, Reading 35, LOS d

Question 85 - #98134

Your answer: A was incorrect. The correct answer was B)

The recognition of bad debt expense has no effect on liabilities, current revenues are reduced by the expected amount of uncollectable accounts. Bad debt expense reduces net income and reduces assets. The recognition of expected warranty expense decreases net income (following the matching principle), and since it is not currently paid (doesn’t reduce assets) it creates or increases a liability at the time of sale.

This question tested from Session 8, Reading 33, LOS a

Question 86 - #97032

Your answer: A was correct!

One of the many ways to express ROE = net profit margin × asset turnover × leverage ratio

ROE = (0.05)(1.465)(1.66) = 0.1216

This question tested from Session 8, Reading 35, LOS f

Question 87 - #97291

Your answer: A was incorrect. The correct answer was B) lengthens.

CCC = collection period + Inv Period – Payment period.

Payment period = (365 / payables turnover) = (365 / 11) = 33; (365 / 12) = 30. This means the CCC actually increased to 83.

This question tested from Session 8, Reading 35, LOS d

Question 88 - #97949

Your answer: A was incorrect. The correct answer was B) average market price of Zachary increased.

An increase in average market price could cause Zachary’s warrants to go from antidilutive to dilutive. If the average price of the stock increases during the year, the warrants are likely to be exercised at some point during the year. Neither of the other choices would do this.

This question tested from Session 8, Reading 32, LOS h

Question 89 - #97682

Your answer: A was incorrect. The correct answer was C) Inventory.

No effect Increase

Page 182: Financial Reporting and Analysis_The Income Statement, Balance Sheet, and Cash Flow Statement

Quick ratio = (cash + marketable securities + receivables) / current liabilities

Current ratio = (cash + marketable securities + receivables + inventory) / current liabilities

This question tested from Session 8, Reading 35, LOS d

Question 90 - #95041

Your answer: A was correct!

It is the direct method, not the indirect method, that presents operating cash receipts and payments and is thus more consistent with the objectives of the cash flow statement. The direct method provides more information than the indirect method and is preferred by analysts who are estimating future cash flows.

This question tested from Session 8, Reading 34, LOS d

Question 91 - #97443

Your answer: A was incorrect. The correct answer was F) $1,400,000.

Other cash outflows is the third step in calculating CFO using the direct method. It consists of Cash taxes paid + Cash interest paid.

Cash interest paid = interest expense less increase in interest payable: ($900,000 – (1,200,000 - $800,000) =) $500,000.

Other cash outflows = $500,000 + 900,000 = $1,400,000

This question tested from Session 8, Reading 34, LOS e

Question 92 - #97867

Your answer: A was correct!

Operating profit = earnings before interest and taxes (EBIT)

Gross profit = net sales – COGS

Net income = earnings after taxes = EAT

This question tested from Session 8, Reading 35, LOS d

Question 93 - #97306

Your answer: A was correct!

Cash collections are $202,400 ($200,000 sales + $2,400 increase in unearned revenue). Cash expenses are $119,900 (–$89,000 wages expense – $1,200 decrease in wages payable – $17,000 insurance expense – $1,200

Cash taxes paid = tax expense of $1,000,000 + decrease in income taxes payable (1,000,000-800,000) = 200,000 - increase in deferred income taxes (2,600,000-2,900,000) = 300,000

$900,000

Page 183: Financial Reporting and Analysis_The Income Statement, Balance Sheet, and Cash Flow Statement

increase in prepaid insurance – $10,400 interest expense – $1,100 decrease in interest payable). Depreciation expense is a non-cash expense.

This question tested from Session 8, Reading 34, LOS e

Question 94 - #97946

Your answer: A was correct!

To compute Jupiter’s basic earnings per share (EPS) use the formula: (net income − preferred dividends) / weighted average common shares outstanding. Weighted average common shares outstanding = [(115,000 × 12) + (60,000 × 9) – (45,000 × 3)] / 12 = 148,750. Basic EPS = $18,300,000 / 148,750 = $123.02.

Using the treasury stock method, if the warrants were exercised cash inflow would be 200 × $100 × 100 = $2,000,000. The number of Jupiter shares that can be purchased with this cash at the average share price is $2,000,000 / $150 = 13,333. The net number of shares that would have been created is 20,000 − 13,333 = 6,667. Diluted EPS = $18,300,000 / (148,750 + 6,667) = $117.75. Since diluted EPS is less than basic EPS, the warrants are dilutive.

This question tested from Session 8, Reading 32, LOS h

Question 95 - #122496

Your answer: A was incorrect. The correct answer was B) expenses are incurred.

Accrual accounting is based on the matching principle, under which revenues are recognized in the same period that the expenses are incurred to generate those revenues.

This question tested from Session 8, Reading 32, LOS c

Question 96 - #97963

Your answer: A was incorrect. The correct answer was C)

The interest expense for three months net of tax is added to the numerator (12% × $100,000 × 3/12 × 60 %) = $1,800. The number of shares added to the denominator are 4,500. (18,000 × 3 / 12).

This question tested from Session 8, Reading 32, LOS h

Question 97 - #97868

Your answer: A was incorrect. The correct answer was C) only one is correct:

Vertical common-size statements enable the analyst to make better comparisons of two firms of different sizes that operate in the same industry. Horizontal common-size financial statements express each line as a percentage of the base year figure; thus, horizontal common-size statements can be used to identify structural changes in a firm’s operating results and financial condition over time.

This question tested from Session 8, Reading 35, LOS a

Question 98 - #98074

$1,800 4,500

Page 184: Financial Reporting and Analysis_The Income Statement, Balance Sheet, and Cash Flow Statement

Your answer: A was incorrect. The correct answer was C) $2.64.

1/1/00 22,000 shares (adjusted for 10% stock dividend) × 12 months = 264,000

6/1/00 7,700 shares (adjusted for 10% stock dividend) × 7 months = 53,900

Total share month = 317,900

Average shares = 317,900 / 12 = 26,492

Basic EPS = ($150,000 − $80,000) / 26,492 = 2.64

This question tested from Session 8, Reading 32, LOS g

Question 99 - #97736

Your answer: A was incorrect. The correct answer was B)

Net income is equal to $41,000 ($100,000 revenue – $40,000 COGS – $20,000 operating expenses + $1,000 realized gain on sale of equipment). Comprehensive income includes all transactions that affect stockholders’ equity except transactions with shareholders. Comprehensive income includes net income, unrealized gains and losses from available-for-sales securities, unrealized gains and losses from cash flow hedging derivatives, and gains and losses from foreign currency translation. Thus, comprehensive income is equal to $43,000 ($41,000 net income + $5,000 unrealized gain from foreign currency translation – $3,000 unrealized loss from cash flow hedging derivatives). Dividends paid is a transaction with shareholders and is not included in comprehensive income.

This question tested from Session 8, Reading 32, LOS i

Question 100 - #97439

Your answer: A was incorrect. The correct answer was B) $70.

This question tested from Session 8, Reading 34, LOS e

Question 101 - #98109

Your answer: A was correct!

Liabilities are equal to $3,790 million ($240 million unearned revenue + $1,570 long-term debt + $1,150 accounts payable + $830 accrued expenses). Stockholders’ equity is equal to $3,420 million ($30 common stock at par + $440 capital in excess of par – $2,000 treasury stock + $5,160 retained earnings – $210 accumulated other comprehensive loss).

This question tested from Session 8, Reading 33, LOS a

Question 102 - #97916

$41,000 $43,000

Dividends paid -$30Sale of stock 20Bonds issued 80

CFF $70

Page 185: Financial Reporting and Analysis_The Income Statement, Balance Sheet, and Cash Flow Statement

Your answer: A was incorrect. The correct answer was B) 79 days.

Cash conversion cycle = receivables days + inventory processing days – payables payment period. Receivables days = 365 / receivables turnover = 365 / 10 = 36.5 days. Inventory processing days = 365 / inventory turnover = 365 / 5 = 73.0 days. Payables payment period = 365 / payables turnover = 365 / 12 = 30.4 days. Cash collection cycle = 36.5 + 73.0 – 30.4 = 79.1 days.

This question tested from Session 8, Reading 35, LOS d

Question 103 - #97829

Your answer: A was correct!

Diluted EPS = adjusted earnings after conversion (EAC) / weighted average plus potential common shares outstanding.

Step 1: Calculate Adjusted EAC

preferred dividends = (0.08)(90)(2,000) = 14,400

convertible debt interest = (60,000)(0.06)(1 – 0.40) = 2,160

adjusted EAC = (30,000 – 14,400 + 2,160) = $17,760

Step 2: Calculate Weighted average plus potential common shares outstanding.

Step 3: Calculate Diluted EPS

Diluted EPS = 17,760 / 11,600 = $1.53.

This question tested from Session 8, Reading 32, LOS h

Question 104 - #97069

Your answer: A was incorrect. The correct answer was C) higher than G Company's because its interest coverage ratio is less than one-third of G Company's.

E Company’s interest coverage ratio (EBIT / interest expense) is (30 / 20) = 1.5.

G Company’s interest coverage ratio is (25 / 5) = 5.0. Higher interest coverage means greater ability to cover required interest and lease payments. Note that 1.5 / 5.0 = 0.30, which means the interest coverage for E Company is less than 1/3 that of G Company.

This question tested from Session 8, Reading 35, LOS e

adjusted EAC: net income - preferred dividends+ after-tax interest on convertible debt= adjusted earnings available for common shares

weighted average common shares = 5,000shares from conversion of convertible bonds = (60 × 110) = 6,600weighted ave. plus potential common shares outst. = 11,600

Page 186: Financial Reporting and Analysis_The Income Statement, Balance Sheet, and Cash Flow Statement

Question 105 - #98137

Your answer: A was correct!

There are only three types of cash flows: financing, investing, and operating.

This question tested from Session 8, Reading 34, LOS a

Question 106 - #97995

Your answer: A was incorrect. The correct answer was B) the weighted average number of preferred shares outstanding.

The equation for Basic EPS (net income – preferred dividends / weighted average number of common shares outstanding) does not include the number of preferred shares outstanding, because the objective is to determine the earnings available to the common shareholder.

This question tested from Session 8, Reading 32, LOS g

Question 107 - #100937

Your answer: A was correct!

Because revenue recognition often relies on judgment and estimates from management, it is not always possible to calculate the appropriate adjustments that would account for the differences between companies’ revenue recognition policies. An analyst should use the policies disclosed in companies’ financial statement footnotes to understand the degree to which their revenue recognition is conservative or aggressive. In general, recognizing revenue sooner is considered aggressive and recognizing revenue later is considered conservative.

This question tested from Session 8, Reading 32, LOS b

Question 108 - #98071

Your answer: A was incorrect. The correct answer was C) $1.15.

The firm’s basic EPS = ($1,700,000 – $1,100,000) / (523,000) = $1.147.

This question tested from Session 8, Reading 32, LOS g

Question 109 - #97825

Your answer: A was incorrect. The correct answer was B) added to earnings available to common shareholders after an adjustment for taxes.

Formula = Diluted EPS = [(Net income − Preferred dividends) + Convertible preferred dividends + (Convertible debt interest)(1 − t)] / [(Weighted average shares) + (Shares from conversion of conv. pfd shares) + (Shares from conversion of conv. debt) + (Shares issuable from stock options)]

This question tested from Session 8, Reading 32, LOS h

Question 110 - #95965

Your answer: A was incorrect. The correct answer was C) -$1,300.

Page 187: Financial Reporting and Analysis_The Income Statement, Balance Sheet, and Cash Flow Statement

The format of the question information suggests the use of the direct cash flow method. In this method, depreciation is not a component of cash flow from operations. Cash flow from operations = (all numbers in thousands of dollars) 45 – 17 – 22 – 6.3 – 1.0 = -1.3, or -$1,300.

This question tested from Session 8, Reading 34, LOS a

Question 111 - #97960

Your answer: A was incorrect. The correct answer was B) convertible bonds.

Simple capital structures do not include any potentially dilutive securities (a security that could decrease earnings per share if exercised). Convertible bonds are potentially dilutive.

This question tested from Session 8, Reading 32, LOS g

Question 112 - #96591

Your answer: A was incorrect. The correct answer was B) 86 days.

2008 expected days of sales outstanding is 66 [365 / (5.0 × 1.1)], 2008 days of inventory on hand is 96 [365 / (4.0 × 0.95)], and 2008 days of payables is 76 [365 / (6.0 × 0.8)]. Expected cash conversion cycle is 86 days [66 days of sales outstanding + 96 days of inventory on hand – 76 days of payables].

This question tested from Session 8, Reading 35, LOS e

Question 113 - #97755

Your answer: A was incorrect. The correct answer was C) $2.04.

Lawson’s basic EPS ((net income – preferred dividends) / weighted average common shares outstanding) is ($1,060,000 – (2,000 × $1,000 × 0.08)) / 420,000 = $2.14. To calculate diluted EPS the convertible preferred shares are presumed to have been converted, the preferred dividends paid are added back to the numerator of the EPS equation, and the additional common shares are added to the denominator of the equation. Lawson’s diluted EPS is $1,060,000 / (420,000 + 100,000) = $2.04.

This question tested from Session 8, Reading 32, LOS g

Question 114 - #97715

Your answer: A was correct!

Available-for-sale securities are reported on the balance sheet at fair value. The unrealized gains and losses bypass the income statement and are reported as a component of stockholders’ equity as a part of other comprehensive income.

This question tested from Session 8, Reading 32, LOS j

Question 115 - #97954

Your answer: A was incorrect. The correct answer was C) 41,550,000.

Outstanding all year 10,000,000 × 2 × 2 × 1 40,000,000 Outstanding for 0.75 years 500,000 × 2 × 2 × 0.75 1,500,000

Page 188: Financial Reporting and Analysis_The Income Statement, Balance Sheet, and Cash Flow Statement

This question tested from Session 8, Reading 32, LOS g

Question 116 - #97894

Your answer: A was incorrect. The correct answer was C)

Rushford’s basic EPS (net income / weighted average common shares outstanding) is $16,500,000 / 300,000 = $55.00. Diluted EPS is calculated under the assumption that the convertible bonds were converted into common stock, the bond interest net of tax is restored to net income, and the additional common shares are added to the denominator of the equation. Rushford’s diluted EPS is [$16,500,000 + (50,000 × $1,000 × 0.09)(1 - .40)] / (300,000 + (50,000 × 2) = $48.00.

This question tested from Session 8, Reading 32, LOS h

Question 117 - #97813

Your answer: A was incorrect. The correct answer was C) antidilutive securities.

Antidilutive securities, upon exercise, increase basic EPS or decrease per share losses. Shares from conversion are not included in the calculation of basic or diluted EPS.

This question tested from Session 8, Reading 32, LOS h

Question 118 - #98020

Your answer: A was incorrect. The correct answer was C) Only one is operating.

Since Pinto is a nonfinancial firm, dividends received would be considered a nonoperating component. An increase in cost of goods sold would be considered a part of normal operations.

This question tested from Session 8, Reading 32, LOS e

© 2010 Kaplan Schweser

Outstanding for 0.25 years 100,000 × 2 × 0.25 50,000 Weighted average number of shares for year: 41,550,000

$55.00 $48.00

Page 189: Financial Reporting and Analysis_The Income Statement, Balance Sheet, and Cash Flow Statement

Kaplan Schweser Printable Exams - 2011 CFA Level 1

You can print this page by going to file -> print in your internet browser.

Question 1 - #97470

Favor, Inc.’s capital and related transactions during 2005 were as follows:

On January 1, $1,000,000 of 5-year 10% annual interest bonds were issued to Cover Industries in exchange for old equipment owned by Cover. On June 30, Favor paid $50,000 of interest to Cover. On July 1, Cover returned the bonds to Favor in exchange for $1,500,000 par value 6% preferred stock. On December 31, Favor paid preferred stock dividends of $45,000 to Cover.

Favor, Inc.’s cash flow from financing (CFF) for 2005 (assume U.S. GAAP) is:

Question 2 - #97791

An analyst has gathered the following information about a company:

A) -$95,000.B) -$45,000.C) -$1,045,000.D) -$1,095,000.E)F)

Balance Sheet AssetsCash 100Accounts Receivable 750Marketable Securities 300Inventory 850Property, Plant & Equip 900Accumulated Depreciation (150)

Total Assets 2750

Liabilities and EquityAccounts Payable 300Short-Term Debt 130Long-Term Debt 700Common Stock 1000Retained Earnings 620

Total Liab. and Stockholder's equity 2750

Income Statement Sales 1500

Page 190: Financial Reporting and Analysis_The Income Statement, Balance Sheet, and Cash Flow Statement

Determine the current ratio and the cash ratio.

Question 3 - #97373

To convert an indirect statement of cash flows to a direct basis, the analyst would:

Question 4 - #97739

Common size income statements express all income statement items as a percentage of:

Question 5 - #97973

The standard equation for computing basic earnings per share (EPS) is:

Question 6 - #97394

Use the following information to calculate cash flows from operations using the indirect method.

Net Income: $12,000

COGS 1100Gross Profit 400SG&A 150Operating Profit 250Interest Expense 25Taxes 75Net Income 150

Current Ratio Cash Ratio

A) 1.98 1.86

B) 4.65 0.93

C) 2.67 1.07

A) subtract increases in inventory from cost of goods sold. B) add decreases in accounts receivables to net sales. C) add increases in accounts payable to cost of goods sold.

A) net income.B) sales.C) assets.

A) [Net Income − Common Dividends] / Weighted Average Number of Common Shares Outstanding.B) [Sales − Cost of Goods Sold] / Number of Preferred Shares Outstanding.C) [Net Income – Preferred Dividends] / Weighted Average Number of Common Shares Outstanding.

Page 191: Financial Reporting and Analysis_The Income Statement, Balance Sheet, and Cash Flow Statement

Depreciation Expense: $1,000 Loss on sale of machinery: $500 Increase in Accounts Receivable: $2,000 Decrease in Accounts Payable: $1,500 Increase in Income taxes payable: $500 Repayment of Bonds: $3,000

Question 7 - #97792

Selected information from Gerrard, Inc.’s financial activities in the most recent year was as follows:

Net income was $330,000.

The tax rate was 40%.

700,000 shares of common stock were outstanding on January 1.

The average market price per share for the year was $6.

Dividends were paid during the year.

2,000 shares of 8% $500 par value preferred shares, convertible into common shares at a rate of 200 common shares for each preferred share, were outstanding for the entire year.

200,000 shares of common stock were issued on March 1.

Gerrard, Inc.’s diluted earnings per share (diluted EPS) was closest to:

Question 8 - #96535

Part 1) The return on equity (ROE) for 20X3 and 20X4 respectively is:

Part 2) If the company’s net profit margin declines to 0.10 in 20X5, what total asset turnover would be needed in order to maintain the same ROE as in 20X4, assuming there is no change in the financial leverage multiplier?

A) Increase in cash of $10,500.B) Increase in cash of $7,500.C) Increase in cash of $9,500.

A) $0.289.B) $0.261.C) $0.197.

Ratio 20X3 20X4 Net profit margin 0.15 0.18 Total asset turnover 1.60 1.75 Financial leverage multiplier 1.00 1.50

A) 24% and 47%. B) 24% and 8%. C) 8% and 24%.

Page 192: Financial Reporting and Analysis_The Income Statement, Balance Sheet, and Cash Flow Statement

Question 9 - #98036

Assume that the exercise price of an option is $11, and the average market price of the stock is $16. Assuming 1,039 options are outstanding during the entire year, what is the number of shares to be added to the denominator of the Diluted EPS?

Question 10 - #98037

At the beginning of 20X7, Bryan’s Bakery Company purchased a secret cookie recipe for $25,000. In addition, Bryan developed a new cake recipe at a cost of $5,000. Bryan expects to use both recipes indefinitely; however, the useful (economic) life of similar recipes has been 10 years. Assuming straight-line amortization, what amount of recipe expense should Bryan report for the year ended 20X7 and what amount should Bryan report as assets related to these recipes on its balance sheet at the end of 20X7?

Question 11 - #97790

GTO Corporation purchased all of the common stock of Charger Company for $4 million. At the time, Charger reported total assets of $3 million and total liabilities of $1 million. At the acquisition date, the fair value of Charger’s assets was $3.5 million and the fair value of Charger’s liabilities was $1.3 million. What amount of goodwill should GTO report as a result of the acquisition and is it necessary for GTO to amortize the goodwill?

Question 12 - #97364

Use the following data from Delta's common size financial statement to answer the question:

A) 2.50. B) 1.50. C) 3.15.

A) 325.B) 714.C) 1,039.

Recipe expense Balance sheet

A) $5,000 $25,000

B) $7,500 $22,500

C) $3,000 $30,000

Goodwill Amortization required

A) $1.8 million Yes

B) $2.2 million No

C) $1.8 million No

Page 193: Financial Reporting and Analysis_The Income Statement, Balance Sheet, and Cash Flow Statement

What is Delta's after-tax return on equity?

Question 13 - #96771

What is a company’s equity if their return on equity (ROE) is 12%, and their net income is $10 million?

Question 14 - #97980

As of the beginning of the year HalfPass Productions, Inc., had the following complex capital structure:

3,000,000 common shares outstanding.

175,000 options with an exercise price of $22.

250,000 warrants with an exercise price of $18.

During the year:

On March 1, the company issued 100,000 new shares of common stock.

On July 1, the board of directors declared a 15% stock dividend.

On September 1, the company repurchased 125,000 shares.

Net income (after-tax) for the year was $7,500,000.

The company paid common dividends of $2,750,000 and preferred dividends of $1,300,000.

The average market price for the common stock was $25 per share.

Assume the fiscal year is January 1 through December 31. At year end, HalfPass’s basic EPS is closest to:

Earnings after taxes = 18%Equity = 40%Current assets = 60%Current liabilities = 30%Sales = $300Total assets = $1,400

A) 18.0%.B) 5.0%.C) 9.6%.

A) $83,333,333.B) $120,000,000.C) $1,200,000.

A) $1.66.B) $1.77.C) $1.94.

Page 194: Financial Reporting and Analysis_The Income Statement, Balance Sheet, and Cash Flow Statement

Question 15 - #97248

For the year ended December 31, 2007, Challenger Company reported the following financial information:

Calculate cash flow from operating activities using the direct method and the indirect method.

Question 16 - #97806

An analyst has gathered the following information about a company:

Revenue $100,000 Cost of goods sold (40,000) Cash operating expenses (20,000) Depreciation expense (5,000) Tax expense (3,000) Net income $32,000 Increase in accounts receivable $7,500 Decrease in inventory $2,500 Increase in short-term notes payable $3,000 Decrease in accounts payable $1,000

Direct method Indirect method

A) $31,000 $31,000

B) $31,000 $34,000

C) $34,000 $34,000

Balance Sheet AssetsCash 100Accounts Receivable 750Marketable Securities 300Inventory 850Property, Plant & Equip 900Accumulated Depreciation (150)

Total Assets 2750

Liabilities and EquityAccounts Payable 300Short-Term Debt 130Long-Term Debt 700Common Stock 1000Retained Earnings 620

Total Liab. and Stockholder's equity 2750

Income Statement Sales 1500

Page 195: Financial Reporting and Analysis_The Income Statement, Balance Sheet, and Cash Flow Statement

What is the receivables collection period?

Question 17 - #95419

A U.S. company uses the LIFO method to value its inventory for their income tax return. For its financial statements prepared for shareholders, the company may:

Question 18 - #97807

The cash conversion cycle is the:

Question 19 - #119453

How would the collection of accounts receivable most likely affect the current and cash ratios?

Question 20 - #97681

McQueen Corporation prepared the following common-size income statement for the year ended December 31, 20X7:

COGS 1100Gross Profit 400SG&A 150Operating Profit 250Interest Expense 25Taxes 75Net Income 150

A) 243.B) 183.C) 365.

A) only use the LIFO method.B) use any other inventory method under generally accepted accounting principles (GAAP).C) use the FIFO method, but must disclose a LIFO reserve.

A) length of time it takes to sell inventory.B) sum of the time it takes to sell inventory and the time it takes to collect accounts receivable.

C) sum of the time it takes to sell inventory and collect on accounts receivable, less the time it takes to pay for credit purchases.

Current ratio Cash ratio

A) Increase Increase

B) No effect Increase

C) No effect No effect

Page 196: Financial Reporting and Analysis_The Income Statement, Balance Sheet, and Cash Flow Statement

For 20X7, McQueen sold 250 million units at a sales price of $1 each. For 20X8, McQueen has decided to reduce its sales price by 10%. McQueen believes the price cut will double unit sales. The cost of each unit sold is expected to remain the same. Calculate the change in McQueen’s expected gross profit for 20X8 assuming the price cut doubles sales.

Question 21 - #97876

Given the following income statement and balance sheet for a company:

What is the gross profit margin?

Sales 100% Cost of goods sold 60% Gross profit 40%

A) $150 million increase.B) $80 million increase.C) $50 million increase.

Balance Sheet Assets Year 2003 Year 2004Cash 500 450Accounts Receivable 600 660Inventory 500 550Total CA 1300 1660Plant, prop. equip 1000 1250Total Assets 2600 2910

LiabilitiesAccounts Payable 500 550Long term debt 700 1102Total liabilities 1200 1652

EquityCommon Stock 400 538Retained Earnings 1000 720Total Liabilities & Equity 2600 2,910

Income Statement Sales 3000Cost of Goods Sold (1000)Gross Profit 2000SG&A 500Interest Expense 151EBT 1349Taxes (30%) 405Net Income 944

A) 0.333.B) 0.472.C) 0.666.

Page 197: Financial Reporting and Analysis_The Income Statement, Balance Sheet, and Cash Flow Statement

Question 22 - #97955

A firm had the following numbers of shares outstanding during the year:

Based on this information, what is the weighted number of shares outstanding for the year?

© 2010 Kaplan Schweser

Beginning of year 8,000,000 shares Issued on April 1 750,000 shares Paid stock divided of 20% on July 1 Issued on October 1 100,000 shares Purchased Treasury stock November 1 1,000,000 shares Split 2 for 1 on December 31

A) 20,783,333.B) 20,266,667.C) 42,444,444.

Page 198: Financial Reporting and Analysis_The Income Statement, Balance Sheet, and Cash Flow Statement

Kaplan Schweser Printable Exams - Test Management Exam 22

Test ID#: 7790114

Question 1 - #97470

Your answer: A was incorrect. The correct answer was B) -$45,000.

Only the preferred stock dividends paid would be considered CFF. Issuing bonds in exchange for equipment and exchanging bonds for stock are both noncash transactions that should be disclosed in a footnote to the Statement of Cash Flows. Interest paid is an operating cash flow under U.S. GAAP.

This question tested from Session 8, Reading 34, LOS e

Question 2 - #97791

Your answer: A was incorrect. The correct answer was B)

Current ratio = [100(cash) + 750(accounts receivable)+ 300(marketable securities) + 850(inventory)] / [300(AP) + 130(short term debt)] = (2000 / 430) = 4.65

Cash ratio = [100(cash) + 300(marketable securities)] / [300(AP) + 130(short term debt)] = (400 / 430) = 0.93

This question tested from Session 8, Reading 35, LOS d

Question 3 - #97373

Your answer: A was incorrect. The correct answer was B) add decreases in accounts receivables to net sales.

A decrease in accounts receivable represents an increase in cash so this should be added to sales. Increases in accounts payable represent an increase in cash so these should be subtracted from cost of goods sold. Increases in inventory represent a use of cash so these would be added to cost of goods sold.

This question tested from Session 8, Reading 34, LOS f

Question 4 - #97739

Your answer: A was incorrect. The correct answer was B) sales.

Common size income statements express all income statement items as a percentage of sales. Note that common size balance sheets express all balance sheet accounts as a percentage of total assets.

This question tested from Session 8, Reading 35, LOS c

Question 5 - #97973

Your answer: A was incorrect. The correct answer was C) [Net Income – Preferred Dividends] / Weighted Average Number of Common Shares Outstanding.

The basic EPS calculation does not consider the effects of any dilutive securities in the computation.

4.65 0.93

Page 199: Financial Reporting and Analysis_The Income Statement, Balance Sheet, and Cash Flow Statement

Basic EPS = [Net Income – Preferred Dividends]/Weighted Average Number of Common Shares Outstanding.

This question tested from Session 8, Reading 32, LOS g

Question 6 - #97394

Your answer: A was correct!

Cash flow from operations would be calculated as +Net Income $12,000 + Depreciation $1,000 + Loss on sale of machinery $500 − A/R $2,000 − A/P $1,500 + Income taxes payable $500 = $10,500.

Repayment of Bonds is a financing activity and would not be included with operating activities. Depreciation is not a cash flow activity and is therefore always added back to net income to calculate CFO. The loss on the sale of machinery is not a cash outflow so it is also added back to calculate CFO. Accounts receivable is subtracted when there is an increase as this increases net income but does not affect cash.

This question tested from Session 8, Reading 34, LOS e

Question 7 - #97792

Your answer: A was incorrect. The correct answer was B) $0.261.

To compute Gerrard’s basic earnings per share (EPS) ((net income – preferred dividends) / weighted average common shares outstanding), the weighted average common shares outstanding must be computed. 700,000 shares were outstanding from January 1, and 200,000 shares were issued on March 1, so the weighted average is 700,000 + (200,000 × 10 / 12) = 866,667. Basic EPS was $330,000 − (2,000 × $500 × 0.08)) / 866,667 = $0.289.

If the convertible preferred shares were converted to common stock, 2,000 × 200 = 400,000 additional common shares would have been issued and dividends on the preferred stock would not have been paid. Diluted EPS was $330,000 / (866,667 + 400,000) = $0.261.

This question tested from Session 8, Reading 32, LOS h

Question 8 - #96535

Part 1) Your answer: A was correct!

ROE for 20X3: 0.15 × 1.6 × 1.0 = 24%. Similarly, for 20X4, ROE will be 47%.

This question tested from Session 8, Reading 35, LOS f

Part 2) Your answer: A was incorrect. The correct answer was C) 3.15.

ROE for 20X5 can be computed as follows: 0.1 × asset turnover × 1.5 = 0.4725. Hence, desired asset turnover will be 3.15.

This question tested from Session 8, Reading 35, LOS f

Question 9 - #98036

Your answer: A was correct!

Page 200: Financial Reporting and Analysis_The Income Statement, Balance Sheet, and Cash Flow Statement

(1,039 options)($11) = $11,429 $11,429 / $16 per share 1039 − 714 = 325 shares or [(16 − 11) / 16]1,039 = 325.

This question tested from Session 8, Reading 32, LOS h

Question 10 - #98037

Your answer: A was incorrect. The correct answer was B)

The recipes are intangible assets. The purchased cookie recipe is capitalized and amortized over 10 years at $2,500 per year ($25,000 cost / 10 years). Since the cake recipe was developed internally, it is expensed immediately. Thus, total expense for 20X7 is $7,500 ($2,500 amortization expense + $5,000 cake recipe expense). The balance sheet value of the purchased recipe at the end of 20X7 is $25,000 – $2,500 = $22,500.

This question tested from Session 8, Reading 33, LOS c

Question 11 - #97790

Your answer: A was incorrect. The correct answer was C)

The acquisition goodwill is equal to $1.8 million [$4 million purchase price – $2.2 million fair value of net assets acquired ($3.5 million assets at fair value – $1.3 million liabilities at fair value)]. Under IFRS or U.S. GAAP, goodwill is not amortized but is subject to an annual impairment test.

This question tested from Session 8, Reading 33, LOS e

Question 12 - #97364

Your answer: A was incorrect. The correct answer was C) 9.6%.

Net income after taxes = 300 × 0.18 = 54 Equity = 1400 × 0.40 = 560 ROE = Net Income / Equity = 54 / 560 = 0.0964 = 9.6%

This question tested from Session 8, Reading 35, LOS d

Question 13 - #96771

Your answer: A was correct!

One of the many ways ROE can be expressed is: ROE = net income / equity

0.12 = $10,000,000 / equity

Equity = $10,000,000 / 0.12 = $83,333,333

This question tested from Session 8, Reading 35, LOS f

Question 14 - #97980

$7,500 $22,500

$1.8 million No

Page 201: Financial Reporting and Analysis_The Income Statement, Balance Sheet, and Cash Flow Statement

Your answer: A was incorrect. The correct answer was B) $1.77.

The question is asking for basic EPS. Thus we can ignore the dilutive options and warrants. Basic EPS = (net income – preferred dividends) / weighted average common shares outstanding

The numerator = $7.5 million – $1.3 million = $6.2 million

Calculating the denominator is a bit more work (calculation detailed in table below):

Average shares = 42,050,000 / 12 = 3,504,167 Basic EPS = $6.2 million / 3.504 million = $1.77

This question tested from Session 8, Reading 32, LOS g

Question 15 - #97248

Your answer: A was correct!

CFO is the same under both methods, the only difference is presentation. Direct method: $92,500 cash collections ($100,000 revenue – $7,500 increase in receivables) – $38,500 cash paid to suppliers (– $40,000 COGS + $2,500 decrease in inventory – $1,000 decrease in payables) – $20,000 cash operating expenses – $3,000 tax expense = $31,000. Indirect method: $32,000 net income + $5,000 depreciation expense – $7,500 increase in receivables + $2,500 decrease in inventory – $1,000 decrease in payables = $31,000. The increase in short-term notes payable is a financing activity.

This question tested from Session 8, Reading 34, LOS d

Question 16 - #97806

Your answer: A was incorrect. The correct answer was B) 183.

Receivables turnover = 1,500(sales) / 750(receivables) = 2.0

Average receivables collection period = 365 / 2 = 182.5 or 183

This question tested from Session 8, Reading 35, LOS d

Question 17 - #95419

Your answer: A was correct!

The LIFO conformity rule in the U.S. requires firms to use LIFO for their financial statements if they use LIFO for income tax purposes.

This question tested from Session 8, Reading 32, LOS d

Event Notes Million Shares # months outstanding Total Beginning Bal. (BB) 3.000 12 36.000 New issue (March 01) 0.100 10 1.000 Stock Dividend 15% on BB 0.450 12 5.400 Stock Dividend 15% on new issue 0.015 10 0.150 Repurchase (Sept .1) -0.125 4 -0.500

Total 42.050

Page 202: Financial Reporting and Analysis_The Income Statement, Balance Sheet, and Cash Flow Statement

Question 18 - #97807

Your answer: A was incorrect. The correct answer was C) sum of the time it takes to sell inventory and collect on accounts receivable, less the time it takes to pay for credit purchases.

Cash conversion cycle = (average receivables collection period) + (average inventory processing period) − (payables payment period)

This question tested from Session 8, Reading 35, LOS d

Question 19 - #119453

Your answer: A was incorrect. The correct answer was B)

Collecting receivables increases cash and decreases accounts receivable. Thus, current assets do not change and the current ratio is unaffected. Because the numerator of the cash ratio only includes cash and marketable securities, collecting accounts receivable increases the cash ratio.

This question tested from Session 8, Reading 35, LOS c

Question 20 - #97681

Your answer: A was incorrect. The correct answer was C) $50 million increase.

20X7 gross profit is equal to $100 million ($1 × 250 million units sold × 40% gross profit margin). The 10% price cut to $0.90 will increase cost of goods sold to 67% of sales [COGS=0.6($1) = $0.60; $0.60 / $0.90 = 67%.]. As a result, gross profit will decrease to 33% of sales. If unit sales double in 20X8, gross profit will equal $150 million ($0.90 × 500 million units × 33% gross profit margin). Therefore, gross profit will increase $50 million ($150 million 20X8 gross profit – $100 million 20X7 gross profit).

This question tested from Session 8, Reading 35, LOS h

Question 21 - #97876

Your answer: A was incorrect. The correct answer was C) 0.666.

Gross profit margin = (gross profit / net sales) = (2,000 / 3,000) = 0.666

This question tested from Session 8, Reading 35, LOS d

Question 22 - #97955

Your answer: A was incorrect. The correct answer was B) 20,266,667.

This question tested from Session 8, Reading 32, LOS g

No effect Increase

Outstanding all year 8,000,000 × 1.2 × 2 × 1.0 19,200,000 Outstanding for 0.75 years 750,000 × 1.2 × 2 × 0.75 1,350,000 Outstanding for 0.25 years 100,000 × 2 × 0.25 50,000 Retired for 2 months -1,000,000 × 2 × (2/12) –333,333

Weighted average number of shares for year: 20,266,667

Page 203: Financial Reporting and Analysis_The Income Statement, Balance Sheet, and Cash Flow Statement

© 2010 Kaplan Schweser